Medchem 2 Final

Pataasin ang iyong marka sa homework at exams ngayon gamit ang Quizwiz!

Cefuroxime contains the "____" group which shields the drug from beta-lactamases

"Oxime"

Which group is responsible for the 3rd generation drugs to be highly resistant to Beta-Lactamses?

"Oxime" (methoxy imino group)

Describe the intrinsic pathway

- Factor 12a stimulates factor 11a - Factor 11a then stimulates 9a - Factor 8a helps 9a to proceed to 10a and follow common pathway

Describe the extrinsic pathway

- Factor 7 exposed to tissue factor which activates 7a - 7a then stimulates 10a to proceed to common pathway - 10a works with 5a to activate 2 to 2a (prothrombin to thrombin) - 2a acts on factor 1 to 1a (fibrinogen to fibrin) - Fibrin comes out of the blood to form strong clot with 13a

ADME of Linezolid and Tedizolid:

-100% bioavailability (Can be used orally/IV) -Good CSF penetration

What is the first and second line drugs for lice?

-1st Permethrin -2nd Malathion, Spinosad, and Benzyl alcohol

What are the ADME properties of Tigecycline?

-A: IV only because it is very limited -D: wide tissue distribution -M: Long half-life -E: Bile & Urine

Adverse reactions for CCB's (Class IV):

-AV nodal block -Headache -Dizziness -Fatigue -Nausea/Constipation

List the GPIIB/IIIa inhibitors:

-Abciximab -Eptifibatide

Which antiplatelet drugs are Glycoprotein IIb-IIIa receptor antagonists?

-Abciximab -Eptifibatide -Tirofiban -Defibrotide

Define Nosocomial infection and give an example of it.

-Acquired during hospital stay -Often difficult to treat (resistant strains)

What are the major Gram + anaerobes organisms covered by Penicillin G?

-Actinomyces -Propionibacterium acnes -Peptostreptococcus -Clostridium (Not C. diff)

Which drugs are considered as "Chain Terminators"?

-Acyclovir -Valacyclovior

Name the drugs that inhibit HSV replication.

-Acyclovir -Valacyclovir -Ganciclovir -Valganciclovir -Cidofovir -Penciclovir -Femciclovir -Trifluridine -Idoxuridine (Hint: -clovir/uridine)

Name the drugs that inhibit Hep B replication.

-Adefovir -Tenofovir -Entecavir -Telbivudine -Lamivudine

What is the coverage of Cefiderocol?

-Aerobic gram - -ESBL, KPC

Name the Nematodes:

-Albendazole/mebendazole -Pyrantel -Ivermectin

What is the function of sex pilus?

-Allows attachment of one bacterium to another -Involved in Horizontal transfer of genes -Resistance

List some drugs decrease metabolism of warfarin due to CYP2C9 inhibition (Remember: they can increase risk of bleeding)

-Amiodarone -Azole antifungals -Clopidogrel -Fluoxetine -Metronidazole -Cimetidine

What are sulbactam and tazobactam combined with?

-Ampicillin + Sulbactam -Tazobactam + Piperacilin

What are the differences between ampicillin and amoxicillin in terms of oral absorption?

-Ampicillin: incomplete oral absorption -Amoxicillin: complete oral absorption

Aside from Rx induced, Arrhythmias may be caused by:

-Anatomical defects -Electrolyte imbalances -Ischemia -Stress -Non-Rx substances (Caffeine, drugs of abuse, antiemetics, nicotine, alcohol)

Adverse reactions of class IA drugs:

-Anticholinergic effects (dry mouth, urinary retention, confusion) -Arrhythmias -GI (Nausea/diarrhea) -Dizziness -Rash

Which drug classes are most common to cause prolonged QT interval? (VERY IMPORTANT TO KNOW)

-Antipsychotics -Antidepressants -Antibacterials -Antiarrhythmics

Atrial arrhythmias occur in the Atrium, what are the 2 different types of atrial arrhythmias?

-Atrial Flutter -Atrial Fibrillation (A-Fib)

What are some of the common approaches to minimize the spread of resistance?

-Avoid inappropriate use -Use narrow spectrum when possible -Reserve newer antibiotics for special cases -Minimize giving antibiotics to livestock -Drugs should not be flushed down the toilet -Dont eat antibiotic-treated food -Get vaccinated (Hib, Prevnar) -Drug withdrawal (stop using antibiotic for a while, resistance strain may go away)

What are the antibiotics that inhibit protein synthesis?

-Bactericidal: Aminoglycosides -Bacteriostatic: Tetracyclines, Macrolides, Clindamycin, Chloramphenicol, Streptogramins, Oxazoladinones (Linezolid)

Which drugs increases hepatic clearance of warfarin due to CYP2C9 induction? (This means reducing efficacy)

-Barbiturates -Carbamazepine -Rifampin

Which drugs inhibit the cell wall synthesis?

-Beta lactams (Penicillins, cephalosporins, Monobactems, Carbapenems) -Others: Fosfomycin, Cycloserine, Bacitracin, Vancomycin

What advantages (bugs covered) do the 2nd generation drugs have over the 1st generation?

-Better Gram - coverage than 1st generation -Gram + coverage is less reliable in 2nd generation than 1st gen -Less susceptible to beta-lactamases than the 1st gen -Some 2nd gen have better CSF penetration than 1st gen (2nd gen is a diverse group)

How are 1st generation cephalosporins different when comparing coverage to Ampicillin/Amoxicillin?

-Better Staph coverage -No Enterococci coverage -No anaerobic coverage -Fewer gram - covered

What are the advantages of low molecular weight heparin when compared to high molecular weight?

-Better absorption -Less nonspecific interactions (decreasing toxicity) -More predictable long T1/2 (renal) -aPTT usually does not need to be monitored (more predictable activity) -Less side effects

Where can transposons "jump"?

-Between plasmid and chromosomal DNA -Between bacterial species -In response to antibiotic exposure

MOA of Clindamycin

-Binds to 50S, inhibits peptidyl transferase, inhibits protein synthesis

MOA of Alteplase:

-Binds to newly formed thrombi with high affinity -Gains potent plasminogen activator activity by binding to fresh thrombus and undergoing conformational change

What are the adverse effects of heparin?

-Bleeding -Heparin-Induced Thrombocytopenia (HIT)

Adverse effects of Glycoprotein IIb-IIIa receptor antagonists

-Bleeding -Thrombocytopenia -Hypotension -Bradycardia

What are the contraindications of warfarin?

-Bleeding tendencies -Recent eye, brain or spinal surgery -History of warfarin-induced skin necrosis -Pregnancy -Uncontrolled hypertension -Unsupervised patients with psychosis, senility, alcoholism, or lack of cooperation, and in patients in risk of falling

Adverse reactions for Beta-blockers (Class II):

-Bronchospasms -Cold extremities (feeling cold) -Bradycardia -Heart block -Insomnia -Depression

What are the other names for Trifluridine?

-CF3 deoxyurine analog -Trifloro deoxythymidine analogue

What is the coverage of Echinocandins?

-Candida sp. -Aspergillus sp.

Adverse reactions of Class IC agents:

-Cardiac (arrhythmias, heart failure) -CNS (dizziness, tremor, blurred vision) -GI (Nausea, CYP450 inhibition)

Adverse reactions of Amiodarone:

-Cardiac issues (negative inotrope, bradycardia, hypotension) -Bradycardia -Hypotension -Pulmonary issues -Endocrine (thyroid) -Hepatic -Neurological

How can normal flora be harmful?

-Causes disease if it escapes its normal location •Bladder infection (E. coli) •Skin puncture wounds •Dental work -Can overgrow and cause disease in immuno-compromised patients •AIDS pts •Cancer pts under chemotherapy •Transplant pts

What is special about the -OH group within the structure of Erythromycin?

-Causes formation of Cyclic ketal that stimulates motilin receptors -Cyclic Ketal is responsible for causing stomach cramps which is an adverse effect

What are the oral route 3rd generation cephalosporins?

-Cefixime -Cefdinir -Cefditoren -Ceftibuten -Cefpodoxime

What are the parenteral route 3rd generation cephalosporins?

-Ceftriaxone -Cefotaxime -Ceftazidime

What are the 2nd generation cephalosporins?

-Cefuroxime (IV/IM) -Cefuroxime axetil (PO) -Cefaclor (PO) -Cefprozil (PO)

What are features of Prokaryotes (bacterial cells)?

-Cell Wall -No nucleus -No cytoplasmic organelles -Ribosomes: 30s + 50s = 70s (odd) -Reproduction by binary fission

Why is mycobacterium tuberculosis difficult to treat?

-Cell envelope is highly resistant to most antibiotics -Grows very slowly (slow growth rate) -Abundance of membrane-bound efflux pump -Replicate inside of alveolar macrophages

Understand Prokaryote (Bacteria)

-Cell wall -No nucleus -No cytoplasmic organelles -Ribosomes: 30S + 50S = 70S (odd) -Reproduction = Binary fission

What are the names of 1st generation Cephalosporins?

-Cephalexin (PO) -Cefadroxil (PO) -Cefazolin (IV/IM)

How did changes in PBPs in MRSA and S. Pneumonia led to resistance to Beta-Lactams?

-Change in PBP2a -Drug cannot reach Ser 403 -Cannot inactivate the enzyme -Most beta-lactams are ineffective

How does resistance to macrolides occur?

-Changes in 50S (mutation in 50S and methylation of 23S rRna) -Efflux pumps -Production of Esterases that inactivate macrolides

What are the mechanisms by which bacteria becomes resistant to tetracyclines?

-Changes in porins (less likely to affect Minocycline) -Drug efflux pumps -Ribosomal resistance or ribosomal protection (changes in ribosomal proteins which decreases drug affinity)

Adverse effects of Quinine:

-Cinchonism (eyes, ear, skin) -Release of insulin (hypoglycemia) -Hemolytic anemia if G6PDHase defficient

What is the coverage of Polymyxins?

-Citrobacter -E. Coli (including ESBL, KPC, MBL) -Enterobacter -Klebsiella -Pseudomonas (Gram negative coverage only)

What is the difference in coverage between Clarithromycin and Azithromycin?

-Clarithromycin: MSSA, MAC (for HIV), URTI, H. pylori -Azithromycin: STDs, CAP (community acquired pneumonia), URTI (include H. influenza), H. pylori

List ADP receptor inhibitors:

-Clopidogrel -Prasugrel -Ticagrelor

What are the different shapes of bacteria?

-Coccus ("sphere") -Bacillus ("rod shaped") -Streptobacillus (chain) -Streptococcus (chain) -Staphylococcus (clusters) -Spirochete (thinner, mobile spirals) -Spirillum (rigid) -Vibrio (curved)

What are the benefits of gut flora?

-Complex ecosystem (>500 species) -Protects from pathogenic bacteria -Stimulates & trains our immune system -Nutritional- digest fiber, produce Vit K, B12 -Link b/w a lack of bacterial flora with obesity and other diseases

What are the advantages of Fondaparinux vs biological (HMWH + LMWH) heparin?

-Consistent dosage form (no monitoring required) -Specific for Factor Xa -Better pharmacokinetics

What are the anti-coagulant drug classes?

-Coumarins (Vitamin K antagonists) -Heparin based (biological and synthetic) -Factor Xa inhibitors -Direct Thrombin Inhibitors

MOA of Clopidogrel:

-Covalently (irreversibly) binds to P2Y ADP receptor -Binds receptor through disulfide bond -Platelet aggregation is inhibited

MOA of Fosfomycin

-Covalently binds and inhibits enolypruvate transferase -Inhibits the conversion of NAG to NAM

List the anti-platelet drug classes

-Cox-1 Inhibitors -PDE3 Inhibitors -ADP (P2Y) receptor antagonists -Glycoprotein IIb/IIIa receptor antagonists -Thrombin receptor (PAR-1) antagonists

Flucytosine has a narrow spectrum as it covers:

-Cryptococcus neoformans -Candida

List the Direct Thrombin Inhibitors:

-Dabigatran -Lepirudin -Desirudin -Bivalirudin -Argatroban

What 3 drugs are Semi-synthetic glcopeptides?

-Dalbavancin -Telavancin -Oritavancin

List the 2 Low molecular weight Heparin drugs (Biological heparin)

-Dalteparin -Enoxaparin

How does bacteria develop resistance to sulfonamides?

-Decrease membrane permeability or Drug efflux -Mutations in enzymes (affinity changes) -Some trains overproduce PABA

Adverse Effects of Acyclovir and Valacyclovir:

-Delirium -Tremor -Hypotension -Nephrotoxicity

Why does the Beta-Lactam ring have affinity for PBPs?

-Different species have different PBPs -Different PBPs have different binding affinities for different drugs

What are the nutritional benefits of gut flora?

-Digest fiber -Produce Vit K and B12

Which PDE3 inhibitors can be used as antiplatelet drugs?

-Dipyridamole -Cilostazol

What are the advantages to Factor Xa inhibitors?

-Directly inhibit Factor Xa -Bind to both free and clot-bound factor Xa -Do not require antithombin, AT-III -Do not require frequent monitoring, dosing -Rapid onset, good bioavailability -All Pgp substrates

What is the distribution, elimination half-life, and route of elimination of Penicillin G?

-Distribution: Widely distributed, includes CSF -Half-Life: 30-40 minutes -Elimination: Renally eliminated -1940's "Bedpan" Therapy

Adverse drug reactions of Ethambutol:

-Dose dependent optic neuritis (loss of visual acuity; red-green color blindness (reversible)) -Hyperuricemia -Rash; drug fever

What factors determine for us to not always predict in vivo response in susceptibility tests?

-Drug distribution (serum vs target tissue) -Host factors (immunity, pus) -Treatment-induced resistance (Ex: exposure to most cephalosporins will induce beta lactamases

What bacteria's are covered by Gentamicin, Tobramycin, and Amikacian?

-E. Coli -Proteus -Enterobacter -Citrobacter -Klebsiella -Providencia -Serratia -Pseudomonas

What are the major bugs covered by the combination of Piperacillin + Tazobactam?

-E. Coli (if susceptible) -Bacteroides -Pseudomonas (More than ticarcillin) -Acinetobacter

What bugs are covered with Ceftazidime + Avibactam?

-ESBL -CRE

What organisms are not covered by Aztreonam?

-ESBL -KPC -R-MBL

What is the broad-spectrum coverage of Omadacycline?

-ESBL -MRSA -Anaerobes (some)

What is the Gram - coverage of Fosfomycin?

-ESBL -KPC -Some MBL strains

Name the Anti-HIV Non-nucleoside analog DNA polymerase inhibitors

-Efavirenz -Nevirapine -Delaviridine -Etravirine -Rilpivirine

What are the 4 unique side effects of Foscarnet?

-Electrolyte abnormalities -Bone abnormalities -Nephrotoxicity -CNS effects

For a bacteria to reproduce, what occurs during Binary fission?

-Elongation and DNA replication -Septum formation -Cell separation to make 2 daughter cells

Bactericidal drugs are preferred in patients with:

-Endocarditis -Meningitis -Leukopenic pateints

What are 3 major factors that can causes thrombosis?

-Endothelial injury (Hypertension, High cholesterol, increased blood glucose in diabetes) -Abnormal blood flow -Hypercoagulability

What are the 2 types of toxins that determine virulence of some bacteria?

-Endotoxin -Exotoxin

What are the two types of toxins that determine virulence of some bacteria?

-Endotoxins -Exotoxins

What is the gram - coverage of carbapenems?

-Enterobacteriaceae (enterics) -H. Influenzae -Moraxella -Neisseria -Pseudomonas

List the Macrolide drugs:

-Erythromycin -Clarithromycin -Azithromycin -Telithromycin

What are the effects of Amiodarone?

-Extends effective refractory period in all cardiac tissue -Decreases pacemaker automaticity -AV nodal block and bradycardia

After heparin binds to antithrombin III, it inactivates which factors?

-Factor IIa (2a) (Thrombin) -Factor Xa (10a) (Also Factors IXa (9a), XIa (11a), XIIa (12a)) (Top two are main focus)

How do flagella, fimbriae, capsule, surface glycoproteins, and spores contribute to virulence?

-Flagella: Helps Bacteria move -Fimbriae: Helps bacteria attach to mucus membranes -Capsule: helps bacteria evade immune system, difficult for drugs to penetrate -Surface glycoprotein: Protects bacteria from immune system -Spores: has all genetic information that is harmful only when alive

What are the 3 MOA's involved with Oritavancin that makes it better than Vancomycin?

-Forms dimer that binds to D-Ala-D-Lac and binds it tightly -Binds to secondary site on peptide and inhibits transpeptidation -Disrupts membrane potential

Adverse reactions of Pyrazinamide:

-GI -Malaise -Arthralgias -Hepatotoxicity (serious-fatal) -Hyperuricemia

Adverse Effects of Griseofulvin:

-GI (N, V, diarrhea, heartburn) -Headache -Photosensitivity -Oral thrush

Adverse reactions of Rifampin:

-GI (may be severe) -Fever, Rash -Fluid discoloration -Flu-like syndrome in high dose regimens

Aspirin adverse effects:

-GI Bleeding (due to inhibition of synthesis of prostaglandins) -Acute renal insufficiency -Thrombocytopenia (Low platelet count) -Hepatits -Angioedema -Reye's syndrome (Avoid in children)

Adverse effects of Sulfonamides:

-GI distress -Renal problems: Crystalluria formation (patients must drink lots of water) -Hypersensitivity (Mild to Stevens-Johnson syndrome) -Bone marrow suppression (Leukopenia, thrombocytopenia) -Hemolytic anemia in patients with G6P-DH deiciency -Interactions with oral anticoagulants by increasing their effects

Adverse effects of Flucytosine:

-GI distress (due to GI flora converting to 5-FU) -Bone marrow suppression (leukopenia, thrombocytopenia) -Hepatotoxicity (reversible)

What are the adverse effects of the common Macrolides?

-GI disturbances (Erythromycin with the cyclic ketal structures penetrates motilin receptors to cause this) -Increase in transaminases -Hepatitis (greatest risk with erythromycin) -Diarrhea -Prolong QT intervals (arrhythmias caused by Erythromycin and Clarithromycin)

Adverse effects of Amantadine and Rimantadine:

-GI irritation -Dizziness -Ataxia -Slurred speech

Adverse effects of Tetracyclines:

-GI irritation (epigastric) -Diarrhea & Colitis -Bones/teeth discoloration (pregnancy, lactation, age < 12) -Photosensitivity (least with Minocycline) -Vestibular toxicity (in Inner ear; with Minocycline)

Adverse effects of fluoroquinolones:

-GI problems (nausea, vomiting, abdominal discomfort) -Diarrhea, colitis (more common with Cipro) -CNS: mild headache, dizziness -CNS (rare): hallucinations, delirium (if taking with NSAIDs) -Photosensitivities -QT interval prolongation (Most common with moxifloxacin; must be used with caution in patients undergoing treatment for arrhythmia) -Tendinopathy (black boxed warning)

What are the adverse effects of Telavancin?

-GI: Metallic taste, nausea, vomiting, diarrhea -Rash -CNS effects: Insomnia, headache, psychiatric -Foamy urine -Nephrotoxicity (could be significant) -False increase in INR -Fetal risk (get pregnancy test before treatment) -Avoid in patients with QT prolongation

What is the broad spectrum coverage of Rifampin?

-Gram + bacteria -Legionella -Neisseria -H. pylori

Coverage of Doxycycline:

-Gram +: MSSA, MRSA (community acquired) -Atypicals: Chlamydia, Mycoplasma, Legionella, Rickettsiae (antibiotic of choice) -Anaerobes & spirals: H. Pylori, T. Pallidum, Borrelia -Non-Enteric Gm -: Moraxella, H. Influenzae, V. cholerae, Yersinia Protozoa: Plasmodium falcipuram

What kind of bugs are not covered by polymyxins?

-Gram positive -Spirals -Atypicals

What are Nitromidazoles used for?

-Gynecological infections -Pseudomembranous colitis -Abdominal wounds/infections -H. Pylori infections

What organisms are covered by Aztreonam?

-H. influenza -M. Catarrhalis -Neisseria -Enterobacteriaceae -Pseudomonas (Selective for Gram -)

Contraindication to heparin

-HIT -Hypersensitivity to heparin or pork products -Active bleeding (major) -Bleeding tendencies such as hemophilia, thrombocytopenia, or hepatic disease with hypoprothrombinemia -Uncontrolled hypertension

What is the gram - coverage of Cefepime?

-HNPEK (Haemophilus, Neisseria, Proteus, E. Coli, Klebseilla) -CAPES (Citrobacter, Acinetobacter, Providencia, Enterobacter, Serratia) (More reliable than 3rd gen) -Pseudomonas -N. Gonorrhea (No significant activity against Anaerobes)

What additional species (compared to Pen G) are covered by aminopenicillins?

-HPE Haemophilus influenza -Proteus mirabilis -E. Coli

Describe the features of fimbriae.

-Has "hair like" projections -Attaches to mucosal surface -Virulence factor

What are the criteria for drugs entering porins?

-Have to be polar (ex: aminoglycosides) -MW less than 700 Da

Adverse effects of Fluconazole:

-Headache, Nausea -Hepatotoxicity -QT prolongation

What are the toxicity issues related to penicillins?

-Hypersensitivity (regardless of previous exposure to drug (rash, anaphylaxis)) -Jarisch-Herxheimer Reaction -CNS stimulation (on high doses with kidney insufficiency)

Precautions of Aspirin:

-Hypothrombinemia -Vitamin K deficiency -Thrombotic thrombocytopenic purpura -GI lesions (stomach) -Impaired renal function (Kidneys) -Hepatic impairment (Liver)

What are the Carbapenems?

-Imipenem -Meropenem -Doripenem -Ertapenem

What can we use for treatment of Arrhythmia?

-Implantable cardioverter defibrillators (ICD) (implanted in patients at high risk for death and monitors v-fib and v tach) -Catheter ablation (heats and destroys abnormal tissue) -Drugs

What are the unique MOA that are attributed with the chlorobiphenyl group on Oritavancin?

-Increase in coverage -Increase in effectiveness -Increase in potency

Telithromycin has several -OH groups converted into keto groups in its structure and contains an "alkyl-aryl extension" What is the mechanism behind this extension?

-Increases affinity for ribosome binding -Overcomes resistance due to 23S rRNA methylation

What does the aminothiazole moiety in 3rd generation do in place of furan or benzne?

-Increases porin penetration -Increases PBP affinity -Increases Gram - coverage -Increases resistance to Beta-Lactamases

What are the adverse effects of Oritavancin?

-Inhibits CYP2C9 which increases INR -Osteomyelitis

What are the potential adverse effects associated with Cefazolin?

-Inhibits Vitamin K which causes Bleeding -Disulfuram-like reaction with Alcohol (MTD adverse effects are less severe than MTT)

When is it necessary to give antibiotic combinations?

-Initial treatment of unknown severe infections (meningitis) -Mixed infections (abdominal injury, postsurgical infections)

Virulence factors fall into two categories. What are they?

-Invasiveness -Toxicogenisis

What are the 2 categories that virulence falls under?

-Invasiveness (factors that help bacteria invade the host and stay) -Toxicogenesis

What are the other names for idocuridine?

-Iodo deoxyuridine analog -Iodo deoxy-demethylated thymidine analogue

What are the 1st line anti-tuberculosis agents?

-Isoniazid (INH) -Rifampin (RMP) -Pyrazinamide (PZA) -Ethambutol (EMB) "RIPE"

What are the major PRPs?

-Isoxazolyl Penicillins: Oxacillin (IV/IM), Cloxacillin (IM/PO), Dicloxacillin (PO) -Other: Nafcillin (IV/IM)

How is telithromycin different when it comes to building resistance towards macrolides?

-It has a bulky alkyl-aryl extension -Has second binding site, which has no affinity for efflux pumps -Overcomes some resistance

What is clavulanic acid and how does it improve coverage by aminopenicillins?

-It is a beta-lactam inhibitor -When combined with Aminopenicillin and Beta-Lactam inhibitor, coverage increases

What advice would you give to patients when taking fluoroquinolone drugs orally?

-It is ok to take with food but avoid dairy products -Separate antacids by a few hours (just like macrolides)

Name the major Azole drugs?

-Ketoconazole -Itraconazole -Fluconazole -Voriconazole -Posaconazole -Isavuconazole

Adverse effects of Entecavir:

-Lactic acidosis -Hepatomegaly

Adverse effects of Ganciclovir and Valganciclovir:

-Leukopenia -Thrombocytopenia -Mucositis -Liver dysfunction

What are Quinupristin + Dalfopristin used for?

-Life threatening infections by VRE, VRSA -Complicated skin & skin structure infection (S. aureus)

What are the 2 Oxazolidinones?

-Linezolid -Tedizolid

What are the major Gram + rods organisms covered by Penicillin G?

-Listeria -Corynebacterium

Which bugs are NEVER covered by any of the cephalosporins?

-Listeria -Enterococci (most) -Anaerobes (with exceptions) -Atypicals

Azithromycin has an additional insert in it's structure unlike Erythromycin. This produces:

-Longer t1/2 -Increased stability -No abdominal cramps

When sympathetic effects are activated what can occur?

-Lowering of threshold potential -Shorter duration action potentials -More rapid depolarization

What coverage do Carbapenems not cover in terms of resistant strains?

-MRSA -KPC -MBL

What is the coverage of Daptomycin?

-MRSA -VRE (both Enterococci Faecalis & Faecium) -Streptococci (except Viridians) (Remember: Gram positive coverage only)

What is the coverage of Vancomycin? (Remember only Gram +)

-MSSA -MRSA -Enterococcus Faecalis -Clostridium spp. (Including C. difficile) (Oral only despite poor oral absorption)

What is the coverage of Tigecycline?

-MSSA -MRSA -Streptococci (including resistant strains) -Enterococci -Enterobacteriaceae -KPC+, ESBL -Mycoplasma spp. -Anaerobes (NO PSEUDOMONAS)

What is the gram + coverage of carbapenems?

-MSSA -Strep. Pneumonia -Viridians group strep. -Enterococcus faecalis -Listeria

What are the major bugs covered by 1st generation cephalosporins?

-MSSA -Streptococci -Proteus Mirabilis (Not indole +) -E. Coli -Klebsiella spp.

How do MSSA, HA-MRSA, and CA-MRSA respond to PRPs?

-MSSA: Can use PRPs -HA-MRSA: Harder to treat -CA-MRSA: Easier to treat but are more virulent (deadly)

What protein synthesis inhibitors binds to 50S, blocking peptide bond formation?

-Macrolides -Telithromycin -Clindamycin -Chloramphenicol

Which carbapenems cover ESBL and KPC but not MBL?

-Meropenem + Vaborbactam -Imipenem + Cilastatin + Relebactam

What is the Penicillinase-Resistant Penicillins (PRPs)?

-Methicillin (discontinued) -Oxacillin -Cloxacillin -Dicloxacillin -Nafcillin

Adverse effects of Primaquine:

-Methoglobinemia if deficient in CYPB5 reductase -Hemolytic anemia in G6PDhase deficient

Which Beta-Blockers would be appropriate to use for arrhythmias? (Class II drugs)

-Metoprolol -Esmolol -Atenolol -Bisoprolol -Nebivolol (Notice: Beta-1-blockers)

What are the drugs that are considered Nitromidazoles?

-Metronidazole -Tinidazole

When is it appropriate to use Penicillin G Benzathine?

-Mild URTI (S. pyogenes) -Prophylaxis of Rheumatic fever -Syphilis (T. Pallidum) (Used for strains susceptible to Pen G.)

What are the advantages of Semisynthetic Glycopeptide over Vancomycin?

-More effectiveness -Longer Half-lives -Better tissue penetration

Why are natural penicillins referred to as benzylpenicillins?

-More potent -Erratic PO absorption (It is always given as IM or IV) -Unstable at low pH (it is broken down to penillic acid at low pH)

What are the differences of prasugrel from clopidogrel?

-More rapid -More potent -Decreased polymorphism (does not depend on CYP2C19) -More risk of bleeding

What is horizontal transfer of resistance?

-Most resistant genes are within plasmids -Genes are passed on from non-pathogenic to pathogenic bacteria

What is cefepime used for?

-Multi-drug resistant species -Reserved for serious infections (Pneumonia (HAP), Complicated UTI, Complicated intraabdominal infections, Febrile neutropenia) (Parenteral use only)

What are the two major sugars of the bacterial cell wall?

-NAG -NAM

How does the addition of a single amino group in ampicillin differentiate its coverage spectrum from that of Pen G?

-NH2 addition increases the polarity which increases Gram - coverage -Better porin penetration and increases stability against Beta-lactamases

Adverse effects of Linezolid and Tedizolid:

-Nausea, diarrhea, headache -Bone marrow suppression, thrombocytopenia (reversible) -Inhibits MOA (avoid giving with tyramine, decongestants) -Serotonin syndrome when used with SSRI's (causes fever, agitation, tremors)

Adverse effects of Itraconazole:

-Nausea, vomiting -Rash (Immune compromised patients) -Drug interactions due to P450 inhibition

Adverse effects of Nitromidazoles:

-Nausea/Vomiting -Metallic taste -Antabuse-like reaction (Metronidazole should not be used with alcohol) -Neurotoxicity (dizziness, vertigo, ataxia)

Adverse reactions of Nitrofurantoin:

-Nausea/Vomiting, headache, rash -Peripheral neuropathy -Acute pneumonitis (cough, fever, dyspnea; self-limiting)

What are the names of the Aminoglycosides used orally or topically?

-Neomycin -Kanamycin

Adverse Effects of PRPs:

-Nephritis especially with Methicillin -Nafcillin more likely to cause hypokalemia

What are the toxicity issues of Aminoglycosides?

-Nephrotoxicity -Ototoxicity -Neuromuscular blockade

What are the major toxicity issues with Colistin?

-Nephrotoxicity (proteinuria, reduced CrCl) -Neurotoxicity (Paresthesias, dizziness, headache, virtigo)

What group of antianginals must first be converted to nitric oxide?

-Nitroglycerin -Isosorbide Dinitrate -Isosorbide Mononitrate

What are features of Eukaryotes (animal cells)?

-No Cell Wall -Nucleus -Cytoplasmic organelles -Ribosomes: 40s + 60s = 80s (even) -Mitotic structures

What are the differences of Non-thienopyridines compared to thienopyridines?

-No activation -Faster Onset -Reversible binding

Understand Animal Cells:

-No cell wall -Nucleus -Cytoplasmic organelles -Ribosomes: 40S + 60S = 80S (even) -Reproduction = Mitosis

What are the 2 stages of bacterial growth in a culture?

-Normal bacterial growth -Bacterial growth with antibiotics

How does resistance develop against PRPs?

-OXA genes (various different groups) -Part of the ESBL subgroup -Hydrolyze most Beta-Lactams -Many not inhibited by clavulanic acid

Name the NS5A Inhibitors:

-Ombitasvir -Ledipasvir -Elbasvir -Velpatasvir -Daclatasvir (Hint: -asvir)

Cefepime has a broad spectrum coverage due to:

-Oxime group -N-methylpyrrolidine group

How are PRP drugs eliminated? How is Nafcillin eliminated

-PRP drugs: Renally -Nafcillin: Liver

What are the advantages of PRPs over Penicillin G?

-PRPs are useful for MSSA -Resistant to acid compared to Pen G

Symptoms of Arrhythmias:

-Palpitation (Heart "skipped a beat") -Fatigue -Syncope -Angina -Dizziness -Dyspnea (shortness of breath)

What are the Narrow Spectrum Penicillins? (Penicillinase-sensitive (Natural))

-Penicillin G (IV/IM) -Penicillin G Benzathine (IM) -Penicillin G Procaine (IM) -Penicillin V (oral)

What are the 4 Beta-Lactams?

-Penicillins -Cephalosporins -Carbapenem -Monobactam

What are the components of the bacterial cell wall?

-Peptidoglycan (Peptides) -NAG (N-acetylglucosamine) -NAM (N-acetylmuramic acid)

Adverse drug reactions of Pretomanid:

-Peripheral neuropathy -Acne -Musculoskeletal pain -Hepatotoxicity -Lactic acidosis -Anemia -QT prolongation

Adverse effects of Quinupristin + Dalfopristin:

-Phlebitis -Arthralgia-Myalgia -Hyperbilirubinemia

What are the adverse effects of Vancomycin?

-Phlebitis -Ototoxicity and Nephrotoxicity (worse when combined with other nephrotoxic drugs) -Red man syndrome -Myelosupression

Adverse Effects of Daptomycin:

-Phlebitis, fever, rash, diarrhea, headache -Myopathy at high dose, rare & reversible (Monitor pts already on statins) -Peripheral neuropathy -False increase in INR -Eosinophilic pneumonia (2-4 weeks after treatment)

Activation of ADP receptors mediates what?

-Platelet shape change -Expression of function GPIIb-IIIa

What are the 2 types of polymyxin?

-Polymyxin B -Polymyxin E (Colistin)

What are Gentamicin, Tobramycin, and Amikacin used for?

-Post-op infections -Burns -Intra-abdominal infections -Inhaled (CF); topical

Which electrolytes are all potential targets of antiarrhythmics?

-Potassium (polarizes) -Sodium (depolarizes) -Calcium (depolarizes)

What are the endogenous anticoagulant factors?

-Prostacyclin (PGI2) -Anti-thrombin III -Proteins C and S -Tissue factor pathway inhibitor (TFPI) -Tissue-type plasminogen activator (t-PA)

What are the benefits of normal flora?

-Protect host from pathogenic bacteria -Stimulate and train the immune system -Nutritional-digest fiber -Produces Vitamin K and B12 -Link between normal flora and obesity/diseases

What are contraindications of Dronedarone?

-QTc > 500 mS -Bradycardia (>50 bpm) -Used with CYP3A4 inhibitors

Name the antimalarial drugs:

-Quinine -Chloroquine -Primaquine -Mefloquine -Artemisinins

Name the integrase Inhibitors:

-Raltegravir -Elvitegravir -Dolutegravir

Name the Anti-HIV integrase inhibitors

-Raltegravir -Elvitegravir -Dolutegravir (Hint: -gravir)

ADME of sulfonamides:

-Rapidly and efficiently absorbed from GI tract -Good tissue distribution (Plasma protein bound) -Metabolism (acetylation) reduces activity but not toxicity -Renally excreted

What are fecal transplants used to treat?

-Recurring colitis (Clostridium difficile) -Obesity -Insulin resistance -Autoimmune disease

Describe bacterial chromosomal DNA.

-Relatively stable -Gives genetic identity

Non-adherence when treating Tuberculosis increases risks of:

-Resistant TB strains -Treatment failure -Relapse

What is the coverage of Ceftaroline?

-Resistant to Gram + infections such as MRSA -Some Gram - but not pseudomonas, not effective bs ESBL or KPC strains

Which drugs are considered Rifamycins?

-Rifampin -Rifamixin -Fidaxomicin

List the oral factor Xa inhibitors:

-Rivaroxaban -Apixaban -Edoxaban -Betrixaban (discontinued)

What are the 2 pacemaker cells in the conduction system?

-SA node (Sinoatrial) -AV node (Atrioventricular)

Name the Anti-HIV inhibitors of viral maturation

-Saquinavir -Ritonavir -Fosamprenavir -Indinavir -Nelfinavir -Lopinavir -Atazanavir -Tipranavir -Darunavir (Hint: -navir)

What are the Protease inhibitors that inhibit viral maturation?

-Saquinavir -Ritonavir -Fosamprenavir -Indinavir -Nelfinavir -Lopinavir -Atazanavir -Tipranavir -Darunavir (Endings with "-Navir")

What is the gram + coverage of Cefepime?

-Similar to 3rd gen but more effective against Beta-Lactam producers -MSSA -Streptococcus pyogenes -Viridians Streptococci -Streptococcus pneumonia

Describe the features of flagella.

-Single or multiple -Responsible for Motility -Can be a virulence factor -Swim "upstream" to avoid being flushed out

What types of infections can be caused by streptococcal pneumonia?

-Sinusitis -Otitis Media -Pneumonia -Meningitis -Bacteremia

Growth during infections is determined by:

-Site of infection -Inoculum size -Immune status of the host

What are the common locations of normal flora?

-Skin -Nose -Mouth -Vagina -Gut (critical to health)

Describe plasmid DNA.

-Small circular DNA -Mobile -Adaptation -Resistance genes

Name the drugs that inhibit Hep C replication.

-Sofosburvir -Ribavirin

What is the Gram + coverage of Fosfomycin?

-Staphylococci -MSSA -MRSA -Some Enterococci

What bacteria has a "chain" appearance?

-Streptobacillus -Streptococcus

What are the major Gram + Cocci organisms covered by Penicillin G?

-Streptococci species -Enterococcus Faecalis

What are the names of the Aminoglycosides used for IV/IM?

-Streptomycin -Gentamicin -Tobramycin -Amikacin

How is acquired resistance aquired?

-Sudden and dramatic increase in MIC or MBC -Bacterial Mutation -Selection pressure (antibiotic overuse)

What are Sulbactam and Tazobactam combined with?

-Sulbactam + Ampicillin -Tazobactam + Piperacillin (Zosyn)

What is Fluconazole used for?

-Systemic mycoses -Cryptococcal meningitis -Candidiasis (b/c wide tissue distribution), but with some resistance

What are black boxed warnings of Dronedarone?

-Systolic heart failure -Permanent atrial fibrillation

Name the NS3/4A Inhibitors:

-Telaprevir -Boceprevir -Simeprevir -Paritaprevir -Grazoprevir (Hint: -previr)

Name the Anti-Hep C inhibitors of viral maturation

-Telaprevir -Boceprevir -Simeprevir -Paritaprevir -Grazoprevir -Ombitasvir -Ledipasvir -Elbasvir -Velpatasvir -Dacalatasvir (Hint: -previr/asvir)

How are tetracyclines eliminated?

-Tetracyline: Mostly renal elimination -Doxycycline: Renal & Fecal -Minocycline: 30% in feces & urine

What does Anti-thrombin III inactivate?

-Thrombin (IIa) (2a) -IXa (9a) -Xa (10a) -XIa (11a) -XIIa (12a)

Which antiplatelet drugs are ADP (P2Y) receptor antagonists?

-Ticlopidine -Clopidogrel -Prasugrel -Ticagrelor -Cangrelor

What are common symptoms of cinchonism?

-Tinnitus (ringing of the ear) -Flush -Excessive sweating -Nausea/vomiting/diarrhea -Confusion

What are the Gram - spiral organisms covered by penicillin G?

-Treponema Pallidum -Borrelia Burgdorferi

What are the coverages of Amoxicillin +/- Clavulanic Acid?

-UTI: S. saprophyticus, E. Coli, Proteus, Klebsiella -URTI: S. Pneumonia, H. Influenzae, Moraxella -Bites/Wounds: Anaerobic/Gm - coverage (with CA) -Dental/Prophylaxis: Viridans S, Actinomyces, Enterococci -GI/Food poison: Campylobacter, E. Coli, Salmonella, Shigella

Give examples of Nosocomial infection.

-Urinary tract infections -Lower respiratory tract infections -Skin & soft tissue infections

What is the coverage of Oritavancin?

-VISA -VRSA -VRE -Enterococcus Faecium

What are the 2 drugs that are known as natural glycopeptides?

-Vancomycin -Teicoplanin

What are the major adverse effects of Telithromycin?

-Ventricular arrhythmias -Visual disturbances -Liver failure (Most serious)

List three parameters of oxygen demand:

-Ventricular wall stress (a function of both afterload (arteries) and preload (veins) -Heart rate -Contraction

What is the safety profile of Aztreonam?

-Very safe -Good alternative to aminoglycosides (similar coverage) -No cross-reaction in penicillin-allergic patients (if pt is allergic to penicillin it can still be used)

Adverse effects of Voriconazole:

-Visual disturbances (Blurred vision, hallucination, altered color vision) -Increased liver transaminases -Rash -Multiple drug interactions

ADME of Clindamycin:

-Well absorbed -Good distribution -Hepatic deactivation (Liver metabolism) through N-demethylation, sulfoxide formation

Bacterial pathogenesis should be viewed in context of _______, _________, and ________.

-What (Organism) -Where (Location within host) -Who (Health status of host)

How do changes in PBPs lead to resistance?

-When PBP loses its affinity for a Beta-Lactam, resistance occurs -In MRSA, the strains have PBPs have lost most of the affinity for Beta-Lactams

What is the advantage of using both Quinupristin + Dalfopristin together?

-When using one by itself or if something happens to one of the drugs they will produce a Bacteriostatic effect -When both drugs are working perfectly in combination the effect will be Bactericidal

Name the drugs that inhibit HIV replication.

-Zidovudine -Stavudine -Limavudine -Tenofovir -Abacavir -Didanosine -Emtricitabine

Name the NRTIs and their abbreviations:

-Zidovudine (AZT, ZDV) -Stavudine (d4T) -Lamivudine (3TC) -Emtricitabine (FTC) -Didanosine (DDI) -Abacavir (ABC) -Tenofovir Disoproxil (TDF)

How does a capsule contribute to virulence?

-helps bacteria evade the immune system -difficult for drugs to penetrate

What are the endogenous anticoagulant checkpoints to stop blood clots in intact blood vessels?

-t-PA -Prostacyclin (PGI2) -Proteins C and S -TFPI -Anti-thrombin III

What are some examples of exotoxins?

-tetanus -botulin -anthrax

What are bacteriocins?

-toxins produced by the "normal" flora -toxic to invading bacteria, but not toxic to the host

50S: Peptidyl transferase is a __(1)__ ribosomal RNA, and when the enzyme modifies it by __(2)___ it leads to resistance

1) 23S 2) Methylation

Aminoglycosides are imported into bacteria through an energy pump that depend on ___(1)___ respiration, so they have no impact on __(2)___ bacteria

1) Aerobic 2) Anaerobic

Enfuvirtide, Maraviroc, and Fostemsavir are known as entry inhibitors because they inhibit viral __(1)__ and __(2)__

1) Attachment 2) Entry

What are the basic steps of Gram staining?

1) Bacteria invisible 2) Add crystal violet: Both will be same color 3) Add iodine: Both will be same color 4) Decolorization with alcohol: G+ keeps stain but G- loses stain 5) Counter stain (safranin red): G- is light red/pink and G+ is violet

Red man syndrome is caused by the release of __(1)__ which is why vancomycin needs to be infused ___(2)___

1) Cytokines 2) Slowly

Plazomicin is a new drug that is effective against __(1)___ and __(2)__ strains for complicated UTI

1) ESBL (KPC) 2) CRE

LPS is an ___(1)___ and is __(2)___ which means it causes fever which makes it a virulence factor

1) Endotoxin 2) Pyrogenic

Bacillus anthracis also provides a potent __(1)___ that gives the ability to be highly pathogenic ___(2) ____

1) Exotoxin 2) Virulence factor

Voriconazole goes through __(1)__ metabolism by __(2)__

1) Extensive 2) CYP2C19

Posaconazole is __(1)__ metabolized and inhibits __(2)__

1) Extensively 2) CYP3A4

Griseofulvin absorption is increased with __(1)__ food and it undergoes __(2)__ metabolism

1) Fatty 2) Liver

Moxifloxacin is eliminated through __(1)___ route while all other fluoroquinolones are eliminated through __(2)___ route

1) Fecal 2) Renal

Mammals cannot synthesize ___(1)____ (diet is the source). Mammalian DHF-reductase has very low affinity for ___(2)____

1) Folic acid 2) Trimethoprim

Place the following events of platelet activation by TxA2 in primary hemostasis in order: (1st, 2nd, 3rd, 4th, 5th) A) Activation of GPIIb-IIIa B) Generation of thromboxane A2 by activated platelets C) Platelet aggregation D) Binding of fibrinogen to GPIIb-IIIa E) Activation of thromboxane A2

1) Generation of thromboxane A2 by activated platelets 2) Activation of thromboxane A2 receptor 3) Activation of GPIIb-IIIa 4) Binding of fibrinogen to GPIIb-IIIa 5) Platelet aggregation

Chloramphenicol can cause ___ (1) __ syndrome which relates to ____(2)____ pathways being limited in neonates

1) Gray Baby 2) Glucuronidation

Ritonavir is not a drug that is used for __(1)__. It is a potent ___(2)___ inhibitor. It is used as a ___(3)___ for NS3/4A, NS5A, and NS5B inhibitors

1) Hep C 2) CYP3A4 3) Booster

Warfarin is by monitored ___(1)___; while Heparin is monitored by ___(2)___

1) INR 2) aPTT

Bacteriostatic drugs __(1)___ bacterial growth; Bacteria resumes __(2)__ upon drug removal

1) Inhibits 2) Growth

Transposons (Tn) are also known as ___ (1) ___ because they excise from one location & insert into another target __(2)_

1) Jumping genes 2) DNA

Echinocandins are IV given drugs that undergo __(1)__ metabolism and can cause __(2)__

1) Liver 2) Hepatotoxicity

Aminoglycosides are rapidly inactivated by __(1)__ pH, divalent __(2)__, and hyperosmolar conditions (sepsis), therefore decreases effectiveness of aminoglycosides

1) Low 2) Cations

Which drugs only work for HIV-1 family?

1) NNRTIs: -Efavirenz, Nevirapine, Delaviridine, Etravirine, Rilpivirine, Doravirine 2) Maraviroc 3) Fotemsavir

Since gram __(1)__ bacteria contain an outside lipid layer, they utilize porins to import __(2)___ nutrients into their ___(3)___ space. These porins do not generally allow entry of molecules bigger than __(4)__ dalton in size.

1) Negative 2) Polar 3) Periplasmic 4) 700

The difference between Clarithromycin and Erythromycin are: Clarithromycin has an -__(1)__ group while erythromycin has an -OH group. This makes clarithromycin __(2)__ stable, has __(3)__ bioavailability and more effective with the same coverage. Clarithromycin does not form a cyclic ketal meaning it does not produce __(4)__

1) OCH3 group 2) More 3) Better 4) Cramps

Gatifloxacin is available ONLY as an __(1)___ form and NOT oral due to __(2)___

1) Ophthalmic 2) Liver toxicity

Bacterial pathogenesis should be viewed in context of __(1)__, __(2)__, and __(3)__

1) Organism (what) 2) Location in host (where) 3) Health status of host (who)

What 3 reactions occur during primary hemostasis?

1) Platelet Adhesion 2) Platelet granule release reaction 3) Platelet Aggregation and consolidation

Aminoglycosides are very ___(1)___, poorly absorbed orally, and are used ___(2)___

1) Polar 2) Parenterally

If a structure contains an OH indicating a ribose it is an __(1)__ whereas if it has an H indicating a deoxyribose, it is an __(2)__

1) RNA 2) DNA

Most __(1)__ are single stranded and most __(2)__ are double stranded

1) RNA 2) DNA

Imipenem is easily broken down by ____(1)____, therefore it requires coadministration of ___(2)____, which inhibits the dipeptidase

1) Renal dehydropeptidase 2) Cilastatin

Ribavirin inhibits viral genome ___(1)___. It is an incomplete analogue of ___(2)___. It is used in ___(3)___

1) Replication 2) Guanine 3) Combination

Amphotericin B has a __(1)__ IV infusion with a serum half life of __(2)__ days

1) Slow 2) 15 days

Enfuvirtide is administered only __(1)__ while Maraviroc and Fostemsavir are administered __(2)__

1) Subcutaneously 2) Orally

Rifamixin is used to treat ___(1)____ & ___(2)__ due to poor systemic absorption

1) Traveler's diarrhea 2) IBD

Voriconazole is an analog of Fluconazole. Their structures are fairly similar but Voriconazole has a __(1)__ and Fluconazole has a ___(2)___

1) Triazole 2) Fluorinated azole

Which 3 enzymes activate and aggregate platelets?

1) TxA2 2) ADP 3) Thrombin

RNA has __(1)__ whereas DNA has __(2)__

1) Uracil 2) Thymine

Amphotericin B has __(1)__ tissue distribution but poor __(2)__ levels

1) Wide 2) CSF

Flucytosine is an oral drug that has __(1)__ tissue distribution and it penetrates the __(2)__

1) Wide 2) CSF

Liquid Itraconazole should be taken __(1)__ food because it __(2)__ bioavailability

1) Without 2) Increases

Which enzyme do Azoles target?

14-a-demethylase

Match the following drugs to their actions: (Number to letter) 1) Warfarin 2) Heparin 3) Fondaparinux 4) Direct thrombin inhibitors A) These analogs inactivate factors B) Inactivates Xa C) Interferes with vitamin K, thus activation of factors D) Directly inactivates thrombin

1:C 2:A 3:B 4:D

Going from one generation to another, how do cephalosporins compare in general coverage of Gram + bacteria?

1st > 2nd = 3rd < 4th

Going from one generation to another, how do cephalosporins compare to sensitivity to beta-lactamases?

1st > 2nd > 3rd > 4th

Which generations of cephalosporins are Beta-Lactamase inducers?

1st, 2nd, 3rd generations

Warfarin blocks which factors?

2,5,7,9,10 Targets inactive factors

For most indications, which INR level is recommended?

2-3

Bacterial ribosome subunits are ____ + ____ = ____

30S + 50S = 70S

What is the abbreviation for Lamivudine?

3TC

Which generation of Fluoroquinolones is best to use for Gram + bacteria such as MSSA, Strep., E. Faecalis, and Listeria?

3rd generation (Levofloxacin, Moxifloxacin, Gemifloxacin)

Going from one generation to another, how do cephalosporins compare to CSF penetration?

4th = 3rd > 2nd > 1st

Going from one generation to another, how do cephalosporins compare in general coverage of Gram - bacteria?

4th > 3rd > 2nd > 1st

Which generation of cephalosporins are not beta-lactamase inducers?

4th generation

What is the elimination of Colistin?

50% Colistimethate is renally unchanged

Gut flora is a complex ecosystem of more than ___ species.

500

What is the half life length of Amiodarone?

54 days

What is the pH of gastric secretions? A) 1.5-2.0 (acidic) B) 7.25-8.0 (basic)

A) 1.5-2.0 (acidic)

Which generation of cephalosporins are least useful for treatment of meningitis? A) 1st B) 3rd C) 2nd D) 4th

A) 1st

Which generation of Cephalosporins has the best gram negative coverage? A) 3rd B) 1st C) 2nd

A) 3rd

Which of the following is an active metabolite of sulfasalazine having anti-inflammatory actions? A) 5-Aminosalicylic Acid B) Sulfapyridine

A) 5-Aminosalicylic Acid

Isosorbide mononitrate causes which of the following: A) A decrease in preload B) A decrease in afterload C) No change in afterload or preload

A) A decrease in preload

Isoniazid (INH) is which of the following? (select all that apply) A) AUC/MIC dependent B) Concentration dependent C) Bactericidal D) Bacteriostatic

A) AUC/MIC dependent C) Bactericidal

Pyrazinamide (PZA) is which of the following? (select all that apply) A) AUC/MIC dependent B) Concentration dependent C) Bactericidal D) Bacteriostatic

A) AUC/MIC dependent C) Bactericidal D) Bacteriostatic

Ethambutol (EMB) is which of the following? (select all that apply) A) AUC/MIC dependent B) Concentration dependent C) Bactericidal D) Bacteriostatic

A) AUC/MIC dependent B) Concentration dependent C) Bactericidal

Which of the following glycoprotein IIb-IIIa receptor antagonists bind with an irreversible manner? A) Abciximab B) Tirofiban C) Eptifibatide D) Defibrotide

A) Abciximab

Which of the following drugs are glycoprotein IIb/IIIa receptor antagonists? (select all that apply) A) Abciximab B) Eptifibatide C) Tirofiban D) Vorapaxar E) Defibrotide

A) Abciximab B) Eptifibatide C) Tirofiban E) Defibrotide

Excretion of amantadine is increased in ____ urine A) Acidic B) Basic

A) Acidic

Which of the following events occur following the activation of H2 receptors on the parietal cells? A) Activation of Gs protein B) Activation of Gi Protein C) Inactivation of Gs Protein D) Inactivation of Gi Protein

A) Activation of Gs protein

Which of the following requires multiple dosing frequency? A) Acyclovir B) Valacyclovir

A) Acyclovir

Which of the following Anti-Hep B medications are Nucleotide analogues? (select all that apply) A) Adefovir B) Tenofovir C) Telbivudine D) Lamivudine

A) Adefovir B) Tenofovir

Resistance and nephrotoxicity is a major concern of: A) Adefovir Disoproxil B) Tenofovir Disoproxil

A) Adefovir Disoproxil

Which of the following nucleobases are purines? (select all that apply) A) Adenine B) Guanine C) Cytosine D) Uracil E) Thymine

A) Adenine B) Guanine

For a drug that has concentration-dependent killing, what is the best way to enhance its therapeutic effect? A) Aim for a peak serum concentration that is 8-10 times the MIC B) Increasing dosing frequency

A) Aim for a peak serum concentration that is 8-10 times the MIC

Which of the following is excreted in an unchanged form in the urine? A) Amantadine B) Rimantadine

A) Amantadine

Which of the following anti-influenza drugs inhibit viral uncoating? (select all that apply) A) Amantadine B) Zanamavir C) Rimantadine D) Oseltamivir E) Peramivir

A) Amantadine C) Rimantadine

Which of the following cover pseudomonas? (select all that apply) A) Amikacin B) Meropenem C) Ertapenem D) Azithromycin E) Clindamycin

A) Amikacin B) Meropenem

Which of the following bind to 30s ribosomal subunit? (select all that apply) A) Aminoglycosides B) Tetracyclines C) Macrolides D) Clindamycin

A) Aminoglycosides B) Tetracyclines

What drugs bind to 30S in Bacterial protein synthesis? (select all that apply) A) Aminoglycosides B) Macrolides C) Tetracyclines D) Clindamycin E) Chloramphenicol F) Streptogramins G) Oxazolidinones (Linezolid) H) Telithromycin

A) Aminoglycosides C) Tetracyclines

Which of the following class III antiarrhythmics is safe to use for a patient with a sulfa allergy? A) Amiodarone B) Sotalol C) Dorendarone D) Ibutilide

A) Amiodarone

Which of the following are Class III antiarrhythmic agents? (select all that apply) A) Amiodarone B) Dronedarone C) Lidocaine D) Sotalol E) Ibutilide F) Dofetilide

A) Amiodarone B) Dronedarone D) Sotalol E) Ibutilide F) Dofetilide

Which of the following covers Listeria? A) Ampicillin B) Nafcillin C) Cefoxitin D) Cefepime

A) Ampicillin

Which of the following cross the blood brain barrier sufficiently enough to treat meningitis? (select all that apply) A) Ampicillin B) Cefazolin C) Cefotaxime D) Ceftriaxone

A) Ampicillin C) Cefotaxime D) Ceftriaxone

This is a chemical substance that destroys microorganism with minimal damage to the host A) Antimicrobial Drug B) Antibiotic

A) Antimicrobial Drug

Amino acid residue _____ present on the H2 receptor membrane is essential for ligand docking and positioning A) Asp 98 B) Asp 186 C) Asp 196 D) Tyr 182

A) Asp 98

Which of the following drugs is able to block cyclooxygenase, production of TxA2, and aggregation to prevent blood clotting? A) Aspirin B) Tylenol C) Meloxicam D) Hydrocodone

A) Aspirin

Which of the following inhibits Thromboxane A2? A) Aspirin B) Morphine C) Tenecteplase D) Ticagrelor

A) Aspirin

Which of the following beta-lactamase inhibitors is NOT a beta-lactam itself? A) Avibactam B) Tazobactam C) Sulbactam D) Clavulanic acid

A) Avibactam

Which of the following are NOT Beta-Lactams? (select all that apply) A) Avibactam B) Sulbactam C) Vaborbactam D) Relebactam

A) Avibactam C) Vaborbactam D) Relebactam

Mycoplasma pneumonia is susceptible to which of the following? (select all that apply) A) Azithromycin B) Doxycycline C) Gentamicin D) Clindamycin

A) Azithromycin B) Doxycycline

Which of the following provide adequate coverage for mycoplasma pneumoniae? (select all that apply) A) Azithromycin B) Doxycycline C) Gentamicin D) Ceftriaxone

A) Azithromycin B) Doxycycline

Fluoroquinolones produce a ______ effect A) Bactericidal B) Bacteriostatic

A) Bactericidal

Which receptors are responsible for the action of Metoprolol in angina? A) Beta-1 B) Beta-2 C) Beta-3

A) Beta-1

Which of the following is derived from medical leech and directly inhibits thrombin? A) Bivalirudin B) Enoxaparin C) Eptifibatide D) LMWH

A) Bivalirudin

This type of spectrum targets both gram + and gram - bacteria A) Broad Spectrum B) Narrow spectrum C) Extended Spectrum

A) Broad Spectrum

The advantage of Plazomicin over Gentamicin is: A) CRE/KPC coverage B) MRSA coverage C) Pseudomonas coverage D) Anaerobic coverage

A) CRE/KPC coverage

The advantage of Plazomicin over gentamicin is: A) CRE/KPC coverage B) MRSA coverage C) Pseudomonas coverage D) Anaerobic coverage

A) CRE/KPC coverage

Which CYP enzymes does Voriconazole inhibit? (select all that apply) A) CYP2C19 B) CYP2C9 C) CYP450 D) CYP3A4

A) CYP2C19 B) CYP2C9 D) CYP3A4

Which of the following statements are true regarding Fosfomycin? (Select all that apply) A) Can be used safely during Pregnancy B) Can be used for Treatment of pyelonephritis if the species is susceptible to the drug C) Inhibits cell wall synthesis D) Covers gram negatives but not gram positives involved in UTI's E) Covers major gram negative and gram positive species involved in UTI's

A) Can be used safely during Pregnancy C) Inhibits cell wall synthesis E) Covers major gram negative and gram positive species involved in UTI's

Which of the following are Echinocandins? (select all that apply) A) Caspofungin B) Micafungin C) Anidulafungin D) Fluconazole

A) Caspofungin B) Micafungin C) Anidulafungin

Which of the following describes the function of Peptidoglycan synthase? A) Catalyzes NAG-NAM complex by taking it outside and flipping it over to the cell wall outside of the cell wall membrane B) Removes phosphate and recycles bactoprenol C) Takes NAG-NAM complex outside of the inner layer

A) Catalyzes NAG-NAM complex by taking it outside and flipping it over to the cell wall outside of the cell wall membrane

Which of the following provide the best coverage of CAPES? A) Cefepime B) Ceftriaxone C) Cefuroxime D) Cefazolin

A) Cefepime

Which of the following covers MRSA? A) Ceftaroline B) Ceftolozane C) Ceftriaxone D) Ceftazidime

A) Ceftaroline

Which of the following is/are effective against VRSA? (select all that apply) A) Ceftaroline B) Daptomycin C) Dalbavancin D) Oritavancin E) Linezolid

A) Ceftaroline C) Dalbavancin D) Oritavancin E) Linezolid

Which of the following cover pseudomonas? A) Ceftazidime B) Ceftolozane C) Piperacillin + Tazobactam D) Meropenem E) Aztreonam

A) Ceftazidime C) Piperacillin + Tazobactam D) Meropenem E) Aztreonam

Which of the following cover KPC positive strains? (select all that apply) A) Ceftazidime + Avibactam B) Meropenem + Vaborbactam C) Aztreonam D) Colistin

A) Ceftazidime + Avibactam B) Meropenem + Vaborbactam D) Colistin

Which of the following cover KPC positive strains? (select all that apply) A) Ceftazidime + Avibactam B) Meropenem + Vaborbactam C) Aztreonam D) Amikacin E) Tigecycline

A) Ceftazidime + Avibactam B) Meropenem + Vaborbactam E) Tigecycline

Which 3rd generation Cephalosporin is eliminated through the liver and can cause biliary sludging in neonates? A) Ceftriaxone B) Cefotaxime C) Ceftazidime D) Cefixime

A) Ceftriaxone

Which 3rd generation drugs cover meningitis & borrelia? (select all that apply) A) Ceftriaxone B) Cefotaxime C) Ceftazidime D) Cefixime E) Cefdinir F) Cefditoren G) Ceftibuten H) Cefpodoxime

A) Ceftriaxone B) Cefotaxime

Which of the following cover N. gonorrhoea? (select all that apply) A) Ceftriaxone B) Cefotaxime C) Cefpodoxime D) Cefixime

A) Ceftriaxone B) Cefotaxime C) Cefpodoxime D) Cefixime

Which of the following penetrate the BBB and can be used in treatment of bacterial meningitis? (select all that apply) A) Ceftriaxone B) Cefepime C) Cefotaxime D) Cefazolin E) Ampicillin

A) Ceftriaxone C) Cefotaxime E) Ampicillin

Which 3rd generation drugs cover N. gonorrhea? (select all that apply) A) Ceftriaxone B) Cefotaxime C) Ceftazidime D) Cefixime E) Cefdinir F) Cefditoren G) Ceftibuten H) Cefpodoxime

A) Ceftriaxone D) Cefixime H) Cefpodoxime

Which 3rd generation drugs are best for Gram + coverage? (select all that apply) A) Ceftriaxone B) Cefotaxime C) Ceftazidime D) Cefixime E) Cefdinir F) Cefditoren G) Ceftibuten H) Cefpodoxime

A) Ceftriaxone F) Cefditoren H) Cefpodoxime

Which of the following 2nd generation cephalosporins is administered parenterally? A) Cefuroxime B) Cefuroxime axetil C) Cefaclor D) Cefprozil

A) Cefuroxime

Which of the following would be the best option for treating Lyme disease? A) Cefuroxime B) Cefazolin C) Cephalexin D) Cefotetan

A) Cefuroxime

All drugs that inhibit _____ are bactericidal A) Cell wall synthesis B) Folic acid synthesis C) Protein synthesis

A) Cell wall synthesis

Which of the following are the mechanisms involved in resistance to beta-lactams? (select all that apply) A) Changes in the affinity of PBPs for beta-lactams B) Changes in porin structures C) Beta-Lactamase production

A) Changes in the affinity of PBPs for beta-lactams B) Changes in porin structures C) Beta-Lactamase production

Which of the following drugs inhibit formation of hemozin? A) Chloroquine B) Artemisinins C) Primaquine D) Quinine

A) Chloroquine

Name the reservoir form of Cidofovir responsible for its longer intracellular half-life A) Cidofovir-Phosphocholine B) Cidofovir-Diphosphate

A) Cidofovir-Phosphocholine

Which H2 antagonists specifically undergoes biotransformation via methyl hydroxylation? A) Cimetidine B) Ranitidine C) Nizatidine D) Famotidine

A) Cimetidine

Which of the following H2 antagonist undergoes metabolism via methyl hydroxylation? A) Cimetidine B) Nizatidine

A) Cimetidine

Which of the following drugs may produce maximum CYP450 enzyme inhibition? A) Cimetidine B) Ranitidine C) Nizatidine D) Famotidine

A) Cimetidine

Which of the following inhibits DNA replication & transcription? A) Ciprofloxacin B) Lefamulin C) Linezolid D) Doxycycline

A) Ciprofloxacin

Oral absorption of which of the following will be decreased by dairy products? (select all that apply) A) Ciprofloxacin B) Doxycycline C) Linezolid D) Clindamycin

A) Ciprofloxacin B) Doxycycline

Tums will decrease oral absorption of: (select all that apply) A) Ciprofloxacin B) Doxycycline C) Linezolid D) Clarithromycin

A) Ciprofloxacin B) Doxycycline

Due to decrease of automaticity of SA nodal cells, a shift threshold to more positive potential and decreased slope of phase 4 occurs with which class of drugs? A) Class I B) Class II C) Class III D) Class IV

A) Class I

Which class of antiarrhythmics have effects on both myocytes and pacemaker cells? A) Class I B) Class II C) Class III D) Class IV

A) Class I

Which class of drugs in the pacemaker cells decrease automaticity of SA nodal cells? A) Class I B) Class II C) Class III D) Class IV

A) Class I

Which of the following class of drugs are Na+ channel blockers? (Hint: it has 3 subclasses) A) Class I B) Class II C) Class III D) Class IV

A) Class I

Which subclass of class I drugs have a moderate Na+ channel block and prolong (block) repolarization of K+ channels? A) Class IA B) Class IB C) Class IC

A) Class IA

Which of the following are Beta-Lactamase inhibitors? (select all that apply) A) Clavulanic Acid B) Sulbactam C) Avibactam D) Tazobactam

A) Clavulanic Acid B) Sulbactam D) Tazobactam

Select all the species covered by Vancomycin: (Select all that apply) A) Clostridium difficile B) Pseudomonas C) MRSA D) Haemophilus Influenzae E) Enterococcus faecalis D) VRSA

A) Clostridium difficile C) MRSA E) Enterococcus faecalis

Which drug is similar in that it is used as a booster but only in Stribild? A) Cobicistat B) Emtricitabine C) Tenofovir

A) Cobicistat

Lefamulin is approved for the treatment of: A) Community Acquired Bacterial Pneumonia B) UTI and pyelonephritis C) Skin and soft tissue infections

A) Community Acquired Bacterial Pneumonia

Lamivudine (3TC) + Zidovudine (AZT) = A) Comvibir B) Epzicom C) Trizivir

A) Comvibir

Cycloserine inhibits: A) Conversion of L-Ala to D-Ala B) Conversion of D-Ala to L-Ala C) Conversion of NAG to NAM D) Conversion of NAM to NAG

A) Conversion of L-Ala to D-Ala

Which of the following are true about the roles of thrombin (IIa)? (select all that apply) A) Converts fibrinogen into fibrin (Ia) B) Activates factor XIII which cross links the fibrin polymers into a stable clot C) Activates Factor XI which cross links the fibrin polymers into a stable clot D) Catalyzes the feedback activation of factors VIII and V E) Strongly activates platelets, causing granule release, platelet aggregation

A) Converts fibrinogen into fibrin (Ia) B) Activates factor XIII which cross links the fibrin polymers into a stable clot D) Catalyzes the feedback activation of factors VIII and V E) Strongly activates platelets, causing granule release, platelet aggregation

Which of the following is a type of suborder of mycobacterium that has high guanine cytosine content? A) Corynebacterineae B) Acid-Fast C) Mycobacteria D) Mycolic Acid

A) Corynebacterineae

Which of the following inhibits racemase which interferes with production of d-alanine for cell wall incorporation? A) Cycloserine B) Capreomycin C) Bedaquiline D) Pretomanid

A) Cycloserine

Usually, the terminal amino acids within the pentapeptide attached to NAM are: A) D-Ala-D-Ala B) L-Lys-L-Ala C) L-Ala-L-Ala D) D-Lys-D-Ala

A) D-Ala-D-Ala

Rifampin binds the B-subunit of _____ at the allosteric site of the bacteria A) DDRP B) Nicotinamidase C) Multi-drug resistant TB (MDR-TB) D) Extensively-drug resistant TB (XDR-TB)

A) DDRP

Cytomegalovirus (CMV) is considered what kind of virus based? A) DNA B) RNA

A) DNA

Herpes (HSV-1, HSV-2, Varicella-zoster virus (VZV)) is considered what kind of virus based? A) DNA B) RNA

A) DNA

Which of the following direct thrombin inhibitors is the only oral drug from the following: (rest being parenteral) A) Dabigatran B) Lepirudin C) Desirudin D) Bivalirudin E) Argatroban

A) Dabigatran

What are the effects of PGE2 receptor activation on the adenylyl cyclase down stream signaling? (select all that apply) A) Decreased AC down signaling B) Decreased cAMP levels C) Decreased protein kinase levels D) Decreased proton pump activity

A) Decreased AC down signaling B) Decreased cAMP levels C) Decreased protein kinase levels D) Decreased proton pump activity

Which of the following needs to be combines with antacids to increase oral bioavailability? A) Didanosine (DDI) B) Abacavir (ABC) C) Tenofovir disoproxil (TDF)

A) Didanosine (DDI)

Trimethoprim inhibit which of the following? A) Dihydrofolate reductase B) Dihyrdopteroate synthase C) Topoisomerases

A) Dihydrofolate reductase

Which of the following drugs are PDE3 Inhibitors? (select all that apply) A) Dipyridamole B) Clopidogrel C) Aspirin D) Cilostazol

A) Dipyridamole D) Cilostazol

Which of the following are class IA drugs? (select all that apply) A) Disopyramide B) Flecainide C) Quinidine D) Procainamide

A) Disopyramide C) Quinidine D) Procainamide

Which of the following is a Anti-HSV drug that inhibits viral attachment and entry? A) Docosanol B) Enfuvirtide C) Maraviroc

A) Docosanol

Which of the following are covered by Piperacillin + Tazobactam? (select all that apply) A) E. Coli (susceptible) B) Bacteroides C) Pseudomonas D) E. Coli (ESBL) E) Acinetobacter F) MRSA G) E. Coli (KPC)

A) E. Coli (susceptible) B) Bacteroides C) Pseudomonas E) Acinetobacter

An enteric gram negative rod that commonly causes UTIs A) E. coli B) Neisseria gonorrhoeae C) Staphyloccus Saprophyticus D) Moraxella Catarrhalis

A) E. coli

Which of the following drug(s) inhibit factor Xa? (select all that apply) A) Edoxaban B) Warfarin C) Fondaparinux

A) Edoxaban C) Fondaparinux

Which of the following NNRTIs are considered to be 1st generation? (select all that apply) A) Efavirenz B) Nevirapine C) Delaviridine D) Doravirine

A) Efavirenz B) Nevirapine C) Delaviridine

This is a structural part of bacteria that causes the immune system to react such as causing fevers. It comes from Lipopolysaccharide (LPS) and contain Gram - bacteria. A) Endotoxin B) Exotoxin

A) Endotoxin

Which of the following is the only antiretroviral drug for HIV that must be given SubQ? A) Enfuvirtide B) Maraviroc C) Fostemsavir

A) Enfuvirtide

Which of the following are nucleoside inhibitors because they have no phosphate groups? (select all that apply) A) Entecavir B) Adefovir C) Telbivudine

A) Entecavir C) Telbivudine

Which of the following is susceptible to vancomycin? A) Enterococcus faecalis B) E. Coli C) Salmonella D) Haemophilus influenzae

A) Enterococcus faecalis

Which of the following binds to GPIIb-IIIa receptor reversibly and is used with aspirin and heparin? A) Eptifibatide B) Abciximab

A) Eptifibatide

Exposed tissue factor due to vascular injury or trauma activates which pathway? A) Extrinsic Pathway B) Intrinsic Pathway C) Common Pathway

A) Extrinsic Pathway

These enzymes synthesizes mycolic acid from 2-carbon units carried by acetyl CoA that forms a large waxy barrier and is essential for outer membrane formation in the process of mycobacterial cell wall synthesis. A) Fatty acid synthetases (FAS) 1 and 2 B) Nicotinamidase C) Arabinogalactan D) Pyridoxine

A) Fatty acid synthetases (FAS) 1 and 2

Mycolic acid has large, branched alkyl chains that is synthesized by: A) Fatty acid synthetases 1 and 2 B) Arabinogalactan C) Mycolic acid

A) Fatty acid synthetases 1 and 2

Which of the following are important anti-TB drug targets? (select all that apply) A) Fatty acid synthetases 1 and 2 B) Arabinogalactan C) Mycolic acid

A) Fatty acid synthetases 1 and 2 B) Arabinogalactan C) Mycolic acid

This bacterial structure helps bacteria move and swims upstream to avoid being flushed out. It is a virulence factor and is an example of vibrio cholera A) Flagella B) Fimbriae

A) Flagella

This class IC agent is a fluorinated benzamide derivative and is typically reserved for unresponsive arrhythmias A) Flecainide B) Propafenone C) Quinidine D) Lidocaine

A) Flecainide

Which of the following drugs are subclass IC agents? (select all that apply) A) Flecainide B) Propafenone C) Quinidine D) Lidocaine

A) Flecainide B) Propafenone

Which of the following drugs specifically targets Factor Xa? A) Fondaparinux B) Dalteprin C) Enoxaparin

A) Fondaparinux

Which of the following is the best recommendation to keep in mind when taking oral minocycline? A) Food is okay but avoid taking it with dairy B) Take it with a full glass of milk C) Take it on an empty stomach

A) Food is okay but avoid taking it with dairy

Which of the following protease inhibitors is a phosphate drug? A) Fosamprenavir B) Delaviridine C) Amprenavir D) Tipranavir

A) Fosamprenavir

____ inhibits conversion of NAG to NAM A) Fosfomycin B) Cycloserine C) Bacotracin

A) Fosfomycin

Which of the following is true about time-depending killing? (select all that apply) A) Further increase in concentration above MIC has minimal benefit B) Maximize duration of exposure C) Has no significant Post-Antibiotic Effect (PAE) D) Has significant Post-Antibiotic Effect (PAE) E) Examples of time-dependent killing drugs are beta-lactams and macrolides

A) Further increase in concentration above MIC has minimal benefit B) Maximize duration of exposure C) Has no significant Post-Antibiotic Effect (PAE) E) Examples of time-dependent killing drugs are beta-lactams and macrolides

Which of the following requires a more frequent dose? A) Ganciclovir B) Valganciclovir

A) Ganciclovir

Which of the following viral drugs can be used for CMV? (select all that apply) A) Ganciclovir B) Valganciclovir C) Cidofovir D) Foscarnet

A) Ganciclovir B) Valganciclovir C) Cidofovir D) Foscarnet

Which of the following polymerase inhibitors have chain elongation? (select all that apply) A) Ganciclovir B) Valganciclovir C) Penciclovir D) Famciclovir E) Acyclovir

A) Ganciclovir B) Valganciclovir C) Penciclovir D) Famciclovir

Which of the following drugs demonstrate a significant PAE? A) Gentamicin B) Penicillin C) Erythromycin

A) Gentamicin

Porin is required for important of polar molecules into: A) Gram negative bacteria B) Gram positive bacteria C) Both gram positive and gram negative

A) Gram negative bacteria

Vancomycin has a MW of 1400. It will not penetrate: A) Gram negative bacteria B) Gram positive bacteria

A) Gram negative bacteria

Which of the following viruses have RNA genome? (select all that apply) A) HIV B) HBV C) HCV D) Influenza

A) HIV C) HCV D) Influenza

Which of the following are DNA based viruses? A) HSV, Varicella, Cytomegalovirus, Hep B B) HIV, Hepatitis A & C, Influenza

A) HSV, Varicella, Cytomegalovirus, Hep B

Gram negative bacteria differ from Gram positive in that they: (select all that apply) A) Have porins B) Have periplasmic space C) Contain LPS D) Lack LPS E) Have thicker cell walls than gram positives F) Haver thinner cell walls than gram positives

A) Have porins B) Have periplasmic space C) Contain LPS F) Haver thinner cell walls than gram positives

The major (and a serious) concern with Telithromycin is: A) Hepatotoxicity B) Nephrotoxicity C) Myelosuppression

A) Hepatotoxicity

Which of the following neurotransmitter is released following the activation of M1 and gastrin receptors on the paracrine cells? A) Histamine B) Acetylcholine C) Dopamine D) Serotonin

A) Histamine

The sex pili are involved in ____ transfer of resistance genes A) Horizontal B) Vertical

A) Horizontal

Foscarnet can cause: A) Hypocalcemia B) Hypercalcemia

A) Hypocalcemia

Which class of antiarrhythmic drugs are Na+ channel blockers that shortens phase 3 repolarization in ventricular muscle fibers? A) IB B) II C) III D) IA E) IC F) IV

A) IB

Which of the following is an antidote (reversal agent) for Dabigatran etexilate? A) Idarucizumab B) Andexanet alfa C) Vitamin K D) Protamine E) Antifibrinolytic drugs

A) Idarucizumab

What are the effects of the activation of gastrin and M1 receptors in the parietal cells? (select all that apply) A) Increased intracellular calcium levels B) Increased levels of protein kinases C) Increased activity of proton pump

A) Increased intracellular calcium levels B) Increased levels of protein kinases C) Increased activity of proton pump

All third and fourth generation cephalosporins contain an aminothiazole moiety. What are the advantages of having this group? (select all that apply) A) Increased resistance to beta-latamases B) Increased affinity for PBPs C) Increased porin penetration D) Increased Oral bioavailability

A) Increased resistance to beta-latamases B) Increased affinity for PBPs C) Increased porin penetration

What are the beneficial effects of decreasing the acid secretion in the gastric mucosa? A) Increased wound healing B) Decrease in number of lesions C) Increased inflammation

A) Increased wound healing B) Decrease in number of lesions

In signal transmission, atria and ventricles contract ______ A) Independently B) Together

A) Independently

Which of the following is a TNF-alpha monoclinical antibody? A) Infliximab B) Milk of magnesia C) Prednisolone D) Loperamide

A) Infliximab

Which of the following distinguishes cefiderocol from other cephalosporins? A) It binds iron and is imported into bacteria B) It binds to and inhibits porins C) It is effective against listeria and Enterococci

A) It binds iron and is imported into bacteria

Amoxicillin differs from ampicillin in that it contains an additional hydroxy group. What does the hydroxy group do? A) It improves solubility and oral absorption B) Protects the beta-lactam ring from stomach acid C) Increases the affinity of the drug for gram negative PBPs D) Protects the beta-lactam ring from Beta-Lactamases

A) It improves solubility and oral absorption

Why is Pen G not used orally? A) It is broken down to penillic acid at low pH B) It irritates the gastric lining C) It is broken down by esterases in the stomach

A) It is broken down to penillic acid at low pH

Which of the following is true regarding sex pilus? A) It is involved in horizontal transfer of genes B) It is a modified form of flagella C) It is required for vertical transfer of genes

A) It is involved in horizontal transfer of genes

Which NRTIs are least toxic? (select all that apply) A) Lamivudine B) Emtricitabine C) Zidovudine D) Stavudine

A) Lamivudine B) Emtricitabine

Which of the following is an atypical? A) Legionella B) Campylobacter C) Listeria D) Treponema Pallidum

A) Legionella

Which of the following inhibits DNA replication & transcription? A) Levofloxacin B) Lefamulin C) Linezolid D) Daptomycin

A) Levofloxacin

Which class IB antiarrhythmic drug is effective for ischemic myocardium and is safer for prolonged QT situations? A) Lidocaine B) Phenytoin C) Amiodarone D) Mexiletine

A) Lidocaine

Which of the following are class IB antiarrhythmic drugs? (select all that apply) A) Lidocaine B) Phenytoin C) Amiodarone D) Mexiletine

A) Lidocaine B) Phenytoin D) Mexiletine

Which of the following virulence factors is an endotoxin? A) Lipopolysaccharide B) Tetanus Toxin C) Anthrax toxin D) Botulin Toxin

A) Lipopolysaccharide

Generally speaking, cephalosporins are not useful against: A) Listeria B) Gram + bacteria C) Pseudomonas D) Gram - bacteria

A) Listeria

Which of the following is a gram positive rod? A) Listeria B) Neisseria C) Legionella D) Salmonella

A) Listeria

Which of the following is/are covered by Amoxicillin (when used alone)? (select all that apply) A) Listeria B) MSSA C) Pseudomonas D) Actinomyces E) Neisseria Meningitides

A) Listeria D) Actinomyces E) Neisseria Meningitides

Which of the following is an opioid antidiarrheal (pathedine congener)? A) Loperamide B) Hyoscine C) Lubiprostone D) Mebeverine

A) Loperamide

Borrelia burgdorferi is a spirochete that causes: A) Lyme disease B) Syphilis

A) Lyme disease

Which of the following designations refers to colistin resistance? A) MCR-1 B) CRE C) Amp C

A) MCR-1

Nafcillin is useful due to its excellent coverage of: A) MSSA B) MRSA C) KPC D) ESBL

A) MSSA

Oxacillin covers: A) MSSA B) E. Faecalis C) Listeria D) Neisseria meningitides

A) MSSA

Which of the following is an osmotic laxative? A) Magnesium hydroxide B) Kaolin C) Bisacodyl D) Prucalopride

A) Magnesium hydroxide

Which of the following salts are commonly used as osmotic laxatives? (select all that apply) A) Magnesium sulfate B) Calcium carbonate C) Aluminum hydroxide D) Magnesium hydroxide

A) Magnesium sulfate D) Magnesium hydroxide

Unlike imipenem, Meropenem contains a methyl substitution group on the core ring structure. What is the major consequence of this methyl substitution? A) Makes the drug stable against renal dehydropeptidase B) Makes the drug more resistant to beta-lactamase C) Makes the drug more stable in stomach acid, and thus increases oral bioavailability D) Increases the affinity for PBPs in pseudomonas

A) Makes the drug stable against renal dehydropeptidase

Which of the following are used for the treatment of C. difficile colitis? (select all that apply) A) Metronidazole B) Rifamixin C) Fidaxomicin D) Vancomycin

A) Metronidazole C) Fidaxomicin D) Vancomycin

Which susceptibility test requires minimum drug concentration to inhibit growth. It is relevant to bacteriostatic drugs A) Minimum Inhibitory Concentration (MIC) B) Minimum Bactericidal Concentration (MBC)

A) Minimum Inhibitory Concentration (MIC)

Which of the following fluoroquinolones is best to use for Anaerobic bacteria such as B. fragilis and Clostridium spp.? A) Moxifloxacin B) Ofloxacin C) Ciprofloxacin D) Levofloxacin

A) Moxifloxacin

Which of the following fluoroquinolones is best used to treat abdominal infections? A) Moxifloxacin B) Ofloxacin C) Ciprofloxacin D) Levofloxacin

A) Moxifloxacin

Which of the following are found in the mycobacteria cell wall? A) Mycolic Acid B) Octanoic acid C) N-acetylmuramic acid D) Arabinogalactan

A) Mycolic Acid C) N-acetylmuramic acid D) Arabinogalactan

Which of the following are covered by Doxycycline? (select all that apply) A) Mycoplasma B) MSSA C) Chlamydia pneumoniae D) H. influenzae E) Klebsiella pneumoniae F) Pseudomonas aeruginosa

A) Mycoplasma B) MSSA C) Chlamydia pneumoniae D) H. influenzae

Identify all the species covered by Tigecycline: (select all that apply) A) Mycoplasma B) Pseudomonas C) MRSA D) KPC-positive E. Coli

A) Mycoplasma C) MRSA D) KPC-positive E. Coli

Which of the two sugars within the bacterial cell wall has a pentapeptide attached to it? A) NAM B) NAG

A) NAM

Which of the following drugs is/are parenteral only? (select all that apply) A) Nafcillin B) Cefaclor C) Aztreonam D) Meropenem E) Cephalexin

A) Nafcillin C) Aztreonam D) Meropenem

Which of the following are effective against MSSA? A) Nafcillin B) Ampicillin C) Cefazolin D) Aztreonam

A) Nafcillin B) Ampicillin C) Cefazolin

Which of the following is a 1st generation fluoroquinolone? A) Nalidixic acid B) Ciprofloxacin C) Ofloxacin D) Levofloxacin E) Moxifloxacin F) Gemifloxacin G) Delafloxacin H) Gatifloxacin

A) Nalidixic acid

Identify all the major adverse effects associated with Vancomycin: (select all that apply) A) Nephrotoxicity B) Hepatotoxicity C) Myelosuppression D) Red man syndrome E) Biliary sludging

A) Nephrotoxicity C) Myelosuppression D) Red man syndrome

Which of the following situations/patients require bactericidal antibiotics? (select all that apply) A) Neutropenic patients B) Endocarditis C) UTI D) Meningitis

A) Neutropenic patients B) Endocarditis D) Meningitis

Which of the following is true regarding neutrophils and bacterial infections? A) Neutrophils phagocytose bacteria B) Neutrophils produce antibodies that target bacteria

A) Neutrophils phagocytose bacteria

Use of which of the following agents is associated with the development of tolerance? A) Nitroglycerin B) Aspirin C) Abciximab D) Protamine

A) Nitroglycerin

Ethambutol follows ____ kinetics A) Non-linear B) Linear

A) Non-linear

This is a hospital acquired infection and it is more difficult to treat (resistant strains). Examples of this infection include Clostridium difficile and MRSA. A) Nosocomial Infection B) Superinfection

A) Nosocomial Infection

Sofosbuvir is a: A) Nucleotide B) Nucleoside

A) Nucleotide

Which of the following fluoroquinolones are used to treat uncomplicated and complicated UTI, Prostatitis, and GI tract infections? (select all that apply) A) Ofloxacin B) Levofloxacin C) Ciprofloxacin D) Moxifloxacin E) Gemifloxacin

A) Ofloxacin C) Ciprofloxacin

Pseudomonas Aeruginosa is usually not a problem in young healthy patient but typically infects patients with a compromised immune status. In this regard, this bacterial species is considered to be a(n): A) Opportunistic B) Super bug C) Nosocomial D) Highly pathogenic

A) Opportunistic

Which of the following is still able to inhibit synthesis of a cell wall containing D-Ala-D-Lac terminal peptide? A) Oritavancin B) Telavancin C) Vancomycin D) Daptomycin

A) Oritavancin

Which of the following will be most effective against VRSA? A) Oritavancin B) Dalbavancin C) Teicoplanin D) Telavancin

A) Oritavancin

Which of the following is a hydrophilic drug and comes as an oral capsule and suspension? A) Oseltamivir B) Zanamivir

A) Oseltamivir

Which of the following is an Ester Prodrug? A) Oseltamivir B) Zanamivir

A) Oseltamivir

Which of the following is has more resistance? A) Oseltamivir B) Zanamivir

A) Oseltamivir

Which of the following covers E. Faecalis? A) Pen G B) Aztreonam C) Cefotetan D) Cefuroxime E) Ceftriaxone

A) Pen G

Which of the following has the lower bioavailability? A) Penciclovir B) Famciclovir

A) Penciclovir

Which of the following examples illustrates an antagonistic effect of a drug combination? A) Penicillin + Tetracycline B) Penicillin + Aminoglycoside C) Antibiotic + Antifungal drug

A) Penicillin + Tetracycline

Class D are which kind of Beta-Lactamses? A) Penicillinases B) Cephalosporins D) ESBL, KPC E) NDM-1

A) Penicillinases

This is a small circular DNA. It is mobile, has adaptation, and resistance genes A) Plasmid DNA B) Chromosomal DNA

A) Plasmid DNA

Which of the following is susceptible to SMX + TMP? A) Pneumocystis jirovecii B) Clostridium difficile C) Rickettsiae D) VRE

A) Pneumocystis jirovecii

Class III antiarrhythmic drugs block _____ channels A) Potassium B) Calcium C) Magnesium D) Sodium

A) Potassium

Which electrolyte leaves the cells when channels open? A) Potassium B) Sodium C) Calcium D) Magnesium

A) Potassium

The following process is when ventricular muscle stretch from the amount of blood returning to the heart A) Preload B) Afterload

A) Preload

Sofosbuvir is a: A) Prodrug B) Bioprecursor C) Active metabolite

A) Prodrug

The following bacteria is resistant to many antibiotics because it expresses highly selective porins that are capable of excluding even small polar drugs A) Pseudomonas aeruginosa B) Klebsiella pneumoniae C) Staphyloccus aureus D) E. coli

A) Pseudomonas aeruginosa

Which of the following is well tolerated and safe to use? A) Pyrantel B) Ivermectin C) Albendazole/Mebendazole

A) Pyrantel

Which of the following nematodes causes paralysis in worms? A) Pyrantel B) Ivermectin C) Albendazole/Mebendazole

A) Pyrantel

Which of the following are true regarding the beta-lactam ring? (select all that apply) A) Reacts with and inactivates beta-lactams that it binds to B) Functions as a "suicide inhibitor" of many PBPs that it binds to C) It is not reactive unless fused to a secondary ring D) It is reactive because it contains an electrophilic carbonyl group E) It is reactive because it is under strain

A) Reacts with and inactivates beta-lactams that it binds to B) Functions as a "suicide inhibitor" of many PBPs that it binds to D) It is reactive because it contains an electrophilic carbonyl group E) It is reactive because it is under strain

H. Pylori infection A) Release of inflammatory cytokines B) Inhibits the Adenyl cyclase C) Increase in intracellular Ca2+ levels D) Release of proton in gastric mucosa

A) Release of inflammatory cytokines

Which of the following is a nucleoside that inhibits viral genome replication? A) Ribavirin B) Sofosbuvir

A) Ribavirin

Which of the following represents castor oil? A) Ricinolic acid B) Kaolin C) Bisacodyl D) Anthraqionone glycoside

A) Ricinolic acid

Which of the following is a potent inducer of CYP2E1 and causes serious drug interactions if taken with Isoniazid? A) Rifampin B) Pyridoxine C) Ethambutol D) Pyrazinamide

A) Rifampin

Which of the following drugs binds the Beta-subunit of DNA dependent RNA polymerase (DDRP or rpoB) at allosteric site? A) Rifampin (RMP) B) Isoniazid (INH) C) Pyrazinamide (PZA) D) Ethambutol (EMB)

A) Rifampin (RMP)

Pneumocystis jirovecci is covered by which of the following? (select all that apply) A) SMX + TMP B) Ceftazidime + Avibactam C) Clindamycin D) Ciprofloxacin

A) SMX + TMP C) Clindamycin

Pneumocystis jirovecci is covered by which of the following? (select all that apply) A) SMX + TMZ B) Ceftazidime + Avibactam C) Clindamycin D) Ciprofloxacin

A) SMX + TMZ C) Clindamycin

Which of the following is a concern when using imipenem + cilastatin in susceptible patients? A) Seizures B) Myopathy C) Nephrotoxicity D) Bleeding disorders

A) Seizures

Which of the following drugs affects maturation of HCV? (select all that apply) A) Simeprevir B) Dasabuvir C) Ribavirin D) Sofosbuvir E) Ledipasvir

A) Simeprevir B) Dasabuvir E) Ledipasvir

Methyl substitution makes meropenem, doripenem, and ertapenem ____ against the peptidase A) Stable B) Unstable

A) Stable

What is the name and shape of the bacteria responsible for Bacillus anthracis? A) Streptobacillus (Chain) B) Bacillus (Rod shaped) C) Coccus (Sphere) D) Streptococcus (Chain) E) Staphylococcus (Clusters)

A) Streptobacillus (Chain)

A patient with dental abscess is given Clindamycin for 10 days. Unfortunately, the patient goes on to develop pseudomembranous colitis. This colitis is best described as: A) Superinfection B) Opportunistic infection C) Nosocomial Infection

A) Superinfection

Which of the following about Prostacyclin (PGI2) is true? (select all that apply) A) Synthesized and secreted by the endothelium B) Decreases cAMP levels within platelets which inhibits platelet aggregation and granule release C) Increases cAMP levels within platelets which inhibits platelet aggregation and granule release D) Increases cAMP levels within the vascular smooth muscle cells and induces vascular smooth muscle relaxation E) Decreases cAMP levels within the vascular smooth muscle cells and induces vascular smooth muscle relaxation

A) Synthesized and secreted by the endothelium C) Increases cAMP levels within platelets which inhibits platelet aggregation and granule release D) Increases cAMP levels within the vascular smooth muscle cells and induces vascular smooth muscle relaxation

Jarisch-Herxheimer reaction is usually associated with the antibiotic treatment of: A) Syphilis B) Meningitis C) Gonorrhea D) Chlamydia

A) Syphilis

_____ and _____ are both approved for to be useful as anti-HIV as well as anti-HBV drugs A) Tenofovir B) Ribavirin C) Boceprevir D) Lamivudine

A) Tenofovir D) Lamivudine

Which of the following is true about Concentration-dependent killing? A) The greater the concentration, the greater (and faster) the killing effect B) We must aim for a Peak/MIC ration of 8-10 or more C) The higher the concentration the better the target will be reached D) Post-Antibiotic effect usually present E) Post-Antibiotic effect is not present F) Examples of drugs are aminoglycosides and fluoroquinolones

A) The greater the concentration, the greater (and faster) the killing effect B) We must aim for a Peak/MIC ration of 8-10 or more C) The higher the concentration the better the target will be reached D) Post-Antibiotic effect usually present F) Examples of drugs are aminoglycosides and fluoroquinolones

Which of the following is true about Oseltamivir and Zanamivir? A) They are antiviral drugs that are used for both prophylaxis and treatment of influenza A and B B) They are antiviral drugs that are used only for prophylaxis of influenza A and B C) They are antiviral drugs that are used only for treatment of influenza A and B

A) They are antiviral drugs that are used for both prophylaxis and treatment of influenza A and B

What consideration should be given to neutropenic patients? A) They may require more aggressive and timely antibiotic treatment B) Antibiotics will cause more adverse effects in these patients and should be avoided

A) They may require more aggressive and timely antibiotic treatment

Identify all major adverse effects associated with Linezolid: (select all that apply) A) Thrombocytopenia B) Serotonin syndrome C) Proteinurea D) Vestibular toxicity E) Bone marrow suppression

A) Thrombocytopenia B) Serotonin syndrome E) Bone marrow suppression

Which of the following drugs are ADP (P2Y) receptor antagonists? (select all that apply) A) Ticlopidine B) Clopidogrel C) Aspirin D) Vorapaxar E) Prasugrel

A) Ticlopidine B) Clopidogrel E) Prasugrel

Which of the following ADP (P2Y) receptor antagonists fall under the subset as Thienopyridines and bind irreversibly? A) Ticlopidine B) Ticagrelor C) Clopidogrel D) Cangrelor E) Prasugrel

A) Ticlopidine C) Clopidogrel E) Prasugrel

A spiral bacteria that causes syphillis A) Treponema Pallidum B) Haemophilus influenzae C) Mycoplasma pneumoniae D) streptococcus pyogenes E) Clostridium Difficile

A) Treponema Pallidum

Emtricitabine (FTC) + Tenofovir Disoproxil (TDF) = A) Truvada B) Descovy C) Atripla

A) Truvada

Which HAART are used for PrEP? (select all that apply) A) Truvada B) Descovy C) Atripla

A) Truvada B) Descovy

The horizontal transfer of resistance genes can occur between: (select all that apply) A) Two non-pathogenic factors B) Pathogenic and non-pathogenic species C) Two pathogenic species

A) Two non-pathogenic factors B) Pathogenic and non-pathogenic species C) Two pathogenic species

Which of the following do Linezolid, Daptomycin, and Streptogramins have in common? A) VRE coverage B) ESBL C) KPC coverage D) CAPES coverage

A) VRE coverage

Which of the following drugs are called as "Chain terminators"? (select all that apply) A) Valacyclovir B) Ganciclovir C) Penciclovir D) Cidofovir E) Idoxuridine F) Acyclovir

A) Valacyclovir F) Acyclovir

Which of the following polymerase inhibitors are considered prodrugs? (select all that apply) A) Valganciclovir B) Valacyclovir C) Famciclovir D) Acyclovir

A) Valganciclovir B) Valacyclovir C) Famciclovir

Which of the following has/have poor systemic absorption when administered orally? (select all that apply) A) Vancomycin B) Linezolid C) Fidaxomicin D) Rifamixin

A) Vancomycin

Which of the following has/have poor systemic absorption when administered orally? (select all that apply) A) Vancomycin B) Linezolid C) Fidaxomicin D) Rifamixin E) Gentamicin

A) Vancomycin C) Fidaxomicin D) Rifamixin E) Gentamicin

Which of the following are effects of TxA2? (select all that apply) A) Vasoconstriction B) Vasodilation C) Platelet Activation D) Platelet Aggregation

A) Vasoconstriction C) Platelet Activation D) Platelet Aggregation

Which of the following are the 2 Non-DHP calcium channel blockers (CCB's) that are in class IV of antiarrhythmic drugs? (select all that apply) A) Verapamil B) Amlodipine C) Diltiazem D) Nicardipine

A) Verapamil C) Diltiazem

Exposure to antibiotics causes selection pressure on bacteria, giving rise to a bacterial population that harbors resistance genes. This is: A) Vertical transfer B) Horizontal transfer C) Conjugative transfer D) Intrinsic resistance

A) Vertical transfer

The selection pressure caused by the presence of an antibiotic results in the ____ transfer of resistance genes A) Vertical transfer B) Horizontal transfer

A) Vertical transfer

This bug is present in the oral cavity as part of the normal flora but can cause endocarditis in susceptible patients A) Viridians Streptococci B) Enterococcus faecalis C) Staphylococcus aureus D) Streptococcus pyogenes

A) Viridians Streptococci

VKORC-1 is inhibited by: A) Warfarin B) Vitamin K 2,3 -epoxide C) Hydroquinone D) Prothrombin

A) Warfarin

Identify oral anticoagulants: (select all that apply) A) Warfarin B) Argatroban C) Rivaroxaban D) Dabigatron Etexilate

A) Warfarin C) Rivaroxaban D) Dabigatron Etexilate

Which clotting factors will be inhibited by Enoxaparin? A) Xa B) Thrombin (IIa) C) VIIa D) Ia

A) Xa

Which clotting factors will be inhibited by Fondaparinux? A) Xa B) Thrombin (IIa) C) VIIa D) Ia

A) Xa

Which clotting factors will be inhibited by Heparin? (select all that apply) A) Xa B) Thrombin (IIa) C) VIIa D) Ia

A) Xa B) Thrombin (IIa)

What is the abbreviation for Abacavir?

ABC

This helps in the release of TXA2 (Thromboxane A2) which causes vasoconstriction & granule release

ADP

Non-thienopyridines mimic purine, which receptors are they more selective to?

ADP receptors

Acyclovir and Valacyclovir are used for prophylaxis in which group of patients?

AIDS and immunocompromised patients

What is the coverage of Nitroimidazoles?

ALMOST ALL OF THE ANAEROBES: -Clostridium spp. (including C. difficile) -B. Fragilis -Peptostreptococcus -H. pylori Protozoa: -Trichomonas vaginalis -Entamoeba histolytica -Giardia

In signal transmission, the signal only enters ventricles and is delayed at which node?

AV node

This is a node that is in the middle between the atria and the ventricle

AV node

What is the abbreviation for Zidovudine?

AZT and ZDV

Define Arrhythmia

Abnormal rhythm known as dysrhythmias or irregular heartbeats

Excretion of Amantadine is increased in ____ urine

Acidic

Which Non-Enteric Gram - Rod causes: -Ventilator-pneumonia -Wounds -Resistance (All Hospital related)

Acinetobacter

Sarecycline is a narrow spectrum antibiotic used for the treatment of:

Acne

Which Gram + Rod Anaerobe causes: -Gum/oral disease/abscess -Endocarditis

Actinomyces

MOA of Nitrofurantoin:

Activated by bacterial reductase intermediates that causes DNA damage

High molecular weight heparin dose is determined by monitoring the:

Activated partial thromboplastin time (aPTT)

Nine Anti-HSV Nucleoside/nucleotide analogs (4a)

Acyclovir, Valacyclovir, Ganiciclovir, Valganiciclovir, Codofovir, Penciclovir, Femciclovir, Trifluridine, Idoxuridine

What does Amantadine and Rimantadine block?

Adamantanamine (M2 channel)

Five anti HepB Nucleoside/nucleotide analogs (4a)

Adefovir, entecavir, telbivudine, lamivudine, tenofovir

Adenine + sugar = ?

Adenosine

Two viral uncoating inhibitors for influenza

Amantadine, ramantadine

This aminoglycoside is a semisynthetic drug reserved for resistant-difficult to treat infections (Resistant pseudomonas)

Amikacin

Which drug was designed to overcome resistance of aminoglycosides?

Amikacin

What protein synthesis inhibitor binds to 30S, and forces errors?

Aminoglycosides

What are examples of Extended spectrum activity?

Aminopenicillins, anti-pseudomonals

How is amoxicillin structurally and functionally different from ampicillin?

Amoxicillin has an OH group (hydroxy) on the benzene ring and ampicillin doesn't. Improves solubility and oral absorption

What is the hypothesized MOA of Nitroimidazoles?

Anaerobic enzymes donate an e- to the nitro group which forms free radicals and damages DNA and proteins and causes a Bactericidal effect

What has been approved as an antidote for the oral factor Xa inhibitors: rivaroxaban, apixaban, and edoxaban

Andexanet alfa

The alpha-orientation of the dimethylamino group on ring A of tetracyclines are critical for:

Antibiotic Activity

Both Metronidazole and Tinidazole have similar spectra, but Tinidazole is reserved as an ______ drug

Antiparasitic

Heparin-based anticoagulants bind to _______

Antithrombin III

Interferon-a (IFN-a) works by increasing the formation of _____ _____ and promotion formation of natural killer cells that destroy infected liver cells

Antiviral proteins

Chloramphenicol is not used much anymore because it causes idiosyncratic, irreversible, dose-independent, fatal ____ _____

Aplastic anemia

What are the vascular effects of calcium channel blockers in angina?

Arteriolar vasodilators

Dipyridamole in combination with ____ to prevent ischemic (thrombotic) stroke in patients with a previous stroke

Aspirin

Which COX-1 inhibitor can be used as an antiplatelet drug?

Aspirin

Chlamydia pneumoniae

Atypical

Chlamydia trachomatis

Atypical

Legionella

Atypical

Mycoplasma Pneumoniae

Atypical

Rickettsia spp.

Atypical

The ability of the heart to generate and conduct electrical impulses on its own

Automaticity

Place the macrolides in order from best tissue distribution to least:

Azithromycin > Clarithromycin & Telithromycin > Erythromycin

Which generation cephalosporin is the best choice for treating MSSA? A) 2nd B) 1st C) 3rd

B) 1st

What is the pH of gastric mucous layer? A) 1.5-2.0 (acidic) B) 7.25-8.0 (basic)

B) 7.25-8.0 (basic)

Daptomycin should not be used in: A) A patient with SSTI involving vancomycin resistant enterococci B) A patient with pneumonia C) A patient with infective Enterococcal endocarditis

B) A patient with pneumonia

Which of the following causes hypersensitivity? A) Didanosine (DDI) B) Abacavir (ABC) C) Tenofovir disoproxil (TDF)

B) Abacavir (ABC)

Which of the following binds to GPIIb-IIIa receptor irreversibly and has a significant risk of bleeding? A) Eptifibatide B) Abciximab

B) Abciximab

Which of the following is a type of bacteria that has high content of mycolic acids in their cell walls? A) Corynebacterineae B) Acid-Fast C) Mycobacteria D) Mycolic Acid

B) Acid-Fast

Which of the following is NOT covered by the combination of Ampicillin + Sulbactam? A) MSSA B) Acinetobacter C) B. Fragilis D) Enterococcus Faecalis

B) Acinetobacter

Nosocomial infections (are): A) Acquired in community B) Acquired in hospitals C) Involve more than one strain D) Involve fungal co-infection

B) Acquired in hospitals

Which of the following events occur following the activation of PGE2 receptors on the parietal cells? A) Activation of Gs protein B) Activation of Gi Protein C) Inactivation of Gs Protein D) Inactivation of Gi Protein

B) Activation of Gi Protein

Which of the following mechanisms can propel the transportation of H+ ions in the gastric secretions? (select all that apply) A) Activation of PGE2 receptors B) Activation of protein kinases via release of stored Ca2+ C) Activation of adenylate cyclase resulting into elevated cAMP levels D) Blocking gastrin receptors E) H2 receptor antagonism

B) Activation of protein kinases via release of stored Ca2+ C) Activation of adenylate cyclase resulting into elevated cAMP levels

Which of the following is a nucleotide inhibitor due to its phosphate group? A) Entecavir B) Adefovir C) Telbivudine

B) Adefovir

Which of the following requires one less step in activation and thus takes much longer period of time before the virus develops resistance to it? (Its phosphate group helps with resistance) A) Entecavir B) Adefovir C) Telbivudine

B) Adefovir

Which process is associated with pumping blood out of the heart? A) Preload B) Afterload

B) Afterload

Which of the following antagonizes the action of plasmin? A) Factor IIa B) Alpha2-antiplasmin C) Fibrinogen D) Prothrombin

B) Alpha2-antiplasmin

Which of the following is most effective against resistant Pseudomonas? A) Gentamicin B) Amikacin C) Kanamycin D) Tobramycin

B) Amikacin

Which of the following has the best oral absorption? A) Pen G B) Amoxicillin C) Ampicillin D) Nafcillin

B) Amoxicillin

Moxifloxacin is the only fluoroquinolone that provides significant coverage of: A) Gram negatives B) Anaerobes C) Gram positives D) Atypicals

B) Anaerobes

Which of the following is an antidote (reversal agent) for Xabans (rivoraxaban, apixaban, edoxaban)? A) Idarucizumab B) Andexanet alfa C) Vitamin K D) Protamine E) Antifibrinolytic drugs

B) Andexanet alfa

This is a substance that inhibits growth of other microorganisms A) Antimicrobial Drug B) Antibiotic

B) Antibiotic

In mycobacteria cell wall synthesis N-acetylmuramic acid units are linked to ____ polymers A) Fatty acid synthetases 1 and 2 B) Arabinogalactan C) Mycolic acid

B) Arabinogalactan

Which of the following drugs are activated inside malaria food vacuole and has reactive metabolites that kill the parasite? A) Chloroquine B) Artemisinins C) Primaquine D) Quinine

B) Artemisinins

Binding of H2 antagonists to which of the following amino acid results into H2 receptor inactivation? A) Asp 98 B) Asp 186 C) Asp 196 D) Tyr 182

B) Asp 186

Which of the following agents inhibits cyclooxygenase? A) Clopidogrel B) Aspirin C) Eptifibatide D) Alteplase

B) Aspirin

Arrhythmias are usually _____, but can also be bothersome by causing palpitations A) symptomatic B) Asymptomatic

B) Asymptomatic

Renal excretion is NOT the major route of elimination for: A) Aztreonam B) Azithromycin C) Cefepime D) Gentamicin

B) Azithromycin

What is the name and shape of the bacteria responsible for E. Coli? A) Streptobacillus (Chain) B) Bacillus (Rod shaped) C) Coccus (Sphere) D) Streptococcus (Chain) E) Staphylococcus (Clusters)

B) Bacillus (Rod shaped)

_____ inhibits phosphate that regenerates bactoprenol, thus stopping cell wall synthesis in Stage II A) Beta-Lactams B) Bacitracin C) Fosfomycin D) Cycloserine

B) Bacitracin

Due to its high lipophilicity, Tigecycline is not very effective in patients with: A) SSTIs B) Bacteremia C) Intraabdominal Infections D) Pneumonia

B) Bacteremia

What is the chemical nature of Amantadine and Rimantadine? A) Acidic B) Basic

B) Basic

Which class of drugs are considered class II of antiarrhythmic drugs? A) Antidepressants B) Beta-Blockers C) Anti-bacterials D) Antiemetics

B) Beta-Blockers

Which of the following designations refers to carbapenem resistance? A) MCR-1 B) CRE C) Amp C

B) CRE

Prasugrel is activated by which CYP enzyme? A) CYP2D6 B) CYP3A4 C) CYP1A2 D) CYP2C19

B) CYP3A4

Which of the following is more selective? A) Ticagrelor B) Cangrelor

B) Cangrelor

Which of the following are 2nd generation cephamycins? (select all that apply) A) Cefuroxime B) Cefoxitin C) Cefotetan D) Cefprozil

B) Cefoxitin C) Cefotetan

Which of the following cover KPC (CRE) positive E. Coli strains? (select all that apply) A) Imipenem + Cilastatin B) Ceftazidime + avibactam C) Meropenem + Vaborbactam D) Imipenem + cilastatin + relebactam

B) Ceftazidime + avibactam C) Meropenem + Vaborbactam D) Imipenem + cilastatin + relebactam

Which of the following is mainly excreted through biliary route? A) Daptomycin B) Ceftriaxone C) Vancomycin D) Piperacillin

B) Ceftriaxone

Which of the following 2nd generation cephalosporins are administered orally? (select all that apply) A) Cefuroxime B) Cefuroxime axetil C) Cefaclor D) Cefprozil

B) Cefuroxime axetil C) Cefaclor D) Cefprozil

Class C are what kind of Beta-Lactamases? A) Penicillinases B) Cephalosporins D) ESBL, KPC E) NDM-1

B) Cephalosporins

Lamivudine works by which of the following mechanisms? (select all that apply) A) Forming faulty nucleic acid by chain elongation B) Chain termination C) Blocking reverse transcriptase D) Blocking allosteric site of the enzyme

B) Chain termination C) Blocking reverse transcriptase

Which of the following are the mechanisms shared by both gram negative and positive bacteria acquiring resistance to beta-lactams? A) Change in porins B) Change in PBP C) Production of beta-lactamases

B) Change in PBP C) Production of beta-lactamases

This is a relatively stable DNA and gives genetic identity A) Plasmid DNA B) Chromosomal DNA

B) Chromosomal DNA

Which of the following is the active metabolite of Cidofovir? A) Cidofovir-Phosphocholine B) Cidofovir-Diphosphate

B) Cidofovir-Diphosphate

Imipenem is combined with cilastatin because: A) Cilastatin inhibits beta-lactamases B) Cilastatin inhibits renal dipeptidases C) Cilastatin increases oral bioavailability D) Cilastatin decreases renal secretion of Imipenem

B) Cilastatin inhibits renal dipeptidases

Which of the following is most likely to be used in a complicated UTI? A) Levofloxacin B) Ciprofloxacin C) Gemifloxacin D) Moxifloxacin

B) Ciprofloxacin

Which of the following fluoroquinolones are best used for atypical bacteria such as Mycobacteria and B. anthracis? (select all that apply) A) Ofloxacin B) Ciprofloxacin C) Levofloxacin D) Moxifloxacin

B) Ciprofloxacin C) Levofloxacin D) Moxifloxacin

Which of the following are 2nd generation fluoroquinolones? (select all that apply) A) Nalidixic acid B) Ciprofloxacin C) Ofloxacin D) Levofloxacin E) Moxifloxacin F) Gemifloxacin G) Delafloxacin H) Gatifloxacin

B) Ciprofloxacin C) Ofloxacin

Which of the following drugs can produce synergistic actions with PPI's? A) Clopidogrel B) Clarithromycin C) Sucralfate

B) Clarithromycin

Which of the following is/are metabolized by CYP3A4? (select all that apply) A) Azithromycin B) Clarithromycin C) Erythromycin D) Telithromycin

B) Clarithromycin C) Erythromycin D) Telithromycin

Which subclass of class I drugs have a mild Na+ channel block and a shorten repolarization of K+ channels? A) Class IA B) Class IB C) Class IC

B) Class IB

Identify all the species covered by metronidazole: (select all that apply) A) Streptococcus pneumoniae B) Clostridium difficile C) Klebsiella pneumoniae D) Trichomonas vaginalis E) H. pylori

B) Clostridium difficile D) Trichomonas vaginalis E) H. pylori

Rifampin (RMP) is which of the following? (select all that apply) A) AUC/MIC dependent B) Concentration dependent C) Bactericidal D) Bacteriostatic

B) Concentration dependent C) Bactericidal

As far as microbial coverage goes, what is the major advantage of Tedizolid over Linezolid? A) Additional coverage of enteric Gram negatives B) Coverage of Linezolid-resistant staphylococci C) Additional coverage of non-enteric Gram negatives

B) Coverage of Linezolid-resistant staphylococci

____ inhibits Racemase (conversion of L-Ala to D-Ala) A) Fosfomycin B) Cycloserine C) Bacitracin

B) Cycloserine

Amino Acid residue ____ present on the proton pump is the essential site for binding for all PPI's A) Asp 98 B) Cys 813 C) Asp 196 D) Tyr 182

B) Cys 813

Torsades de pointes (TDP) defines as a ______ arrhythmia A) Safe B) Dangerous

B) Dangerous

Myopathy is a concern in patients taking this drug while on statins: A) Vancomycin B) Daptomycin C) Cilostin D) Doripenem

B) Daptomycin

Myopathy is a concern in patients taking this drug while on statins: A) Vancomycin B) Daptomycin C) Colistin D) Doripenem

B) Daptomycin

Bile acid sequestrants _____ warfarin absorption A) Increase B) Decrease

B) Decrease

Inadequate diet and antibiotics can cause a _____ of vitamin K in a patient A) Overload B) Deficiency

B) Deficiency

Which of the following fluoroquinolones has a Better MRSA coverage than all other fluoroquinolones and is therefore useful in skin/soft tissue infections caused by resistant strains? A) Ciprofloxacin B) Delafloxacin C) Ofloxacin D) Levofloxacin

B) Delafloxacin

What major advantage does Delafloxacin have over Ciprofloxacin? A) Delafloxacin is available as an oral drug while ciprofloxacin is a parenteral drug B) Delafloxacin has a better MRSA coverage C) Delafloxacin has a better gram negative coverage

B) Delafloxacin has a better MRSA coverage

A patient who has an allergy to sulfa should avoid which of the following anti-virals? (select all that apply) A) Fosamprenavir B) Delaviridine C) Amprenavir D) Tipranavir

B) Delaviridine C) Amprenavir D) Tipranavir

Emtricitabine (FTC) + Tenofovir Alafenamide (TAF) = A) Truvada B) Descovy C) Atripla

B) Descovy

Trimethoprim inhibits: A) Dihydropteroate synthase B) Dihydrofolate reductase C) Topoisomerase D) Gyrase

B) Dihydrofolate reductase

Sulfonamides inhibit which of the following? A) Dihydrofolate reductase B) Dihyrdopteroate synthase C) Topoisomerases

B) Dihyrdopteroate synthase

Sulfonamides inhibit: A) Dihydrofolate reductase B) Dihyrdopteroate synthase C) Topoisomerases

B) Dihyrdopteroate synthase

Which of the following is the primary site of absorption of PPI's? A) Stomach (acidic) B) Duodenum (basic) C) Small intestines

B) Duodenum (basic)

Dalfopristin and Quinupristin are: A) Orally bioavailable B) Easily degraded in the stomach

B) Easily degraded in the stomach

Which of the following Integrase inhibitor drugs is used in combination with Cobicistat? A) Raltegravir B) Elvitegravir C) Dolutegravir

B) Elvitegravir

Which of the following are Anti-HIV drugs that inhibit viral attachment and entry? (select all that apply) A) Docosanol B) Enfuvirtide C) Maraviroc

B) Enfuvirtide C) Maraviroc

Lamivudine (3TC) + Abacavir (ABC) = A) Comvibir B) Epzicom C) Trizivir

B) Epzicom

Which of the following NNRTIs are considered to be 2nd generation? (select all that apply) A) Efavirenz B) Etravirine C) Rilpivirine D) Doravirine

B) Etravirine C) Rilpivirine

Which of the following best describes the coverage of Ceftaroline? A) Excellent coverage of Gram negative including pseudomonas but not KPC B) Excellent coverage of Gram positives including MRSA and VRSA C) Excellent coverage of Gram negatives including pseudomonas and KPC D) Excellent coverage of Gram positives including MRSA but not VRSA

B) Excellent coverage of Gram positives including MRSA and VRSA

Which of the following best describes the coverage of Ceftolozane + Tazobactam? A) Excellent gram positive coverage, including MRSA but not VRSA B) Excellent gram negative coverage, including pseudomonas and ESBL strains C) Excellent gram positive coverage, including MRSA and VRSA D) Excellent gram negative coverage, including pseudomonas but not ESBL strains

B) Excellent gram negative coverage, including pseudomonas and ESBL strains

These are toxins that are synthesized and released by bacteria. It is produced by tetanus, botulin, and anthrax A) Endotoxin B) Exotoxin

B) Exotoxin

Botulin toxin is an example of: (select all that apply) A) Endotoxin B) Exotoxin C) Virulence Factor D) Resistance Factor

B) Exotoxin C) Virulence Factor

Which of the following is a prodrug? A) Penciclovir B) Famciclovir

B) Famciclovir

This bacterial structure has "hair-like" projections and helps bacteria attach to mucosal surfaces (Ex: lungs, throat, bladder) and prevents bacteria from being washed out. It is a virulence factor and is an example of E. coli A) Flagella B) Fimbriae

B) Fimbriae

Which of the following drugs is responsible for electrolyte abnormalities? A) Acyclovir B) Foscarnet C) Docosanol D) Trifluridine

B) Foscarnet

In which of the given pairs the two drugs have almost identical coverage? A) Gentamicin and Azithromycin B) Gentamicin and Aztreonam C) Gentamicin and Amoxicillin D) Gentamicin and Clindamycin

B) Gentamicin and Aztreonam

Which of the following will contain endotoxin? A) Gram + bacteria B) Gram - bacteria

B) Gram - bacteria

Histamine H2 receptor activation on the parietal cells activates which protein? A) Gi B) Gs

B) Gs

Which of the following acts as a gateway for transporting H+ ions from the gastric parietal cells? A) Na+/K+ ATPase pump B) H+/K+ ATPase pump

B) H+/K+ ATPase pump

A vaccine is not available for which of the following infections? A) HAV B) HCV C) Zoster D) Influenza A

B) HCV

Posaconazole is used as prophylaxis against Candida & Aspergillus in which group of patients? A) Diabetic Patients B) HIV Patients C) Hypertensive Patients D) Hypotensive Patients

B) HIV Patients

Which of the following are RNA based viruses? A) HSV, Varicella, Cytomegalovirus, Hep B B) HIV, Hepatitis A & C, Influenza

B) HIV, Hepatitis A & C, Influenza

A non-enteric gram negative rod involved in upper RTI's A) Treponema Pallidum B) Haemophilus influenzae C) Mycoplasma pneumoniae D) streptococcus pyogenes E) Clostridium Difficile

B) Haemophilus influenzae

Which of the following neurotransmitters are stored from the gastric paracrine cells? A) Serotonin B) Histamine

B) Histamine

Which of the following is an antimuscarinic spasmolytic agent? A) Loperamide B) Hyoscine C) Lubiprostone D) Mebeverine

B) Hyoscine

Which class of antiarrhythmic drugs are Beta-Blockers that inhibit phase 4 depolarization in SA and AV nodes? A) IB B) II C) III D) IA E) IC F) IV

B) II

Ampicillin differs from Pen G by a single amino group. What does this group do? (select all that apply) A) Protect the beta-lactam ring from beta-lactamases B) Increase the polarity and Gram negative coverage C) Increase the stability of the beta-lactam ring in acid

B) Increase the polarity and Gram negative coverage C) Increase the stability of the beta-lactam ring in acid

Which of the following events might be a potential treatment strategy for PUD? (select all that apply) A) Reduced gastric emptying B) Increased Gastric emptying C) Increased eradication of H. pylori infection D) Increased wound healing

B) Increased Gastric emptying C) Increased eradication of H. pylori infection D) Increased wound healing

Which of the following are covered by cefazolin (assuming susceptible strain)? (select all that apply) A) Moraxella Catarrhalis B) Indole - proteus C) Indole + proteus D) MSSA E) Klebsiella pneumoniae F) Serratia G) E. Coli H) Staphylococcus saprophyticus

B) Indole - proteus D) MSSA E) Klebsiella pneumoniae G) E. Coli H) Staphylococcus saprophyticus

Rifampin is the strongest known ____ of CYP450 system A) Inhibitor B) Inducer

B) Inducer

Prostaglandin E2 receptors activation A) Release of inflammatory cytokines B) Inhibits the Adenyl cyclase C) Increase in intracellular Ca2+ levels D) Release of proton in gastric mucosa

B) Inhibits the Adenyl cyclase

Damaged surface stimulated by atherosclerosis or artificial heart valve activates which pathway? A) Extrinsic Pathway B) Intrinsic Pathway C) Common Pathway

B) Intrinsic Pathway

Mycoplasma do not contain the cell wall and are thus resistant to penicillins. This is an example of: A) Acquired resistance B) Intrinsic resistance C) PAE

B) Intrinsic resistance

Beta-Lactams: A) Directly bind to peptidoglycan B) Irreversibly bind to Penicillin Binding proteins C) Competitively inhibit peptidoglycan synthase

B) Irreversibly bind to Penicillin Binding proteins

Which of the following has a black boxed warning of severe fatal hepatitis that may occur late in therapy with age being a risk factor? A) Rifampin (RMP) B) Isoniazid (INH) C) Pyrazinamide (PZA) D) Ethambutol (EMB)

B) Isoniazid (INH)

Which of the following is a prodrug that is activated within bacteria by enzyme KatG to isonicotinoyl radical that forms adduct with NAD. It inhibits InhA and KasA and inhibits mycolic acid synthesis (cell death) A) Rifampin (RMP) B) Isoniazid (INH) C) Pyrazinamide (PZA) D) Ethambutol (EMB)

B) Isoniazid (INH)

Probenecid can be co-administered with Pen G because: A) Is a beta lactamase inhibitor B) It decreases renal secretion of Pen G C) It protects Pen G from hydrolysis by renal peptidase D) It protects Pen G from hepatic inactivation

B) It decreases renal secretion of Pen G

Which of the following best describes the use of Nitrofurantoin? A) It has excellent tissue distribution and can be used for pyelonephritis B) It reaches high concentrations in urine but not enough in serum for systemic infections C) It has poor absorption and is available only as a parenteral drug

B) It reaches high concentrations in urine but not enough in serum for systemic infections

Which of the following is usually very well tolerated but can cause skin, rash and GI symptom reactions as adverse effects? A) Pyrantel B) Ivermectin C) Albendazole/Mebendazole

B) Ivermectin

Which of the following nematodes binds to CI channels in worms causing influx of CI, disrupting function (paralysis of worm muscle) A) Pyrantel B) Ivermectin C) Albendazole/Mebendazole

B) Ivermectin

Which of the following is an adsorbant antidiarrheal agent? A) Ricinolic acid B) Kaolin C) Bisacodyl D) Anthraqionone glycoside

B) Kaolin

Aztreonam covers: A) Mycoplasma pneumoniae B) Klebsiella Pneumoniae C) Bacteroides fragilis D) Streptococcus pneumoniae

B) Klebsiella Pneumoniae

A gram negative rod that is common cause of UTI, but can also cause hospital acquired pneumonia A) Haemophilus influenzae B) Klebsiella pneumoniae C) Chlamydia pneumoniae D) Streptococcus pneumoniae E) Mycoplasma pneumoniae

B) Klebsiella pneumoniae

Which of the following Nucleoside/nucleotide analogs are Anti-Hep B drugs as well as Anti-HIV drugs? (select all that apply) A) Sofosburvir B) Lamivudine C) Tenofovir D) Ribavirin

B) Lamivudine C) Tenofovir

Which of the following fluoroquinolones are used to treat Upper respiratory tract infections, CAP, skin and soft structure infections? (select all that apply) A) Ofloxacin B) Levofloxacin C) Ciprofloxacin D) Moxifloxacin E) Gemifloxacin

B) Levofloxacin D) Moxifloxacin E) Gemifloxacin

Serotonin syndrome is an adverse effect most commonly associated with: A) Daptomycin B) Linezolid C) Streptogramins D) Chloramphenicol

B) Linezolid

Serotonin syndrome is an adverse effect most commonly associated with: A) Daptomycin B) Linezolid C) Streptogrammins D) Chloramphenicol

B) Linezolid

This gram positive rod is not usually a concern in healthy patients but can cause serious infections, including meningitis, in patients who are immune compromised, elderly or pregnant A) Bacillus anthracis B) Listeria C) Streptococcus agalactiae D) Neisseria meningitides

B) Listeria

None of the cephalosporins are effective against the following species: (select all that apply) A) Neisseria gonorrhoeae B) Listeria meningitides C) Enterococcus faecalis D) Borrelia Burgdorferi

B) Listeria meningitides C) Enterococcus faecalis

Which of the following complexes is more selective for inhibition of Xa compared to thrombin? A) High molecular weight Heparin (HMWH) B) Low molecular weight Heparin (LMWH)

B) Low molecular weight Heparin (LMWH)

What drugs bind to 50S in Bacterial protein synthesis? (select all that apply) A) Aminoglycosides B) Macrolides C) Tetracyclines D) Clindamycin E) Chloramphenicol F) Streptogramins G) Oxazolidinones (Linezolid) H) Telithromycin

B) Macrolides D) Clindamycin E) Chloramphenicol F) Streptogramins G) Oxazolidinones (Linezolid) H) Telithromycin

What drugs share binding sites on 50S in Bacterial protein synthesis? (select all that apply) A) Aminoglycosides B) Macrolides C) Tetracyclines D) Clindamycin E) Chloramphenicol F) Streptogramins G) Oxazolidinones (Linezolid) H) Telithromycin

B) Macrolides D) Clindamycin E) Chloramphenicol H) Telithromycin

Which antiretroviral is approved for CCR-5 tropic HIV-1 virus? A) Enfuvirtide B) Maraviroc C) Fostemsavir

B) Maraviroc

Which of the following is used for treatment as well as prophylaxis? A) Artemisinins B) Mefloquine C) Chloroquine D) Primaquine

B) Mefloquine

Which of the following is an immunosuppressive? A) Infliximab B) Methotrexate C) Amitriptyline D) Loperamide

B) Methotrexate

Which of the following is/are used for C. diff colitis? (select all that apply) A) Clindamycin B) Metronidazole C) Vancomycin D) Fidaxomicin E) Lefamulin

B) Metronidazole C) Vancomycin D) Fidaxomicin

Which of the following is used for constipation? A) Infliximab B) Milk of magnesia C) Prednisolone D) Loperamide

B) Milk of magnesia

Which susceptibility test requires minimum drug concentration to kill bacteria. It is relevant to bactericidal drugs. A) Minimum Inhibitory Concentration (MIC) B) Minimum Bactericidal Concentration (MBC)

B) Minimum Bactericidal Concentration (MBC)

Which 2 drugs are better able to penetrate bacteria because they don't undergo dehydration? (select all that apply) A) Tetracycline B) Minocycline C) Doxycyxline

B) Minocycline C) Doxycyxline

Which of the following best describes the antimicrobial coverage of Daptomycin? A) Most gram negatives, except pseudomonas B) Most gram positives, including VRE C) Most gram negatives, including pseudomonas D) Most gram positives, except VRE

B) Most gram positives, including VRE

Which of the following has the best anaerobic coverage? A) Levofloxacin B) Moxifloxacin C) Gemifloxacin D) Ciprofloxacin

B) Moxifloxacin

The bacterial cell wall contains: (select all that apply) A) Fatty acids B) N-acetylmuramic acid C) N-acetylglucosamine D) Peptides E) Cholesterol

B) N-acetylmuramic acid C) N-acetylglucosamine D) Peptides

Which of the two sugars is modified by peptide addition? A) NAG B) NAM

B) NAM

Dasabuvir is a _____ inhibitor A) NS3/4A B) NS5B C) NS5A D) Nucleotide

B) NS5B

Which of the following is more likely to be associated with hypokalemia? A) Oxacillin B) Nafcillin C) Cephalexin D) Amoxicillin

B) Nafcillin

Which of the following is/are not exclusively excreted by the kidneys? (select all that apply) A) Cefazolin B) Nafcillin C) Aztreonam D) Ertapenem E) Ceftriaxone

B) Nafcillin E) Ceftriaxone

This type of spectrum is selective against a few species A) Broad Spectrum B) Narrow spectrum C) Extended Spectrum

B) Narrow spectrum

The cell wall of Gram ____ is contained within the periplasmic space A) Positive B) Negative

B) Negative

Gram negative cocci that causes a difficult to treat STD A) E. coli B) Neisseria gonorrhoeae C) Staphyloccus Saprophyticus D) Moraxella Catarrhalis

B) Neisseria gonorrhoeae

The major toxicological concerns regarding Polymyxins are? A) Hepatotoxicity and myopathy B) Nephrotoxicity and Neuropathy C) Visual disturbance and metallic taste

B) Nephrotoxicity and Neuropathy

This enzyme converts analogue of nicotinamide Pyrazinamide to Pyrazinoic Acid A) DDRP B) Nicotinamidase C) Multi-drug resistant TB (MDR-TB) D) Extensively-drug resistant TB (XDR-TB)

B) Nicotinamidase

Pyrazinamide (PZA) is primarily active against _____ mycobacteria A) Growing B) Non-growing

B) Non-growing

Sofosbivur is a _____ analogue and a chain _____ A) Nucleoside; elongator B) Nucleotide; terminator C) Nucleotide; elongator D) Nucleoside; terminator

B) Nucleotide; terminator

Which of the following is a modified tetracycline that can overcome resistance by avoiding efflux and by binding to modified ribosomes? A) Augmentin B) Omadacycline C) Vancomycin D) Clindamycin

B) Omadacycline

The pH of mucous membrane is neutral due to the presence of: (select all that apply) A) Histamine B) PGE2 C) PGI2 D) Bicarbonates

B) PGE2 C) PGI2 D) Bicarbonates

Which of the following covers Treponema pallidum? A) Cloxacillin B) Pen G C) Cefazolin D) Cefaclor

B) Pen G

Which of the following has the narrowest coverage spectrum? A) Nafcillin B) Pen G C) Amoxicillin D) Zosyn

B) Pen G

Which of the following is/are considered to be extended spectrum antibiotics? (select all that apply) A) Dicloxacillin B) Piperacillin C) Penicillin G D) Ampicillin

B) Piperacillin D) Ampicillin

Which of the following cover pseudomonas aeruginosa? (select all that apply) A) Ampicillin + sulbactam B) Piperacillin + Tazobactam C) Nafcillin D) Meropenem E) Aztreonam

B) Piperacillin + Tazobactam D) Meropenem E) Aztreonam

Which aspect of bacterial genome gives it the ability to adapt to changing environment? A) Chromosomal DNA B) Plasmid DNA

B) Plasmid DNA

Which of the following types of antibiotics will likely enter the periplasmic space of Gram - bacteria? (select all that apply) A) Non-polar B) Polar C) Less than 7000 dalton in size D) Less than 700 dalton in size

B) Polar D) Less than 700 dalton in size

This class IC agent is also a Beta-blocker and its S-enantiomer is more potent A) Flecainide B) Propafenone C) Quinidine D) Lidocaine

B) Propafenone

The major concern with using Clindamycin is: A) Nephrotoxicity B) Pseudomembranous colitis C) CNS toxicity D) Hepatoxicity

B) Pseudomembranous colitis

Isoniazid should be administered concurrently with ______ to attenuate adverse drug reactions A) Rifampin B) Pyridoxine C) Ethambutol D) Pyrazinamide

B) Pyridoxine

Sofosbuvir is a derivative of: A) Purine B) Pyrimidine

B) Pyrimidine

HIV (HIV-1 & HIV-2) is considered what kind of virus based? A) DNA B) RNA

B) RNA

Hepatitis (A & C) is considered what kind of virus based? A) DNA B) RNA

B) RNA

Influenza (Types A,B,C) is considered what kind of virus based? A) DNA B) RNA

B) RNA

Which H2 antagonists can interact with the asparagine 159 (Asn 159) residue on the H2 receptor membrane? (select all that apply) A) Cimetidine B) Ranitidine C) Nizatidine D) Famotidine

B) Ranitidine C) Nizatidine

Activation of M1 and gastrin receptors on paracrine cells A) Release of inflammatory cytokines B) Release of Histamine C) Activates Adeneyl cyclase D) Release of proton in gastric mucosa

B) Release of Histamine

Which of the following describes the function of Phosphate? A) Catalyzes NAG-NAM complex by taking it outside and flipping it over to the cell wall outside of the cell wall membrane B) Removes phosphate and recycles bactoprenol C) Takes NAG-NAM complex outside of the inner layer

B) Removes phosphate and recycles bactoprenol

NRTIs inhibit which of the following enzymes? A) DNA polymerase B) Reverse transcriptase C) RNA polymerase D) Protease E) Integrase

B) Reverse transcriptase

Which of the following are NOT absorbed systemically but can be used orally for GI infections? (select all that apply) A) Rifampin B) Rifamixin C) Fidaxomicin D) Vancomycin

B) Rifamixin C) Fidaxomicin D) Vancomycin

Which of the following does NOT require dosage adjustments in renal failure? A) Amantadine B) Rimantadine

B) Rimantadine

Which of the following undergoes liver metabolism and is excreted as metabolites in unchanged drug form in the urine? A) Amantadine B) Rimantadine

B) Rimantadine

Endotoxin is present in: (select all that apply) A) Streptococci B) Salmonella C) E. coli D) Mycoplasma

B) Salmonella C) E. coli

Which new tetracycline is approved for the treatment of acne? A) Omadacycline B) Sarecycline

B) Sarecycline

Coagulation cascade is also termed as: A) Primary Hemostasis B) Secondary Hemostasis

B) Secondary Hemostasis

Which electrolytes enter the cells when channels open? (select all that apply) A) Potassium B) Sodium C) Calcium D) Magnesium

B) Sodium C) Calcium

Which of the following is a nucleotide that inhibits viral genome replication? A) Ribavirin B) Sofosbuvir

B) Sofosbuvir

This is a class III agent that is a K+ channel blocker and is also a beta-blocker. It is useful in patients who cannot tolerate amiodarone. A) Dronedarone B) Sotalol C) Amiodarone D) Ibutilide

B) Sotalol

This species can cause "strep" throat but can also cause necrotizing fasciitis, thus recognized as a "flesh eating bacteria" A) Staphylococcus aureus B) Streptococcus pyogenes C) Enterococcus faecalis D) Streptococcus pneumoniae

B) Streptococcus pyogenes

Which of the following metabolites is a sulfonamide moiety? A) 5-Aminosalicylic Acid B) Sulfapyridine

B) Sulfapyridine

Patient X is prescribed ciprofloxacin to treat her UTI. After a few days, she develops C. diff colitis. This is an example of: A) Acquired resistance B) Superinfection C) Opportunistic infection D) Postantibiotic effect

B) Superinfection

This is a secondary infection that arises after initial treatment of the original infection. ("On top off") A) Nosocomial Infection B) Superinfection

B) Superinfection

Treponema pallidum is a spirochete that causes: A) Lyme disease B) Syphilis

B) Syphilis

Which of the following has the longer half-life? A) Linezolid B) Tedizolid

B) Tedizolid

Which Semisynthetic glycopeptide has a tighter binding with D-Ala-D-Ala and also has a greater potential to disrupt the membranes due to the lipid group? A) Dalbavancin B) Telavancin C) Oritavancin

B) Telavancin This extends the telavancin coverage to include some VRSA, but it is not reliable (Oritavancin is preferred)

Which Anti-Hep B medications are also used for HIV? (select all that apply) A) Adefovir B) Tenofovir C) Telbivudine D) Lamivudine

B) Tenofovir D) Lamivudine

Which clotting factors will be inhibited by Bivalirudin? A) Xa B) Thrombin (IIa) C) VIIa D) Ia

B) Thrombin (IIa)

Which of the following ADP (P2Y) receptor antagonists fall under the subset as Non-thienopyridine (nucleoside analogues) and bind reversibly? A) Ticlopidine B) Ticagrelor C) Clopidogrel D) Cangrelor E) Prasugrel

B) Ticagrelor D) Cangrelor

What are the antipseudomonal Penicillins? (select all that apply) A) Amoxicillin B) Ticarcillin C) Piperacillin D) Cephalexin

B) Ticarcillin C) Piperacillin

Which of the following best defines PAE? A) Time it takes for the toxic effects to show up B) Time during which the bacteria do not grow even when the drug concentration is below MIC C) Paradoxical effect where the drug does not inhibit the bacterial growth in vivo even though it does so in vitro D) Time it takes for the first dose of the drug to work

B) Time during which the bacteria do not grow even when the drug concentration is below MIC

Which of the following glycoprotein IIb-IIIa receptor antagonists are competitive, reversible inhibitors? (select all that apply) A) Abciximab B) Tirofiban C) Eptifibatide D) Defibrotide

B) Tirofiban C) Eptifibatide

Why is it advised to administer oral lubricant (mineral oil) in an upright position? A) To avoid it spilling on the patient body B) To avoid lipid pneumonia

B) To avoid lipid pneumonia

DNA elements that can "jump" from one site to another are known as: A) Conjugative plasmids B) Transposons

B) Transposons

Colistin coverage includes all of the following EXCEPT: A) Citrobacter B) Treponema Pallidum C) Pseudomonas D) E. Coli --KPC +

B) Treponema Pallidum

Which of the following inhibits dihydrofolate reductase? A) Sulfamethoxazole B) Trimethoprim C) Ciprofloxacin D) Metronidazole

B) Trimethoprim

Select all the organisms that are susceptible to Synercid: (select all that apply) A) VRE (E. faecalis) B) VRSA C) VRE (E. faecium) D) MRSA E) Klebsiella pneumoniae

B) VRSA C) VRE (E. faecium) D) MRSA

Which of the following has the higher bioavailability? A) Acyclovir B) Valacyclovir

B) Valacyclovir

Which of the following is a prodrug? A) Acyclovir B) Valacyclovir

B) Valacyclovir

Which of the following polymerase inhibitors have chain termination? (select all that apply) A) Valganciclovir B) Valacyclovir C) Famciclovir D) Acyclovir

B) Valacyclovir D) Acyclovir

Which of the following has a higher bioavailability? A) Ganciclovir B) Valganciclovir

B) Valganciclovir

Which of the following is a prodrug? A) Ganciclovir B) Valganciclovir

B) Valganciclovir

"Red man syndrome" is most commonly associated with: A) Azithromycin B) Vancomycin C) Rifampin D) Ciprofloxacin

B) Vancomycin

"Red man syndrome" is most commonly associated with: A) Azithromycin B) Vancomycin C) Rifampin D) Ciprofloxacin

B) Vancomycin

Which of the following disease states is not dependent on atherosclerosis? A) Stable angina B) Variant angina C) Unstable angina

B) Variant angina

How is voriconazole best absorbed? A) With Food B) Without Food

B) Without Food

Which of the following anti-influenza drugs inhibit viral release? (select all that apply) A) Amantadine B) Zanamavir C) Rimantadine D) Oseltamivir E) Peramivir

B) Zanamavir D) Oseltamivir E) Peramivir

Which of the following polar lipophilic drug and comes as an inhalation powder? A) Oseltamivir B) Zanamivir

B) Zanamivir

Which of the following platelet glycoprotein interacts directly with collagen? A) glycoprotein V (GPV) B) glycoprotein VI (GPVI) C) glycoprotein VII (GPVII) D) glycoprotein VIII (GPVIII)

B) glycoprotein VI (GPVI)

What are the anaerobes coverage of Carbapenems?

B. Fragilis (and many others but not C. diff)

Quinupristin + Dalfopristin has a wide good tissue distribution but poor ___ penetration

BBB

What bacteria has a "rod shaped" appearance?

Bacillus

What type of bacteria is E. Coli and what is its appearance?

Bacillus ("rod shaped")

Which Gram + Rod causes: -Spore forming, anthrax

Bacillus anthracis

Bactericidal drugs kills ____ and causes no regrowth upon drug removal

Bacteria

How does resistance of Daptomycin occur?

Bacteria adds lysine on cell surface which repels positively charged Daptomycin

What is an intrinsic resistance?

Bacteria lacks the target for the drug (Ex: Mycoplasma pneumonia (No cell wall - atypical)

The inhibitors of cell wall synthesis are typically ____

Bactericidal

Unlike most other protein synthesis inhibitors, aminoglycosides are ____, and that this may be related to their unique effect on bacterial proteins synthesis in incorporating the incorrect amino acid

Bactericidal

Which Gram - Rod Anaerobe causes: -Normal flora; peritoneal/wound/surgical infections

Bacteroides

What does a nucleoside consist of?

Base + sugar

What does a nucleotide consist of?

Base + sugar + phosphate

What are the criteria upon which generational classification of cephalosporin is determined?

Based on antimicrobial activity & susceptibility to beta lactamases

Why are serine proteases in the body not affected by penicillin?

Because the body does not recognize D-Ala-D-Ala easily

Why are gram negative bacteria so difficult to kill?

Because the drug first has to enter through porins and enter the periplasmic space; lipophilic drugs (non-polar drugs) cannot enter and neither can large drug molecules (> 700 Da)

Why are beta-lactams bactericidal?

Because they inhibit cell wall synthesis by stopping the crosslinking

Natural penicillins are referred to as:

Benzylpenicillins

Which receptor is responsible for sympathetic activation?

Beta-1 recpetors

Which class of drugs inhibit sympathetic input to pacing regions of the heart (inhibits phase 4 depolarization)?

Beta-Blockers (Class II)

Which group (class) of drugs are useful for arrhythmias triggered by factors such as exercise or stress and for arrhythmias post Myocardial infarction?

Beta-Blockers (Class II)

Unlike other cephalosporins, Cefepime does not induce _______

Beta-Lactamase production

Cefepime is highly stable against:

Beta-Lactamases

____-____ inhibit transpeptidase by inhibiting cross-linking of peptidoglycan chains

Beta-Lactams

What are examples of Bactericidal drugs?

Beta-lactams, Aminoglycosides

How does the additional species coverage by aminopenicillins change if a Beta-Lactamase inhibitor is present?

Better coverage; gain MSSA, HNPEK, CAPES, Pseudomonas, & bacteroides

All macrolides are excreted through:

Bile

Quinupristin + Dalfopristin is mostly eliminated through ____ excretion

Biliary

Bacteria reproduces through ____ _____

Binary fission

MOA of tetracyclines:

Bind to 30S ribosomal subunit which blocks binding of amino-acyl tRNA to the acceptor site which inhibits protein synthesis

MOA of Lefamulin:

Binds to a unique site in 50s ribosome, blocks both acceptor donor sites, prevents peptidyl transfer

MOA of Amphotericin B:

Binds to ergosterol (human cells have cholesterol) and alters membrane permeability

MOA of Vancomycin

Binds to the D-Ala-D-Ala terminus of NAM-pentapeptide and prevents its transfer by the peptidoglycan synthetase

Nucleoside analogs are called _____

Bioprecursors

Alteplase has a high risk of ____

Bleeding

What is the most common adverse effect with warfarin?

Bleeding

MOA of Xofluza:

Blocks transcription of influenza virus enzyme to stop the spread of infection

Why does warfarin have a delayed onset of action?

Blood factors already exist prior to taking warfarin. It can block synthesis of new blood factors (takes about 3-5 days to start working)

Which Non-Enteric Gram - Rod causes: -Whooping Cough

Bordetella Pertusis

Cefuroxime axetil is the only oral cephalosporin used for _____

Borrelia

Which Spiral bacteria causes: -Lyme disease -Reside in ticks

Borrelia Burgdorferi

When the heart has a slow beat this is known as: (Remember: Tom Brady runs slow)

Bradyarrhythmia

The decreased firing rate of < 60 bpm is known as: (Remember: this is less common to occur)

Bradyarrhythmias

Itraconazole has a ____ spectrum coverage and can cover candida, blastomyces, and histoplasma

Broad

What is the coverage of Sulfonamides (SMX + TMP):

Broad spectrum -MSSA, CA-MRSA (mild cases) -S. pneumoniae (some) -Listeria -M. catarrhalis -H. influenzae (some) -Enterobacteriaceae -Pneumocystis jirovecii (previously P. carinii) (HIV patients)

An antibody which binds to GPIIb-IIIa receptors irreversibly is: A) Prasugrel B) Warfarin C) Abciximab D) Tenecteplase

C) Abciximab

Histamine H2 receptor activation A) Release of inflammatory cytokines B) Release of Histamine C) Activates Adeneyl cyclase D) Release of proton in gastric mucosa

C) Activates Adeneyl cyclase

Which of the following causes GI adverse effects and is contraindicated in pregnancy because it is teratogenic? A) Pyrantel B) Ivermectin C) Albendazole/Mebendazole

C) Albendazole/Mebendazole

Which of the following nematodes inhibits tubulin functions in worms? A) Pyrantel B) Ivermectin C) Albendazole/Mebendazole

C) Albendazole/Mebendazole

This class III agent has a primary effect of blocking K+ channels. It has class I and IV effects and it is a noncompetitive antagonist of alpha and beta receptors A) Lidocaine B) Verapamil C) Amiodarone D) Quinidine

C) Amiodarone

Which of the following is used for abdominal pain? A) Infliximab B) Methotrexate C) Amitriptyline D) Loperamide

C) Amitriptyline

Which of the following has the best oral bioavailability? A) Pen G B) Ampicillin C) Amoxicillin D) Piperacillin

C) Amoxicillin

Which of the following is the drug of choice for most life-threatening systemic infections? A) Rifampin B) Fluconazole C) Amphotericin B D) Ketoconazole

C) Amphotericin B

Cefepime has poor coverage of: A) Gram negative B) Gram positive C) Anaerobes D) Pseudomonas

C) Anaerobes

The major reason why a clinician would hesitate to prescribe chloramphenicol to an adult is: A) Nephrotoxicity B) Hepatotoxicity C) Aplastic anemia D) Gray syndrome

C) Aplastic anemia

Which of the following is a polypeptide polymer with NAM units linked to it? A) Fatty acid synthetases (FAS) 1 and 2 B) Mycolic Acid C) Arabinogalactan D) Pyridoxine

C) Arabinogalactan

Which of the following drugs is a COX-1 Inhibitor? A) Dipyridamole B) Clopidogrel C) Aspirin D) Cilostazol

C) Aspirin

Emtricitabine (FTC) + Efavirenz (EFV) = A) Truvada B) Descovy C) Atripla

C) Atripla

All macrolides inhibit CYP3A4, except: A) Erythromycin B) Clarithromycin C) Azithromycin

C) Azithromycin

Which of the following loses its effectiveness due to mutations in 23s rRNA of the 50s ribosomal subunit? A) Gentamicin B) Tetracycline C) Azithromycin D) Tedizolid

C) Azithromycin

Which of the following macrolides does not inhibit CYP450 3A4? (The rest do) A) Erythromycin B) Clarithromycin C) Azithromycin D) Telithromycin

C) Azithromycin

____ inhibits Phosphatase A) Fosfomycin B) Cycloserine C) Bacitracin

C) Bacitracin

The black boxed warnings increased mortality and QT prolongation belong to which of the following drugs? A) Cycloserine B) Capreomycin C) Bedaquiline D) Pretomanid

C) Bedaquiline

Which of the following is reserved for MDR-TB and should be used 3rd line with 3 or more additional agents? A) Cycloserine B) Capreomycin C) Bedaquiline D) Pretomanid

C) Bedaquiline

What is the major advantage of plazomicin over amikacin? A) Better anaerobic coverage B) Oral bioavailability C) Better CRE/KPC coverage

C) Better CRE/KPC coverage

How is Rifampin primarily eliminated? (key word: primarily) A) Renal B) Hepatically C) Bile D) None of the above

C) Bile

Which of the following is a stimulant laxative? A) Ricinolic acid B) Kaolin C) Bisacodyl D) Anthraqionone glycoside

C) Bisacodyl

Which of the following is a spiral bacteria? A) Legionella B) Yersina pestis C) Borrelia burgdorferi D) Listeria

C) Borrelia burgdorferi

Which of the following increases cCMP? A) Sildenafil B) Isosorbide dinitrate C) Both

C) Both

Terbinafine increases effects of Beta-Blockers because it inhibits which CYP enzyme? A) CYP2C9 B) CYP2C19 C) CYP450 D) CYP3A4

C) CYP450

A non-enteric gram negative rod that is a common cause of food poisoning: A) Pseudomonas aeruginosa B) Bordetella pertussis C) Campylobacter jejunil D) Acinetobacter

C) Campylobacter jejunil

Which of the following does NOT have significant oral bioavailability? A) Cephalexin B) Cefadroxil C) Cefazolin

C) Cefazolin

Which of the following will provide the most reliable coverage of CAPES organisms? A) Cefuroxime B) Ceftriaxone C) Cefepime D) Ceftaroline

C) Cefepime

Which of the following covers MRSA? A) Cefazolin B) Ceftriaxone C) Ceftaroline D) Ceftolozane

C) Ceftaroline

Which 3rd generation drug covers pseudomonas? A) Ceftriaxone B) Cefotaxime C) Ceftazidime D) Cefixime E) Cefdinir F) Cefditoren G) Ceftibuten H) Cefpodoxime

C) Ceftazidime

Which 3rd generation drugs are best for Gram - coverage? (select all that apply) A) Ceftriaxone B) Cefotaxime C) Ceftazidime D) Cefixime E) Cefdinir F) Cefditoren G) Ceftibuten H) Cefpodoxime

C) Ceftazidime D) Cefixime G) Ceftibuten

Biliary sludging in neonates is associated with: A) Cefotaxime B) Doripenem C) Ceftriaxone D) Aztreonam

C) Ceftriaxone

Which of the following leads to vancomycin resistance? A) Mutations in transpeptidase B) Mutations of L-Ala to D-Ala C) Change of terminal D-Ala to A-Lac D) Disrupt cell membrane function

C) Change of terminal D-Ala to A-Lac

Which of the following is used for sensitive strains only? A) Artemisinins B) Mefloquine C) Chloroquine D) Primaquine

C) Chloroquine

Which subclass of class I drugs have a significant Na+ channel block and no change in repolarization of K+ channels? A) Class IA B) Class IB C) Class IC

C) Class IC

Slowing repolarization, prolonging effective refractory period, and suppressing re-entry arrhythmias are benefits of which class of antiarrhythmic drugs? A) Class I B) Class II C) Class III D) Class IV

C) Class III

Which of the following class of drugs are K+ channel blockers? A) Class I B) Class II C) Class III D) Class IV

C) Class III

Slow acting PPI's can interact with which amino acid residue present on the proton pump? A) Asp 98 B) Cys 813 C) Cys 822 D) Tyr 182

C) Cys 822

Which of the following nucleobases are pyrimidines? (select all that apply) A) Adenine B) Guanine C) Cytosine D) Uracil E) Thymine

C) Cytosine D) Uracil E) Thymine

Mycobacterium leprae (leprosy) is treated with which of the following drugs? A) Pretomanid B) Isoniazid C) Dapsone D) Ethambutol

C) Dapsone

If a patient is allergic to sulfa, which of the following should not be used? A) Efavirenz B) Nevirapine C) Delaviridine D) Etravirine

C) Delaviridine

Pen G does not cover: A) Streptococcus pyogenes B) Treponema pallidum C) E. coli D) Actinomyces

C) E. coli

Select all the bugs that are covered by Aztreonam? (select all that apply) A) E. coli (KPC positive) B) Mycoplasma Pneumoniae C) E. coli D) Listeria E) Klebsiella pneumoniae F) Pseudomonas aeruginosa

C) E. coli E) Klebsiella pneumoniae F) Pseudomonas aeruginosa

In addition to expressing highly selective porins what is the other major mechanism by which gram negatives develop multidrug resistance (MDR)? A)Inactivation of drugs B) Influx pumps C) Efflux pumps

C) Efflux pumps

Which of the following Anti-Hep B nucleoside analogues should be given on an empty stomach? A) Telbivudine B) Lamivudine C) Entecavir

C) Entecavir

This type of spectrum extends the spectrum of PCNs A) Broad Spectrum B) Narrow spectrum C) Extended Spectrum

C) Extended Spectrum

Tissue factor pathway inhibitor (TFPI) limits the activation of which of the following factors? A) Factor V B) Factor X C) Factor VII D) Factor II

C) Factor VII

Which of the following is used for the treatment of C. diff colitis? A) Rifamixin B) Rifampin C) Fidaxomicin

C) Fidaxomicin

Which fluoroquinolone does not provide gram negative coverage of Enterics (ESBL +), Pseudomonas, and Neisseria? A) Levofloxacin B) Moxifloxacin C) Gemifloxacin D) Ofloxacin

C) Gemifloxacin

Enterococcus faecalis is a/an: A) Anaerobe B) Gram - cocci C) Gram + cocci D) Gram - rod

C) Gram + cocci

Clindamycin's coverage includes: A) Gram negatives and anaerobes B) Gram negatives and protozoa C) Gram positives, anaerobes, and protozoa D) Gram negatives and gram positives

C) Gram positives, anaerobes, and protozoa

Which of the following drugs target Factor IIa and Xa? A) Xabans B) Fondaparinux C) Heparin

C) Heparin

Which class of antiarrhythmic drugs are K+ channel blockers that prolongs phase 3 repolarization in ventricular muscle fibers? A) IB B) II C) III D) IA E) IC F) IV

C) III

Which of the following provides the best coverage for ESBL positive E. coli strains? A) Ampicillin + Sulbactam B) Piperacillin + Tazobactam C) Imipenem + Cilastatin D) Ceftazidime

C) Imipenem + Cilastatin

Which of the following provides the best coverage of ESBL positive proteus strains? A) Cefazolin B) Piperacillin + Tazobactam C) Imipenem + Cilastatin D) Clindamycin

C) Imipenem + Cilastatin

Activation of M1 and gastrin receptors on parietal cells A) Release of inflammatory cytokines B) Inhibits the Adenyl cyclase C) Increase in intracellular Ca2+ levels D) Release of proton in gastric mucosa

C) Increase in intracellular Ca2+ levels

What is the significance of the carboxy group in ticarcillin? A) Protects the drug from degradation by stomach acids B) Increases the stability against beta-lactamases C) Increases penetration into pseudomonas porins D) Increases solubility and oral absorption

C) Increases penetration into pseudomonas porins

For a drug that has time-dependent killing effect, which of the following strategies increases its effectiveness? A) Increase the dose B) Decrease dosing frequency C) Increasing dosing frequency

C) Increasing dosing frequency

Which of the following are covered by ampicillin only (Without sulbactam)? (select all that apply) A) Neisseria gonorrhoeae B) Bacteroides fragilis C) Listeria D) Neisseria meningitides E) Pseudomonas F) MSSA G) MRSA H) Haemophilus Influenzae I) Streptococcus pyogenes

C) Listeria D) Neisseria meningitides H) Haemophilus Influenzae I) Streptococcus pyogenes

Which of the following is a chloride channel agonists? A) Loperamide B) Hyoscine C) Lubiprostone D) Mebeverine

C) Lubiprostone

Which of the following is the major gain of coverage of Nafcillin over Pen G? A) MRSA B) Bacteroides C) MSSA D) Pseudomonas

C) MSSA

Neurologic & psychiatric toxicities is a black boxed warning of which drug? A) Chloroquine B) Primaquine C) Mefloquine D) Quinine

C) Mefloquine

This is defined as a resistance to both Isoniazid and Rifampin and having very high mortality A) DDRP B) Nicotinamidase C) Multi-drug resistant TB (MDR-TB) D) Extensively-drug resistant TB (XDR-TB)

C) Multi-drug resistant TB (MDR-TB)

Which of the following is an aerobic, nonmotile, capsular bacteria that has a thick, waxy, hydrophobic, mycolic acid containing cell (in addition to peptidoglycan/polysaccharide) wall that is highly resilient? A) Corynebacterineae B) Acid-Fast C) Mycobacteria D) Mycolic Acid

C) Mycobacteria

Atypical bacteria that lacks the cell wall A) Treponema Pallidum B) Haemophilus influenzae C) Mycoplasma pneumoniae D) streptococcus pyogenes E) Clostridium Difficile

C) Mycoplasma pneumoniae

Dasabuvir is a: A) NS3/4A inhibitor B) NS5A Inhibitor C) NS5B Inhibitor

C) NS5B Inhibitor

Which of the following is currently least sensitive to Pen G? A) Treponema pallidum B) Neisseria meningitides C) Neisseria gonorrhoeae

C) Neisseria gonorrhoeae

Which of the following H2 antagonists has maximum oral bioavailability? A) Cimetidine B) Ranitidine C) Nizatidine D) Famotidine

C) Nizatidine

Which of the following events might occur following the inactivation of H2 receptors on the parietal cells? A) Activation of Gs protein B) Activation of Gi Protein C) No activation of Gs Protein D) No activation of Gi Protein

C) No activation of Gs Protein

Which of the following can be used for Vancomycin resistant strains? A) Telavancin B) Dalbavancin C) Oritavancin

C) Oritavancin

Which aspect of cell wall synthesis is inhibited by Aztreonam? A) Synthesis of precursors B) Elongation and transfer C) Peptide crosslinking

C) Peptide crosslinking

Which of the following is used for inflammation? A) Infliximab B) Milk of magnesia C) Prednisolone D) Loperamide

C) Prednisolone

Which of the following drugs destroys gametocytes in plasma? A) Chloroquine B) Artemisinins C) Primaquine D) Quinine

C) Primaquine

Flucytosine is a: A) Bioprecurser B) Active Metabolite C) Prodrug

C) Prodrug

Isavuconazonium is a: A) Bioprecurser B) Active Metabolite C) Prodrug

C) Prodrug

Which of the following is not a benefit of normal flora? A) Produce vitamin B12 B) Produce vitamin K C) Produce insulin D) Digest fiber

C) Produce insulin

A less common but more difficult to treat UTI is caused by this indole positive gram negative rod A) E. coli B) Staphylococcus saprophyticus C) Proteus vulgaris D) Proteus mirabilis

C) Proteus vulgaris

In CAPES, "P" refers to: A) Proteus B) Pseudomonas C) Providencia D) Peptostreptococcus

C) Providencia

The major advantages of Ceftazidime over Ceftriaxone is that Ceftazidime has an excellent coverage of: A) Resistant Streptococci B) Neisseria gonorrhoeae C) Pseudomonas

C) Pseudomonas

Which of the following species presents a therapeutic challenge because of its ability to express highly effective efflux pumps that act on multiple drugs? A) Staphylococcus Aureus B) E. Coli C) Pseudomonas Aeruginosa D) MRSA

C) Pseudomonas Aeruginosa

Which of the following causes inhibition of fatty acid synthase (FAS1), disturbance of intracellular pH/electrolytes, and disruption of membrane transport? A) Rifampin (RMP) B) Isoniazid (INH) C) Pyrazinamide (PZA) D) Ethambutol (EMB)

C) Pyrazinamide (PZA)

Cinchonism is a serious adverse effect of which drug from class IA? A) Disopyramide B) Flecainide C) Quinidine D) Procainamide

C) Quinidine

Which of the following is a prototypical class IA drug that is used to treat many types of arrhythmias? A) Disopyramide B) Flecainide C) Quinidine D) Procainamide

C) Quinidine

Which of the following beta-lactamase inhibitors is NOT a beta-lactam? A) Tazobactam B) Clavulanic acid C) Relebactam D) Sulbactam

C) Relebactam

Which of the following beta-lactamase inhibitors is/are NOT beta-lactams? (select all that apply) A) Tazobactam B) Clavulanic Acid C) Relebactam D) Vaborbactam E) Avibactam

C) Relebactam D) Vaborbactam E) Avibactam

Which of the following are 3rd gen Thrombolytics? (select all that apply) A) Anistreplase B) Alteplase C) Reteplase D) Urokinase E) Prourokinase F) Tenecteplase

C) Reteplase F) Tenecteplase

Which of the following drugs is a nucleotide analogue? A) Emtricitabine B) IFN-alpha C) Sofosbuvir D) Ribavirin

C) Sofosbuvir

Gram positive cocci that frequently causes UTI's A) E. coli B) Neisseria gonorrhoeae C) Staphyloccus Saprophyticus D) Moraxella Catarrhalis

C) Staphyloccus Saprophyticus

Gram positive cocci that is most commonly involved in skin and skin structure infections is: A) Staphylococcus Agalactiae B) Streptococcus pneumoniea C) Staphylococcus Aureus D) Enterobacter

C) Staphylococcus Aureus

Which of the following is most sensitive to Pen G? A) Staphylococcus aureus B) Streptococcus Pneumoniae C) Streptococcus Pyogenes D) Enterococcus Faecium

C) Streptococcus Pyogenes

Which of the following is most likely to cause crystalluria? A) Clindamycin B) Ciprofloxacin C) Sulfamethoxazole D) Doxycycline

C) Sulfamethoxazole

Which of the following is most likely to cause crystalluria? A) Clindamycin B) Ciprofloxacin C) Sulfamethoxazole D) Doxycycline

C) Sulfamethoxazole

Which of the following describes the function of Bactoprenol? A) Catalyzes NAG-NAM complex by taking it outside and flipping it over to the cell wall outside of the cell wall membrane B) Removes phosphate and recycles bactoprenol C) Takes NAG-NAM complex outside of the inner layer

C) Takes NAG-NAM complex outside of the inner layer

Which of the following Anti-Hep B medications are Nucleoside analogues? (select all that apply) A) Adefovir B) Tenofovir C) Telbivudine D) Lamivudine

C) Telbivudine D) Lamivudine

Which of the following is a prodrug? A) Didanosine (DDI) B) Abacavir (ABC) C) Tenofovir disoproxil (TDF)

C) Tenofovir disoproxil (TDF)

Which of the following drugs are ADP (P2Y) receptor antagonists? (select all that apply) A) Abciximab B) Eptifibatide C) Ticagrelor D) Vorapaxar E) Cangrelor

C) Ticagrelor E) Cangrelor

What is Sofosbuvir used for? A) Treatment of Chronic Hep A B) Treatment of Chronic Hep B C) Treatment of Chronic Hep C

C) Treatment of Chronic Hep C

Lamivudine (3TC) + Zidovudine (AZT) + Abacavir (ABC) = A) Comvibir B) Epzicom C) Trizivir

C) Trizivir

In urea breath test, H. pylori produces ____ which hydrolyses urea to carbon dioxide and ammonia A) Hydrolase B) Hydroxylase C) Urease D) Dehydrogenase

C) Urease

_____ Prevents transfer of D-Ala-D-Ala peptide to the outside A) Clindamycin B) Augmentin C) Vancomycin D) Keflex

C) Vancomycin

Which of the following is an antidote (reversal agent) for warfarin? A) Idarucizumab B) Andexanet alfa C) Vitamin K D) Protamine E) Antifibrinolytic drugs

C) Vitamin K

Apixaban is an inhibitor of factor ____ A) IIa B) X C) Xa D) Thrombin

C) Xa

What is Fidaxomicin used to treat?

C. diff colitis

What are the nucleosides of DNA?

C: Cytosine G: Guanine A: Adenine T: Thymine

What are the nucleosides of RNA?

C: Cytosine G: Guanine A: Adenine U: Uracil

Three viruses that foscarnet treats

CMV, HSV-1 and HSV-2

Foscarnet is used for:

CMV, HSV-1, HSV-2

Foscarnet Tx

CMV, HSV1 and HSV2

Cefuroxime is the only 2nd generation drug with significant ___ penetration

CNS

What is a major concern of all carbapenems, particularly in patients with renal insufficiency or other underlying causes that predispose them to this adverse effect?

CNS toxicity (Seizures)

Aspirin inhibits ____ irreversibly. This allows for its effect to last for a longer time which is why patients take 81 mg once daily.

COX-1

This CYP enzyme is required for Clopidogrel to activate?

CYP2C19

Which CYP enzyme is responsible for clopidogrel activation?

CYP2C19

The S-isomer of warfarin is metabolized by which CYP enzyme?

CYP2C9

Which enzyme does Fluconazole mainly inhibit which is the reasoning behind why it interacts and increases Warfarin levels?

CYP2C9

Itraconazole is metabolized by which CYP enzyme?

CYP3A4

Pretomanid interacts with substrates of which CYP enzyme?

CYP3A4

Ritonavir is a strong ____ inhibitor

CYP3A4

Quinupristin + Dalfopristin inhibits which CYP enzyme?

CYP3A4 (it interacts with anything that inhibits this enzyme as well)

Itraconazole has drug interactions due to its:

CYP3A4 inhibition

Neuromuscular blockade caused by aminoglycosides is reversible by:

Calcium

Which class of drugs block open (or inactivated ) calcium channels to slow conduction in tissues dependent on Ca2+ current on SA and AV nodes?

Calcium Channel Blockers (CCB's) (Class IV)

Which Non-Enteric Gram - Rod causes: -Food poisoning -GI infections -Animal GI tract

Campylobacter Jejuni

This contains a "slime layer" made up of polysaccharides

Capsule

If a patient has Colistin resistant gene (MCR-1) what can we use to treat?

Carbapenem

What does CRE mean?

Carbapenem Resistant Enterobacteriaceae (Includes both KPC and MBL)

Why should a nitrate not be used with sildenafil?

Causes refractory hypotension

How does 23S rRNA methylation impact macrolides, clindamycin, and chloramphenicol?

Causes resistance

What drug is designed so that it will bind to the PBPs despite the changes in MRSA and S. Pneumonia?

Ceftaroline

What is avibactam combined with?

Ceftazidime

Why do some people have penicillin allergy?

Cellular proteins become Haptens which are covalently modified proteins that become immunogenic

How is the core ring structure of cephalosporins different from penicillins?

Cephalosporins have 2 R groups (Penicillins have 1) and a 6 membered ring (Penicillins have 5)

Which Atypical bacteria causes: -Community acquired pneumonia

Chlamydia pneumoniae

Which Atypical bacteria causes: -Genitourinary infections -Conjunctivitis in neonate

Chlamydia trachomatis

What is the risk associated with long-term use of Nitrofurantoin?

Chronic pulmonary fibrosis (in patients with long-term use and this is irreversible; it is a rare case because it usually is not used for long-term)

This drug is used to treat Acyclovir-resistant HSV and Ganciclovir-resistant HSV:

Cidofovir

What is the reservoir form of cidofovir that is responsible for its longer intracellular half-life?

Cidofovir-Phosphocholine

Which Enteric Gram - Rod causes: -UTI -Neonatal meningitis -Septicemia (All rare and from hospitals)

Citrobacter

Place the macrolides in order from best absorbed to least absorbed:

Clarithromycin & Telithromycin > Azithromycin > Erythromycin

Which subclass of Class I has a relatively rapid rate (Mild) of dissociation from Na+ channels and has a shorter (activated) rate of repolarization?

Class IB

Which subclass of Class I has a slow rate of dissociation from Na+ channels and no effect on repolarization?

Class IC

Macrolides bind to 50S and competes with which 2 other drugs?

Clindamycin and Chloramphenicol

Which Gram + Rod Anaerobe causes: -Neurotoxin; botulinum

Clostridium Botulinum

Which Gram + Rod Anaerobe causes: -C. diff colitis

Clostridium Difficile

Which Gram + Rod Anaerobe causes: -Production of Neurotoxin; Tetanus

Clostridium Tetani

Which Gram + Rod Anaerobe causes: -Food poisoning; gas gangrene

Clostridium perfringens

What bacteria has a "sphere" appearance?

Coccus

If a patient has carbapenem resistant gene (CRE) what can we use to treat?

Colistin

What is CA-MRSA?

Community-acquired MRSA

Aminoglycosides have ____ dependent killing effect

Concentration

Why is Nitrofurantoin not useful for other infections?

Concentration in the serum is not high enough to be useful for other infections

These are cells that control the rhythm of the heart

Contractile cells

What is the function of tissue plasminogen activator (T-PA)?

Converts plasminogen to plasmin

This is caused by the secretion of amiodarone by the lacrimal gland, with accumulation on the corneal surface

Corneal Microdeposits

Which Gram + Rod causes: -Diphtheria (toxin)

Corynebacterium diphtheriae

MOA of Aspirin:

Covalently (irreversibly) binds to active site of COX-1 and COX-2

Entecavir is eliminated renally, therefore dosing is based on ___

CrCl

This type of juice carries polyphenols in the fruit the juice is made from. It disrupts attachment of fimbriae on mucosal membrane and helps with UTI's

Cranberry juice

Daptomycin is a ____ lipopeptide

Cyclic

Cytosine + sugar = ?

Cytidine

Cidofovir is a derivative of:

Cytosine

Which of the following represents Senna? A) Ricinolic acid B) Kaolin C) Bisacodyl D) Anthraqionone glycoside

D) Anthraqionone glycoside

Gentamicin coverage most closely resembles that of: A) Ceftriaxone B) Nafcillin C) Meropenem D) Aztreonam

D) Aztreonam

Generally speaking, drugs that inhibit bacterial cell wall synthesis tend to be: A) Ineffective against gram negatives B) Ineffective against gram positives C) Bacteriostatic D) Bactericidal

D) Bactericidal

Clopidogrel is activated by which CYP enzyme? A) CYP2D6 B) CYP3A4 C) CYP1A2 D) CYP2C19

D) CYP2C19

Which of the following ADP (P2Y) receptor antagonists is administered as an IV only? A) Ticlopidine B) Ticagrelor C) Clopidogrel D) Cangrelor E) Prasugrel

D) Cangrelor

Which of the following has the best gram - coverage? A) Cefaclor B) Ceohalexin C) Cefuroxime D) Cefoxitin

D) Cefoxitin

Mutations in the 23s rRNA of 50s ribosomal subunit result in cross-resistance between macrolides and: A) Tetracyclines B) Fluoroquinolones C) Linezolid D) Clindamycin

D) Clindamycin

Which of the following is an inappropriate drug for the treatment of pneumonia as it is inactivated by lung surfactants? A) Meropenem B) Dalbavancin C) Ciprofloxacin D) Daptomycin

D) Daptomycin

Which of the following is the MOA of daptomycin? A) Inhibit cell wall synthesis B) Inhibit protein synthesis C) Disrupt RNA polymerase function D) Disrupt cell membrane function

D) Disrupt cell membrane function

Which of the following class III agents is reserved for patients with highly symptomatic atrial fibrillation/flutter? A) Amiodarone B) Sotalol C) Dorendarone D) Dofetilide

D) Dofetilide

Which of the following NNRTIs is considered to be a 3rd generation? A) Efavirenz B) Nevirapine C) Delaviridine D) Doravirine

D) Doravirine

The goal of this class III agent was to design a medication with similar efficacy to amiodarone but with fewer adverse reactions. By doing so, its lipophilicity and t1/2 is decreased and its iodines have been removed which removes side effects to the thyroid. Which drug is this? A) Dofetilide B) Sotalol C) Ibutilide D) Dronedarone

D) Dronedarone

Which of the following is not susceptible to vancomycin? A) Staphylococcus aureus B) Enterococcus faecalis C) Streptococcus pyogenes D) E. Coli

D) E. Coli

Class A are what kind of Beta-Lactamases? A) Penicillinases B) Cephalosporins D) ESBL, KPC E) NDM-1

D) ESBL, KPC

Which of the following drug is available in Stribild? A) Raltegravir B) Nevirapine C) Rilpivirine D) Elvitegravir E) Abacavir

D) Elvitegravir

Which Carbapenem has the longest half-life of all the carbapenems? A) Imipenem B) Meropenem C) Doripenem D) Ertapenem

D) Ertapenem

Which of the carbapenems does NOT cover pseudomonas? A) Imipenem B) Meropenem C) Doripenem D) Ertapenem

D) Ertapenem

Which of the following carbapenems does NOT cover Pseudomonas? A) Imipenem + Cilastatin B) Meropenem C) Doripenem D) Ertapenem

D) Ertapenem

Which of the following lacks the coverage of pseudomonas? A) Imipenem B) Meropenem C) Doripenem D) Ertapenem

D) Ertapenem

Which of the following causes inhibition of arabinosyl transferase III and disrupts synthesis of arabinogalactan? A) Rifampin (RMP) B) Isoniazid (INH) C) Pyrazinamide (PZA) D) Ethambutol (EMB)

D) Ethambutol (EMB)

This is defined as resistance to fluoroquinolones + (Kanamycin, Capreomycin, or Amikacin) and having even higher mortality rates A) DDRP B) Nicotinamidase C) Multi-drug resistant TB (MDR-TB) D) Extensively-drug resistant TB (XDR-TB)

D) Extensively-drug resistant TB (XDR-TB)

Which of the following H2 antagonists has least oral bioavailability? A) Cimetidine B) Ranitidine C) Nizatidine D) Famotidine

D) Famotidine

Which of the following is a Non-Nucleoside analog DNA polymerase inhibitor? A) Acyclovir B) Valacyclovir C) Ganciclovir D) Foscarnet

D) Foscarnet

Protease inhibitors are important group of drugs targeting ____ A) HIV and HBV B) HIV and HSV C) HBV and HCV D) HIV and HCV

D) HIV and HCV

MRSA strains developed because these strains: A) Evolved efflux pumps that exported all beta-lactams B) Developed porins that excluded all beta-lactams C) Started producing enzymes that broke down all beta-lactams D) Have PBPs that have lost most of the affinity for beta-lactams

D) Have PBPs that have lost most of the affinity for beta-lactams

Which amino acid on CYP450 competes with Cimetidine for binding with Fe2+ of cytochrome heme? A) Aspartate B) Valine C) Isoleucine D) Histidine

D) Histidine

Which class of antiarrhythmic drugs are Na+ channel blockers that slow phase 0 depolarization in ventricular muscle fibers? A) IB B) II C) III D) IA E) IC F) IV

D) IA

Which of the following class III agents have a high risk of QT prolongation and Tdp? A) Dronedarone B) Sotalol C) Amiodarone D) Ibutilide

D) Ibutilide

Isavuconazole is used for treatment of invasive aspergillosis and mucomycosis particularly in which group of patients? A) Diabetic Patients B) Hypertensive Patients C) Hypotensive Patients D) Immunocompromised Patients

D) Immunocompromised Patients

An E. Coli strain positive for which of the following is resistant to most antibiotics? A) ESBL B) Amp C C) Penicillinase D) KPC

D) KPC

Which of the following is an enteric gram negative rod? A) Staphylococcus aureus B) Treponema pallidum C) Pseudomonas aeruginosa D) Klebsiella pneumoniae

D) Klebsiella pneumoniae

Which of the following are 3rd generation fluoroquinolones? (select all that apply) A) Nalidixic acid B) Ciprofloxacin C) Ofloxacin D) Levofloxacin E) Moxifloxacin F) Gemifloxacin G) Delafloxacin H) Gatifloxacin

D) Levofloxacin E) Moxifloxacin F) Gemifloxacin

Which of the following is used for diarrhea? A) Infliximab B) Milk of magnesia C) Prednisolone D) Loperamide

D) Loperamide

A patient has been taking atorvastatin 20 mg for the past 2 years and begins taking rifampin. Which of the following is correct? A) May need to increase dose of rifampin due to inhibition by atorvastatin B) May need to increase dose of atorvastatin due to inhibition by rifampin C) May need to increase dose of rifampin due to induction by atorvastatin D) May need to increase dose of atorvastatin due to induction by rifampin

D) May need to increase dose of atorvastatin due to induction by rifampin

Which of the following is used as a GI smooth muscle relaxant (reserpine derivative)? A) Loperamide B) Hyoscine C) Lubiprostone D) Mebeverine

D) Mebeverine

Which of the following has the broadest spectrum of coverage? A) Pen G B) Oxacillin C) Aztreonam D) Meropenem

D) Meropenem

A gram negative cocci that often causes otitis media in kids A) E. coli B) Neisseria gonorrhoeae C) Staphyloccus Saprophyticus D) Moraxella Catarrhalis

D) Moraxella Catarrhalis

Which of the following is a large, branched alkyl chain synthesized by fatty acid synthetases 1 and 2? A) Corynebacterineae B) Acid-Fast C) Mycobacteria D) Mycolic Acid

D) Mycolic Acid

Which of the following is susceptible to Pen G? A) E. Coli B) Proteus mirabilis C) Moraxella D) Neisseria meningitides

D) Neisseria meningitides

Ranolazine: A) Decreases heart rate B) Decreases blood pressure C) Increases coronary vasodilation D) None of the above

D) None of the above

Which of the following prostanoids inhibits platelet aggregation? A) PGE2 B) PGD2 C) PGF2alpha D) PGI2

D) PGI2

Glycoprotein GPIIa/IIIb is located on the cell membrane of the: A) Erythrocyte B) Endothelial cells C) Macrophages D) Platelets

D) Platelets

Which of the following inhibits mycolic acid synthesis and is approved in cases of XDR? A) Cycloserine B) Capreomycin C) Bedaquiline D) Pretomanid

D) Pretomanid

Which of the following can be used to prevent relapse. It is not effective against erythrocytic stage and is used in combination with other drugs A) Artemisinins B) Mefloquine C) Chloroquine D) Primaquine

D) Primaquine

Chronic use of which class IA agent can cause lupus-like syndrome? A) Disopyramide B) Flecainide C) Quinidine D) Procainamide

D) Procainamide

Which of the following is an antidote (reversal agent) for Heparin? A) Idarucizumab B) Andexanet alfa C) Vitamin K D) Protamine E) Antifibrinolytic drugs

D) Protamine

Which of the following contains endotoxin? A) Streptococcus pneumoniae B) Enterococcus faecalis C) Mycoplasma pneumonia D) Proteus mirabilis

D) Proteus mirabilis

Which of the following is a Serotonin 5HT4 Receptor Agonist? A) Magnesium hydroxide B) Kaolin C) Bisacodyl D) Prucalopride

D) Prucalopride

Which of the following is another name for vitamin B6? A) Fatty acid synthetases (FAS) 1 and 2 B) Mycolic Acid C) Arabinogalactan D) Pyridoxine

D) Pyridoxine

Activation of Protein Kinase A) Release of inflammatory cytokines B) Inhibits the Adenyl cyclase C) Increase in intracellular Ca2+ levels D) Release of proton in gastric mucosa

D) Release of proton in gastric mucosa

What is the name and shape of the bacteria responsible for Streptococcus pneumoniae? A) Streptobacillus (Chain) B) Bacillus (Rod shaped) C) Coccus (Sphere) D) Streptococcus (Chain) E) Staphylococcus (Clusters)

D) Streptococcus (Chain)

All macrolides compete with Clindamycin and Chloramphenicol except which one of the following? A) Erythromycin B) Clarithromycin C) Azithromycin D) Telithromycin

D) Telithromycin

Which of the following macrolides are the most potent when inhibiting CYP450 3A4? A) Erythromycin B) Clarithromycin C) Azithromycin D) Telithromycin

D) Telithromycin

What is PAE? A) Time it takes for bacteria to become resistant after initiation of treatment B) Time it takes for antibiotic toxicity to occur C) Time it takes for an antibiotic to work D) Time required for bacteria to resume growth after antibiotic concentration is below MIC

D) Time required for bacteria to resume growth after antibiotic concentration is below MIC

What is PAE? A) Time it takes for bacteria to become resistant after initiation of treatment B) Time it takes for antibiotic toxicity to occur C) Time it takes for an antibiotic to work D) Time required for bacteria to resume growth after antibiotic concentration is below MIC

D) Time required for bacteria to resume growth after antibiotic concentration is below MIC

Which of the following protease inhibitors has a black boxed warning for Fatal Hepatotoxicity? A) Fosamprenavir B) Delaviridine C) Amprenavir D) Tipranavir

D) Tipranavir

Which of the following drugs are Thrombin Receptor (PAR-1) antagonists? A) Abciximab B) Eptifibatide C) Tirofiban D) Vorapaxar E) Defibrotide

D) Vorapaxar

Which of the following drugs have azido functional group? A) Lamivudine B) Didanosine C) Tenofovir D) Zidovudine E) Emtricitabine

D) Zidovudine

Group A beta-hemolytic streptococcus that causes "strep" A) Treponema Pallidum B) Haemophilus influenzae C) Mycoplasma pneumoniae D) streptococcus pyogenes E) Clostridium Difficile

D) streptococcus pyogenes

Warfarin has a delayed onset of action due to ______? A) delayed absorption B) longer half life C) requirement of activation of enzyme D) the presence of already activated coagulation factors

D) the presence of already activated coagulation factors

What does the Beta-Lactam ring resemble structurally?

D-Ala-D-Ala

What is the abbreviation for Didanosine?

DDI

Isonicotinoyl-NADP is the active form of Isoniazid that is a potent ___ inhibitor

DHFR inhibitor

Adenovirus is what kind of virus?

DNA

Herpes Virus is what kind of virus?

DNA

The pharmacophore of fluoroquinolones carboxypyridinone, binds to the ___ binding site of the enzymes

DNA

Rifampin is metabolized by:

Deacetylation, hydrolysis

What does the highly reactive carbonyl group on the Beta-Lactam do?

Decrease Stability

All LMWH's are approved for use in the prevention and treatment of:

Deep vein thrombosis (DVT)

Tetracycline only, undergoes a _____ reaction that results in the production of toxic anhydrotetracycline

Dehydration

What is one pharmacological property of warfarin that distinguishes it from all other anticouagulants?

Delayed action

What drug is used as an add-on for the treatment of Covid-19?

Dexamethasone

What is the pharmacophore of aminoglycosides that is linked to other sugars?

Diaminoinositol

What is the primary source of vitamin K (Phytonadione)?

Diet (leafy green vegetables)

MOA of Polymyxins

Disrupt outer membrane which produces a detergent-like effect

One attachment and entry inhibitor for HSV

Docosanol

These pumps keep nutrients inside and pump drugs out to prevent accumulation of drug which can cause resistance

Drug efflux pumps (Some pumps can be multi-drug resistant)

What is the most common cause of acquired prolonged QT interval?

Drug induced

Why was Methicillin removed from the market?

Due to it causing Nephrotoxicity

Why does vancomycin only have coverage for gram + bacteria?

Due to it having a size of 1400 Da. It is too large to go through porins (max 700 Da) meaning it cannot penetrate gram - bacteria

Why is Amiodarone the safest drug of class III?

Due to its affect on the heart tissue and not having reverse-use dependency

Why were Penicillinase-Resistant Penicillins (PRPs) introduced?

Due to the beta-lactamase prototype "Penicillinase" introduced in 1940 and was found to be the cause of resistance to penicillin

Doxycycline does NOT cover: A) MSSA B) CA-MRSA C) Rickettsiae D) Plasmodium falcipuram E) Acinetobacter

E) Acinetobacter

Doxycycline does NOT cover: A) MSSA B) CA-MRSA C) Rickettsiae D) Plasmodium falcipuram E) Acinetobacter

E) Acinetobacter

Which of the following is an antidote (reversal agent) for Aminocaproic acid and tranexamic acid Aprotinin? A) Idarucizumab B) Andexanet alfa C) Vitamin K D) Protamine E) Antifibrinolytic drugs

E) Antifibrinolytic drugs

An anaerobe that causes pseudomembranous colitis A) Treponema Pallidum B) Haemophilus influenzae C) Mycoplasma pneumoniae D) streptococcus pyogenes E) Clostridium Difficile

E) Clostridium Difficile

Which class of antiarrhythmic drugs are Na+ channel blockers that markedly slows phase 0 depolarization in ventricular muscle fibers? A) IB B) II C) III D) IA E) IC F) IV

E) IC

Class B are what kind of Beta-Lactamases? A) Penicillinases B) Cephalosporins D) ESBL, KPC E) NDM-1

E) NDM-1

What is the name and shape of the bacteria responsible for Staphylococcus aureus? A) Streptobacillus (Chain) B) Bacillus (Rod shaped) C) Coccus (Sphere) D) Streptococcus (Chain) E) Staphylococcus (Clusters)

E) Staphylococcus (Clusters)

Which Enteric Gram - Rod causes: -GI -UTI

E. coli

Which coverage do Carbapenems cover in terms of resistant strains?

ESBL

Not Beta-Lactams are ONLY effective against:

ESBL and KPC (Reserved for serious infection with resistant strains: Not MBL)

This occurs during Phase 2 or 3 often due to prolonged repolarization times (drugs, hypokalemia, genetics)

Early Afterdepolarization

Five anti HIV non-nucleoside reverse transcriptase inhibitors (4c)

Efavirenz, nevirapine, delaviridine, etravirine, rilpiravine

Tendinopathy is considered to be a black boxed warning of Fluoroquinolones, it is most common to occur in which type of patients?

Elderly patients who are on corticosteroids

Record of the electrical activity of the heart

Electrocardiogram (EKG or ECG)

Two attachment and entry inhibitors for HIV

Enfuvirutide, maraviroc

3 anti HIV Drugs that inhibit viral attachment and entry

Enfuvirutide, maraviroc, docosonol

Which Enteric Gram - Rod causes: -UTI -Endocarditis -SSTI's

Enterobacter

Which Gram + Cocci causes: -Endocarditis -UTI -Meningitis -Septicemia

Enterococcus Faecalis

Which Gram + Cocci causes: -Rare, more resistant infections -Similar infections as E. Faecalis

Enterococcus Faecium

Which enzyme is stimulated by nitric oxide? Which molecule is produced?

Enzyme: Guanylyl Cyclase Molecule: cGMP

The dimethylamino group on ring A of Tetracyclines undergoes ____ resulting in loss of 50% of activity

Epimerization

The enzyme 14-a-demethylase converts lanosterol into ____

Ergosterol

Macrolide's MOA is helpful against bacteria by producing _____

Exotoxin

Describe the concept of selective toxicity.

Exploit biochemical differences between the pathogen and the host to selectively target the microorganism to minimize adverse effects

Idoxuridine and Trifluridine are available as a topical route for ____

Eyes

Which class of antiarrhythmic drugs are Ca2+ channel blockers that inhibits action potential in SA and AV nodes? A) IB B) II C) III D) IA E) IC F) IV

F) IV

Conjugative plasmids can transfer information from __ to __

F+ to F-

What is the abbreviation for Emtricitabine?

FTC

Biosynthesis of which factors depend on adequate supply of vitamin K?

Factors II (2), VII (7), IX (9), X (10)

T/F: 2nd generation Cephamycins are administered orally

False

T/F: All bacteria contain LPS

False

T/F: All macrolides cause stomach cramps because they all convert to cyclic ketals

False

T/F: Amphotericin B needs to be adjusted for renal and hepatic impairment

False

T/F: Bacteria contain mitochondria

False

T/F: Bacterial ribosomes contain 40s and 60s subunits

False

T/F: Both capsulated and non-capsulated strains can cause meningitis

False

T/F: Cangrelor has the longest duration of action of all ADP antagonists

False

T/F: Carbapenem is available in oral form

False

T/F: Clopidogrel is not considered a prodrug

False

T/F: Doxycycline and Minocycline absorption is affected by food

False

T/F: Erythromycin should be taken with food

False

T/F: First Phosphorylation step is required for Sofosbuvir

False

T/F: Fluconazole does not penetrate the CSF

False

T/F: Fluconazole is effective against Aspergillus

False

T/F: Gram - bacteria have thicker cell walls

False

T/F: Griseofulvin is recommended in pregnancy

False

T/F: Hib vaccine protects against both capsulated and non-capsulated strains

False

T/F: Hib vaccine protects kids from sinusitis and otitis media caused by H. influenzae

False

T/F: KPC and ESBL are covered by aminoglycosides

False

T/F: LPS is considered to be an exotoxin

False

T/F: LPS is present on Gram + bacteria

False

T/F: Mitotic structures are present but hard to see during bacterial cell division

False

T/F: Nizatidine has the lowest oral bioavailability (among all H2 receptors)

False

T/F: PPI's binds to the proton pump in a reversible manner

False

T/F: The enteric coating of PPI's gets dissociated in the acidic pH of the stomach

False

T/F: Vancomycin has coverage over gram negative bacteria

False

T/F: Warfarin directly inhibits thrombin for its anticoagulant effect.

False

T/F: Glycoprotein IIb-IIIa antagonists are all given orally

False, all given IV

T/F: Antiarrhytmic drugs cannot cause arrhythmia because they treat this condition

False, any drug that treats arrhythmia has a side effect of causing it as well

T/F: Warfarin has a faster effect when compared to heparin

False, heparin has a faster effect while warfarin is slower

T/F: Erythromycin is converted into cyclic ketals. Other macrolides convert into cyclic ketals as well

False, only erythromycin converts into cyclic ketals which causes stomach cramping. The others do not.

T/F: Amantadine and Rimantadine are acidic drugs

False, they are basic

T/F: Due to its narrow spectrum, Sarecycline has more impact on the gut flora

False; less impact on the gut flora

T/F: Traditional Beta-lactamase inhibitors such as Clavulanic Acid, Sulbactam, and Tazobactam can be used for ESBL and KPC

False; only non Beta-Lactam can be used

What is the prodrug of Penciclovir?

Famciclovir

What are infusion related Adverse effects of Amphotericin B?

Fever, chills, vomiting, hypotension, thrombophlebitis

Once t-PA converts plasminogen to plasmin, this plasmin then converts fibrin to:

Fibrin degradation products

What is the narrow drug choices that we can use for strains that produce "hardy" beta-lactamases? (Go in order)

First try: Penicillinases Then (If first one does not work): Cephalosporinases Then: Extended Spectrum Beta-Lactamases (ESBL) Then: Carbapenem-Resistant Enterobacteriaceae (CRE)

The addition of what group in quinolones dramatically improved tissue distribution of similar drugs of Nalidixic acid?

Fluorine

One non-nucleoside analog DNA Polymerase inhibitor (4b)

Foscarnet

What is the only drug that is considered a non-nucleoside analog and DNA polymerase inhibitor?

Foscarnet

Rapid reversal of warfarin can be achieved by transfusion with:

Fresh or frozen plasma

Griseofulvin inhibits:

Fungal Mitosis

Which of the following is a 4th generation fluoroquinolone? A) Nalidixic acid B) Ciprofloxacin C) Ofloxacin D) Levofloxacin E) Moxifloxacin F) Gemifloxacin G) Delafloxacin H) Gatifloxacin

G) Delafloxacin

Adverse effects of Chloroquine:

GI, headache, rash, eyes, hair

Three anti-HSV Nucleoside/nucleotide analogs that are also used for CMV (4a)

Ganiciclovir, Valganiciclovir, Cidofovir

Contractile cells communicate through:

Gap junctions

This is the time it takes for a population of bacterial cells to double

Generation time

Which generations of Fluoroquinolones is best to use for Gram - bacteria?

Generations 1, 2, and 3

What are R Determinants?

Genes that contain resistance

This aminoglycoside is cheap, used the most, and is similar to Tobramycin

Gentamicin

What is synergism?

Giving 2 drugs that are clinically significant, greater than additive effects (Ex: Beta-Lactam + Aminoglycoside (Enterococci))

What are additive effects?

Giving 2 drugs that are not clinically significant (Ex: Antibiotic + Antifungal Effects)

How is Flucytosine eliminated?

Glomerular filteration

Fostemsavir inhibits ____

Gp120

Enfuvirtide is an inhibitor that mimics ____ so that the virus cannot multiply

Gp41

Peptostreptococcus spp.

Gram + Cocci Anaerobe

What is the major coverage of PRPs?

Gram + Cocci bacteria: -Staphylococcus MSSA -Streptococcus (Skin and soft tissue infections, osteomyelitis)

Bacillus Anthracis

Gram + Rod

Corynebacterium Diphtheriae

Gram + Rod

Listeria Monocytogenes

Gram + Rod

Propionibacterium Acnes

Gram + Rod

Actinomyces

Gram + Rod Anaerobe

Clostridium Botulinum

Gram + Rod Anaerobe

Clostridium Difficile

Gram + Rod Anaerobe

Clostridium Perfringens

Gram + Rod Anaerobe

Clostridium Tetani

Gram + Rod Anaerobe

Enterococcus Faecalis

Gram + cocci

Enterococcus Faecium

Gram + cocci

Staphylococcus Coagulase

Gram + cocci

Staphylococcus Saprophyticus

Gram + cocci

Staphylococcus aureus

Gram + cocci

Streptococcus Bovis

Gram + cocci

Streptococcus Mutans

Gram + cocci

Streptococcus Pneumoniae

Gram + cocci

Streptococcus Pyogenes

Gram + cocci

Streptococcus agalactiae

Gram + cocci

What is the coverage of Clindamycin?

Gram +: -MSSA -CA-MRSA -S. Pyogenes, Viridians, S. pneumonia Anaerobes: -B. fragilis (some to none) -many others Protozoa: -Plasmodium -Pneumocystis carinii (P. jirovecci) -Toxoplasma gondii (Remember NO GRAM NEGATIVE coverage)

What is the coverage of Quinupristin + Dalfopristin?

Gram +: -MSSA -MRSA -VRSA -Streptococci -VRE (E. faecium only) - No faecalis but can always use Pen G for that (NO GRAM NEGATIVE COVERAGE)

What is the coverage of Macrolides?

Gram +: -MSSA -Some streptococci Gram -: -H. influenza -M. Catarrhalis -N. gonorrhea Atypical: -Mycobacterium avium complex (MAC) -Mycobacterium leprae

What is the coverage of Nitrofurantoin?

Gram +: -S. Saprophyticus Gram -: -E. coli except ESBL + & KPC +

What is the coverage of Chloramphenicol?

Gram +: -Streptococci (including some Pen-resistant) Gram -: -H. influenzae, Neisseria, Salmonella, shigella Anaerobes: -B. Fragilis, many others Atypical: -Chlamydia, Mycoplasma, others

Coverage of Lefamulin:

Gram +: -Streptococci, MRSA, E. Faecium Gram -: -Moraxella, H. influenzae Atypical: -Mycoplasma, Chlamydia

Citrobacter spp.

Gram - Rod (Enteric)

E. Coli

Gram - Rod (Enteric)

Enterobacter spp.

Gram - Rod (Enteric)

Klebsiella pneumoniae

Gram - Rod (Enteric)

Morganella Morganii

Gram - Rod (Enteric)

Proteus Mirabilis

Gram - Rod (Enteric)

Proteus vulgaris

Gram - Rod (Enteric)

Providencia spp.

Gram - Rod (Enteric)

Salmonella spp.

Gram - Rod (Enteric)

Serratia spp.

Gram - Rod (Enteric)

Yersinia enterocolitica

Gram - Rod (Enteric)

Yersinia pestis

Gram - Rod (Enteric)

Acinetobacter spp.

Gram - Rod (Other)

Bordetella Pertussis

Gram - Rod (Other)

Campylobacter Jejuni

Gram - Rod (Other)

Haemophilus Influenzae

Gram - Rod (Other)

Pseudomonas aeruginosa

Gram - Rod (Other)

Vibrio Cholerae

Gram - Rod (Other)

What are the 3 major ways in which Gram - bacteria become resistant to Beta-Lactams?

Gram - becomes resistant: 1) Changes in affinity of PBPs for Beta-Lactams 2) Changes in Porin structure (Only difference) 3) Beta-Lactamase Production (Open B-Lactam ring) Gram + does not change porin structure which is the difference between the two

Moraxella Catarrhalis

Gram - cocci

Neisseria Gonorrhoeae

Gram - cocci

Neisseria Meningitidis

Gram - cocci

Bacteroides spp.

Gram - rod Anaerobe

Azithromycin is more effective for which type of bacteria?

Gram negative (Which is why it is commonly used for Upper respiratory infections)

The molecules of Quinupristin + Dalfopristin are large meaning they have no porin penetration which means no:

Gram negative coverage

Zanamivir has less resistance than Oseltamivir due to its _____ group in its structure which forms more H-bonds

Guanidino

_____ functional group in zanamivir makes it active against different flu strains with lesser chances of resistance

Guanidino

Guanine + sugar = ?

Guanosine

_____ is required for DNA relaxation and Topoisomerase IV is required to untie daughter DNA from the parent strand

Gyrase

What group structure on Amikacin protects it from enzyme modification?

HABA

Direct thrombin inhibitors have similar indications and side effects as heparin based drugs. However, they can be used in patients with ___

HIT

Abacavir Tx

HIV

Atazanavir Tx

HIV

Darunavir Tx

HIV

Delaviridine Tx

HIV

Didanosine Tx

HIV

Dolutegravir Tx

HIV

Efavirenz Tx

HIV

Elvitegravir Tx

HIV

Emtricitabine Tx

HIV

Enfivuritide Tx

HIV

Etravirine Tx

HIV

Fosamprenavir Tx

HIV

Indinavir Tx

HIV

Maraviroc Tx

HIV

Navirs Tx

HIV

Nelfinavir Tx

HIV

Nevirapine Tx

HIV

One virus that integrase inhibitors target

HIV

Raltegravir Tx

HIV

Rilpivirine Tx

HIV

Ritonavir Tx

HIV

Saquinavir Tx

HIV

Sofosbuvir is indicated with patients that are co-infected with ___

HIV

Stavudine Tx

HIV

Tipranavir Tx

HIV

Zidovudine Tx

HIV

gravir Tx

HIV

What two viruses do viral maturation inhibitors treat?

HIV and HepC

NNRTIs are used for ____

HIV-1

What is considered as a significant risk factor of tuberculosis?

HIV-1 infection

Acyclovir Tx

HSV

Clovir / uridine Tx

HSV

Docosonol Tx

HSV

Femciclovir Tx

HSV

Idoxuridine Tx

HSV

Penciclovir Tx

HSV

Trifluridine Tx

HSV

Valacyclovir Tx

HSV

Cidofovir Tx

HSV and CMV

Ganciclovir and Valganciclovir are used for the treatment of:

HSV and CMV

Ganiciclovir Tx

HSV and CMV

valvanciclovir Tx

HSV and CMV

Which Non-Enteric Gram - Rod causes: -Pneumonia -Meningitis -Hib Vaccine

Haemophilus influenzae - capsule

Which Non-Enteric Gram - Rod causes: -Otitis -Sinusitis -Conjunctivitis -Pneumonia (not covered by vaccine)

Haemophilus influenzae - no capsule

Azithromycin penetrates deep into the tissue, this indicates that it has a very long:

Half-life

MOA of Heparin:

Has high affinity for activation of Antithrombin III which inactivated thrombin and factor Xa

MOA of LMWH:

Has shorter polymer that effectively binds factor Xa and does not effectively bind thrombin

How does Gram - produce Beta Lactamase?

Has to produce small amounts in the periplasm

Adverse effects of Artemisinins:

Headache, loss of appetite, muscle pain, weakness

What unique side effect does the Azido group (N3) in Zidovudine cause?

Headaches (sensitive to heat and light)

The disruptions of signals in the heart indicates:

Heart block

Sympathetic activation leads to an increase in what?

Heart rate

Which Spiral bacteria causes: -Gastritis -Ulcers

Helicobactor pylori

Adefovir Tx

HepB

Entecavir Tx

HepB

Telbivudine Tx

HepB

Lamivudine Tx

HepB and HIV

Tenofovir Tx

HepB and HIV

Boceprevir Tx

HepC

Grazopravir Tx

HepC

Paritaprevir Tx

HepC

Ribavirin Tx

HepC

Simeprevir Tx

HepC

Sofosbuvir Tx

HepC

Telaprevir Tx

HepC

previr / asvirs

HepC

Fatal ____ _____ has been reported with the use of Ketoconazole

Hepatic Toxicity

What is the black boxed warning of Terbinafine?

Hepatitis

What major viral disease uses reverse transcription to copy its DNA genome?

Hepatitis B

Acyclovir and Valacyclovir are used for the treatment of ____

Herpes (HSV)

Idoxuridine and Trifluridine are used for the treatment of:

Herpes Keratitis (HSV-1)

What is the distribution of Colistin?

Highly protein bound; has long lasting effects

What is HA-MRSA?

Hospital-acquired MRSA

What is the route of administration of Penicillin G Benzathine?

IM use only, IV leads to death

What can happen if a patient starts taking antibiotics while on warfarin?

INR increases which increases the risk of bleeding (due to decreased amount of vitamin K due to antibiotics)

Amphotericin B has poor absorption which is why it is used as an __ instead of orally

IV

Foscarnet is a polar drug only available in which route of administration?

IV

What are the routes of administration for Colistin?

IV, Inhalation for cystic fibrosis

What are the routes of administration available for Ganciclovir?

IV, Intraocular implant

What are the routes of administration available for Acyclovir?

IV, Topical, Oral

Cidofovir is a prodrug that is available in which routes of administration?

IV, topical, ocular implant

When is Cycloserine contraindicated?

If patient has history of epilepsy

Recarbio is a combination of:

Imipenem + Cilastatin + Relebactam

When are the use of class IA drugs contraindicated?

In patients with: -QT syndromes -Hypersensitivity -Heart block

What does the phosphate group in Fosamprenavir do?

Increase solubility

Esterfication of ceuroxime into cefuroime axetil leads to:

Increased lipophilicity and oral bioavailability

Posaconazole should be taken with food but why is it best to be taken with coca-cola?

Increases absorption

What is the function of the phosphate group in Fostemsavir?

Increases solubility for the drug to be absorbed orally

How does Gram + produce Beta-Lactamase?

Inducible process; produces only when needs to

Oseltamivir Tx

Influenza

Peramivir Tx

Influenza

Ramantidine Tx

Influenza

Zanamavir Tx

Influenza

amantadine Tx

Influenza

MOA of ADP (P2Y) receptor antagonists?

Inhibit ADP-induced expression of platelet glycoprotein receptors and reduce fibrinogen binding and platelet aggregation

MOA of Echinocandins:

Inhibit B-1, 3-glucan synthase which inhibit fungal cell wall synthesis

MOA of Fluoroquinolones

Inhibit gyrase and topoisomerase IV which prevents bacteria's relaxation, separation, and DNA replication

Oseltamivir

Inhibition of viral release

Peramivir

Inhibition of viral release

Zanamavir

Inhibition of viral release

Ramantidine

Inhibition of viral uncoating

amantadine

Inhibition of viral uncoating

MOA of Remdesevir:

Inhibits RNA replication

MOA of Cycloserine

Inhibits conversion of L-Ala to D-Ala

MOA of Penicillin

Inhibits peptide crosslinking

MOA of Rifamycins

Inhibits prokaryotic RNA polymerase (RNA synthesis)

MOA of Linezolid and Tedizolid

Inhibits protein synthesis by binding to 50S

MOA of Bacitracin

Inhibits the regeneration of bactoprenol

What are the effects of Probenecid on Penicillin G excretion?

Inhibits tubular secretion

What is the site of action of Amantadine and Rimantadine?

Inhibits viral uncoating

Elvitegravir is an ____ inhibitor

Integrase

Dolutegravir

Integrase inhibitor

Elvitegravir

Integrase inhibitor

Raltegravir

Integrase inhibitor

The route of administration of Daptomycin is _____ only

Intravenous

Insufficient blood flow to heart via arteries

Ischemic Myocardium

How does low dose Colistin work in combination therapy?

It disrupts the outer membrane which allows greater effects of the companion drug and overcomes resistance due to efflux pumps

What is the role of Vitamin K?

It goes to the liver and acts as a co-factor to make factors 2,7,9,10

Why does Osteconazole not interact with human CYPs?

It has a tetrazole in its structure which causes no affinity for human CYPs

Why does prasugrel have a more rapid onset of action when compared to clopidogrel?

It has an ester bond which allows it to metabolize faster

What is the purpose of factor XIIIa (13a)?

It helps in formation of covalent bonds to make fibrins strong

What is the major use of Nitrofurantoin?

It is a "niche" antibiotic used for UTIs and uncomplicated Cystitis (NOT pyelonephritis)

How is Avibactam different from other Antipseudomonal Penicillins?

It is added with a 3rd gen cephalosporin that covers Pseudomonas

What is the major issue of Penicillin G oral absorption?

It is broken down to penillic acid at low pH so it cannot be used orally

Why can't Daptomycin be used for pneumonia?

It is inactivated by pulmonary surfactants

How does vitamin K supplementation overcome effects of warfarin?

It reduces warfarin effects

Why is Aztreonam called Monobactam?

It's mono beta-lactam ring is not fused to a second ring

What is the cause of resistance to Isoniazid

KatG mutations

Which azole is considered to be a prototype of azoles?

Ketoconazole

Telithromycin is also classified as a _____

Ketolide

Heparin is excreted by which organ?

Kidneys

Which Enteric Gram - Rod causes: -UTI -Pneumonia -Hospital Infections

Klebsiella

What does KPC mean?

Klebsiella, Pneumonia, Carbapenemase

Two anti HepB nucleoside/nucleotide analogs that are also used for HIV (4a)

Lamivudine, Tenofovir

Beta-Blockers black boxed warnings:

Lead to: -Severe angina -MI -Increased BP -Tachycardia (DO NOT stop abruptly, must be tapered off)

What can be the result if the methyl group of lanosterol is not removed and does not form into a double bond?

Leaky membranes, cell malfunction, & cell death

Which Atypical bacteria causes: -Legionaire's disease -Flu-like: water coolers, water storage

Legionella

What is antagonism?

Less effective outcome than 1 single drug (Ex: Beta-lactam + Tetracycline; Bacteriostatic + Bactericidal)

What protein synthesis inhibitor binds to 50S, and prevents ribosomal assembly?

Linezolid

What is the coverage of Linezolid and Tedizolid?

Linezolid: Gram+: -MSSA -MRSA -VRSA -Streptococci (Resistant strains that can become bactericidal) -VRE (Both E. Faecium and E. Faecalis) Tedizolid: -Same as linezolid but includes Linezolid resistance staphylococci

MOA of Daptomycin

Lipid portion goes into bacterial membrane and forms a permeable ion channel which leaks out

Semisynthetic glycopeptides are also known as ________

Lipoglycopeptides

Teicoplanin is not used in the U.S., it contains a _____ chain that is though to make glycopeptides more effective and this inspired other semi-synthetics

Lipophilic

Tigecycline would NOT be used in bacteremia because it has high _____

Lipophilicity

Minocycline is the most lipophilic of all the tetracyclines, how does this affect its PK profile?

Lipophilicity makes it have a longer t1/2

Which Gram + Rod causes: -Listeriosis in pregnant, elderly, and patients with immunodeficiency

Listeria Monocytogenes

All NNRTIs undergo ____ metabolism

Liver

The naphthalene ring in the structure of Nafcillin makes it more likely to be hydrolyzed in the ___

Liver

Quinupristin + Dalfopristin have an extended PAE which produces a:

Long lasting effect

What is acquired resistance?

Loss of sensitivity to a drug that used to work

Penicillin G is susceptible to degradation at ____ pH

Low

Terbinafine may exacerbate symptoms of which disease?

Lupus

Which gene has been found to confer resistance to Colistin & Polymyxin?

MCR-1 gene

What resistant strains are NOT covered by with 3rd generation cephalosporins?

MRSA

Which structural feature is responsible for adverse effects associated with Cefotetan?

MTT side chain (Causes same adverse effects as MTD but more severe)

Chloramphenicol inhibits 50S ribosomes and shares the binding site with which other drugs?

Macrolides and Clindamycin

What are examples of Bacteriostatic drugs?

Macrolides tetracyclines

What is the advantage of the methyl substitution on the ring of Aztreonam?

Makes drug highly resistant to many Beta-Lactamases

What is the major effect produced by nitrates?

Marked relaxation of veins (predominant effect) resulting in decreased preload (Remember: No direct effect on cardiac muscle)

Flucytosine is used in combination therapy with Amphotericin B and an azole for the treatment of ____

Meningitis

Vabomere is the combination of:

Meropenem + Vaborbactam

All tetracyclines are good chelators of ____ Ions

Metal

What does MBL mean?

Metallo-B-Lactamases (Metal-Zn)

What is MSSA?

Methicillin Sensitive Staphylococcus Aureus

How does resistance of Clindamycin occur?

Methylation of 23S rRNA

Griseofulvin binds to ______ in mitotic spindle

Microtubules

Enfuvirtide (T20) is an anti-HIV peptide that ___ gp41

Mimics

Tigecycline is a modified form of _____ containing a tert-butyl glycyl side chain

Minocycline

In Aminoglycoside resistance, bacteria produces enzymes that ____ aminoglycosides such as acetylation, phosphorylation, adenylation

Modify

Which Gram - Cocci causes: -Sinusitis -Otitis Media -Pneumonia

Moraxella Catarrhalis

Which Enteric Gram - Rod causes: -Postoperative UTI's

Morganella Morganii

What does Tn refer to in Tn3?

Multi-resistant transposon that contains Beta-Lactamases resistance genes

A single plasmid may contain ____ R-determinants

Multiple

Linezolid resistance occurs through a unique _____ in ribosome but this does not affect Tedizolid binding

Mutation

Which Atypical bacteria causes: -Upper RTI -Pneumonia

Mycoplasma Pneumoniae

What is the spectrum of activity of Pen G and related drugs?

Narrow Spectrum

Suffix for Anti HIV viral maturation inhibitors

Navirs

Which Gram - Cocci causes: -Gonorrhea

Neisseria Gonorrhoeae

What is the Gram - Cocci organism covered by Penicillin G?

Neisseria Meningitidis

Which Gram - Cocci causes: -Meningococcal Meningitis -Septicemia

Neisseria Meningitidis

What is a major toxicity of Cidofovir?

Nephrotoxicity

What does Zanamivir, Oseltamivir, and Peramivir block?

Neuraminidase

When Ganciclovir and Valganciclovir are combines with myelosuppressive drugs, an adverse effect that can occur is called:

Neutropenia

What is the cause of resistance to Pyrazinamide which can decrease activation?

Nicotinamidases

Is the normal flora always harmless?

No

Foscarnet

Non- nucleoside analog DNA polymerase inhibitor

Why are Non-DHP calcium channel blockers used for antiarrhythmias and not DHP calcium channel blockers?

Non-DHP CCB's effect the heart tissue which is what we need, DHP CCB's effect arterial tissue which is not what we need

Multi-drug resistant TB (MDR-TB) is resistance to Isoniazid (INH) and Rifampin (RMP) that may occur due to:

Non-adherence

Delaviridine

Non-nucleoside reverse transcriptase inhibitor

Efavirenz

Non-nucleoside reverse transcriptase inhibitor

Etravirine

Non-nucleoside reverse transcriptase inhibitor

Nevirapine

Non-nucleoside reverse transcriptase inhibitor

Rilpivirine

Non-nucleoside reverse transcriptase inhibitor

Bicyclic structure creates even more strain making it more reactive and susceptible to ______ attack

Nucleophilic

Abacavir

Nucleoside / Nucleotide analog

Acyclovir

Nucleoside / Nucleotide analog

Cidofovir

Nucleoside / Nucleotide analog

Didanosine

Nucleoside / Nucleotide analog

Emtricitabine

Nucleoside / Nucleotide analog

Entecavir

Nucleoside / Nucleotide analog

Femciclovir

Nucleoside / Nucleotide analog

Ganiciclovir

Nucleoside / Nucleotide analog

Idoxuridine

Nucleoside / Nucleotide analog

Lamivudine

Nucleoside / Nucleotide analog

Penciclovir

Nucleoside / Nucleotide analog

Ribavirin

Nucleoside / Nucleotide analog

Sofosbuvir

Nucleoside / Nucleotide analog

Stavudine

Nucleoside / Nucleotide analog

Telbivudine

Nucleoside / Nucleotide analog

Tenofovir

Nucleoside / Nucleotide analog

Trifluridine

Nucleoside / Nucleotide analog

Valacyclovir

Nucleoside / Nucleotide analog

Zidovudine

Nucleoside / Nucleotide analog

valvanciclovir

Nucleoside / Nucleotide analog

Three categories of Viral Genome Replication inhibitors

Nucleoside/nucleotide analogs, Non-nucleoside analog DNA polymerase inhibitors, Non-Nucleoside Reverse Transcriptase inhibitors

What is Jarisch-Herxheimer reaction?

Occurs from treating Treponema (Syphilis) Symptoms include: Fever, chills, headache, hypotension within hours of 1st dose due to cytokine release

Which two drugs should we avoid taking clopidogrel with due to CYP2C19 interactions?

Omeprazole and Esomeprazole

What are the routes of administration available for Valacyclovir?

Oral

What is the route of administration available for Famciclovir?

Oral

What is the route of administration available for Valganciclovir?

Oral

What is Oteseconazole used for?

Oral treatment of resistant vulvovaginal candidiasis

Where is LPS located within Gram negative bacteria?

Outer lipid bilayer

Conversion of Famciclovir to Penciclovir requires which 2 steps?

Oxidation then Deacetylation (Hydrolysis)

Atrial depolarization is defined as what wave?

P wave

This is the time required for bacteria to recover and resume bacterial growth. It is the continued antibiotic effect after concentration falls below MIC

PAE (Post-Antibiotic Effect)

Sulfuonamides resemble P-aminobenzoic acid because of the ____ position of the ring in their structure

PARA

What are drug-drug interactions of Clopidogrel?

PPI's (Omeprazole) and other CYP2C19 inhibitors

These are cells that control rates of the heart

Pacemaker cells

When you can feel your heart beat, you are feeling a:

Palpitation

Multi-resistance transposons are present in non-pathogenic bacteria, and they jump from non-pathogenic to:

Pathogenic species

What is essential for successful eradication of Tuberculosis disease?

Patient Adherence

What is an example of Narrow spectrum activity?

Penicillin

When Beta-Lactams bind to transpeptidase, a serine protease, to inactive it, what else is inactivated?

Penicillin Binding Proteins

The conjugated form of Natural penicillin G is known as:

Penicillin G Benzathine

Beta-Lactams are ONLY effective against:

Penicillinases

Macrolides inhibit ______ ______ which blocks peptide bond formation

Peptidyl Transferase

Which Gram + Cocci Anaerobe causes: -Neurotoxin; botulinum

Peptostreptococcus

What is the new neuraminidase inhibitor that is only used for influenza infection in hospitalized patients because it has less resistance and more effective binding?

Peramivir (IV route)

This is the space where the bacterial cell wall is contained between the 2 lipid layers of Gram - bacteria?

Periplasmic space

Not being able to convert back to normal sinus rhythm is known as:

Permanent Atrial Fibrillation

Ceftaroline's phosphate group is removed by:

Phosphatases in plasma

Which enzyme do platelets activate to form TxA2?

Phospholipase A2 (PLA2)

Nucleoside analogs must be ______ to become nucleotide analogs

Phosphorylated

All tetracyclines can undergo _____, and the result of this can cause photosensitivity and yellow teeth in some patients

Photo-degradation

Vitamin K is also known as:

Phytonadione

Nematode drugs are useful against what?

Pinworm, roundworm, hookworm

Warfarin crosses the _____, this means it is lipophilic and cannot be used in pregnant patients

Placenta

Activated fibrinogen binding protein GPIIb/IIIa binds to other platelets via fibrinogen during which phase of primary hemostasis?

Platelet Aggregation

What can occur if we block GPIIb-IIIa receptors?

Platelets will not be able to aggregate

Lefamulin is a ____ antibiotic

Pleuromutilin

Amphotericin B is a ____ that is insoluble in water

Polyene

___ in ___ inhibit fimbriae binding.

Polyphenols in cranberries

These are channels of Gram - bacteria that allows polar molecules to go through the outer lipid layer then periplasmic space and into the inner lipid layer

Porins

What is the benefit of Posaconazole compared to Voriconazole?

Posaconazole does not cause visual disturbances the way Voriconazole does

This ADP receptor inhibitor binds covalently (irreversibly) and is activated by CYP3A4 enzyme. It is more potent than clopidogrel

Prasugrel

Vitamin K antagonists should not be used in which group of patients?

Pregnant patients Remember: Heparin and LMWH can be used safely

MOA of Glycoprotein IIb-IIIa receptor antagonists

Prevent binding and cross-linking of platelets by binding to platelet glycoprotein IIb-IIIa receptors

Ceftaroline fosamil is a ____

Prodrug

Colistimethate is a ____ that is converted to Colistin in vivo

Prodrug

Prontosil is the _____ of sulfanilamide

Prodrug

Which Gram + Rod causes: -Skin flora -Sebum -Acne (also anaerobic)

Propionibacterium Acnes

What is the functional significance of the methoxy groups in Methicillin?

Protect the Beta-Lactam ring from Beta-Lactamase

Vitamin K plays a role in the formation of which proteins?

Proteins C and S

Which Enteric Gram - Rod causes: -UTI -Wound infections -P. Mirabilis (indole -) -P vulgaris (indole +)

Proteus

Warfarin dose is based on the patients _____ ____ (PT) which is expressed as INR

Prothrombin time (PT)

Which Enteric Gram - Rod causes: Hospital related: -UTI's -Catheters -Burns

Providencia

What does red man syndrome cause?

Pruritus, erythematous rash on face, neck, upper torso

What is the serious potential adverse effect that is most commonly associated with Clindamycin?

Pseudomembranous Colitis

Give an example of Superinfection.

Pseudomembranous colitis

What is an example of superinfections?

Pseudomembranous colitis caused by clostridium difficile -loss of competing flora because of antibiotic treatment -Can occur during or after treatment

Antipseudomonal Penicillins are used to treat _____

Pseudomonas

Which Non-Enteric Gram - Rod causes: Resistant opportunistic: -UTI -Pneumonia -Wound infections (burns)

Pseudomonas Aeruginosa

Why are pseudomonas so difficult to treat

Pseudomonas are Gram - rods that express that expresses highly selective porin as well as an efflux pump capable of excluding even small polar drugs which makes it harder for antibiotics to stay in the cell which causes resistance

Pyrazinamide (PZA) is a prodrug that is hydrolyzed in acid or by nicotinamidase to form:

Pyrazionoic acid

Foscarnet mimics ______

Pyrophosphate

Ventricular depolarization is defined as what?

QRS complex

Ventricular depolarization and repolarization together is defined as what interval?

QT interval

What is the major adverse effect of ranolazine?

QT prolongation

Coronavirus is what kind of virus?

RNA

HIV is an ____-based virus and has HIV-1 and HIV-2 families

RNA

Retrovirus is what kind of virus?

RNA

What are the two different types of viruses?

RNA and DNA

Fidaxomicin has a unique mechanism by interfering with:

RNA synthesis

Ofloxacin is a ____ mixture

Racemic

Three anti HIV integrase inhibitors

Raltegravir, Elvitegravir, Dolutegravir

What are the 2 main adverse effects of Fosfomycin?

Rash and Headache

What are the effects of beta-blockers in IHD?

Reduce oxygen demand (decrease HR and contactility)

MOA of Aspirin

Reduces TXA2 production which causes an antiplatelet effect

This is the period of time in any cell in which the action potential cannot be generated

Refractory period

Zanamivir, Oseltamivir, and Peramivir are anti-influenza drugs that inhibit viral ____

Release

What is the only FDA approved drug for SARS-CoV-2 (Covid-19)?

Remdesevir

How are Acyclovir and Valacyclovir excreted?

Renally

How is Daptomycin excreted?

Renally

How is Fluconazole eliminated?

Renally

Aztreonam is eliminated _____

Renally (in an unchanged form)

R plasmids or R factors are known as:

Resistance Plasmids

Are antipseudomonal Penicillin's not fully ____ to beta-lactamases

Resistant

B-lactamse binds to nucleophile, inactivating the drug and inactivates itself in the process but makes more and becomes:

Resistant

What is the coverage of Ceftolozane?

Resistant Gram - infections: Pseudomonas, Some ESBL, B. Fragilis

Polymyxins are old drugs that were not used much due to toxicity but are back in use due to their utility against:

Resistant strains

More refractory at lower heart rates (proarrhythmic) is the definition for:

Reverse-use dependency

This is known as mutations in 30S ribosomal subunit render it less susceptible to binding to aminoglycosides

Ribosomal Resistance

Which Atypical bacteria causes: -Spotted fever -Typhus: spread by fleas/ticks

Rickettsia spp.

Amphotericin B is used in synergic with which drug?

Rifampin

What can occur if a patient is deficient on Protein C?

Risk of developing blood clots

What is the black boxed warning for Prasugrel and Ticagrelor?

Risk of more bleeding

Levofloxacin is a pure _ isomer that is more potent and active particularly against Gram + species compared to ofloxacin

S isomer

Beta-Blockers (Class II drugs) have a primary effect on:

SA and AV nodes

For sympathetic effects, which node is most important for normal heart rate control?

SA node

This is a node that is on-top of the right atrium

SA node

Where are Tetracyclines distributed to?

Saliva, bone, nails, teeth, fatty tissue (Minocycline)

Which Enteric Gram - Rod causes: -Food poisoning -Thyroid fever

Salmonella

Nine Anti HIV viral maturation inhibitors

Saquinavir, Ritonavir, Fosamprenavir, Nelfinavir, Lopinavir, Atazanavir, Tipranavir, Darunavir

Define Superinfection.

Secondary infection arising after an initial treatment of the original infection

Which Enteric Gram - Rod causes: -Hospital infections -UTI's -RTI's -Wound and Blood infections

Serratia

What is the role of Ritonavir for HIV and Hep C?

Serves as a booster for Protease inhibitors

Which Enteric Gram - Rod causes: -Shigellosis -GI/diarrhea (Usually all self resolving)

Shigella

What is Docosanol used for?

Shortens healing time of cold sores due to oral herpes (HSV)

Oseltamivir and Zanamivir are analogues of ______ that compete with and inhibit Neuraminidase

Sialic Acid

Cefiderocol is a new cephalosporin that is a ____

Siderophore

This is a chemical produced by a different bacterium. It binds and transports iron in microorganisms. (It binds iron and is imported into bacteria; trojan horse). Once inside it will inhibit cell wall synthesis. What is this molecule?

Siderophore

Amphotericin B can cause Azotemia in ____ toxicity

Slower

What is the active form of Sofosbuvir?

Sofosbuvir Diphosphate

Two anti HepC Nucleoside/nucleotide analogs (4a)

Sofosbuvir, ribavirin

This is an example of a sex pilus:

Specialized fimbriae

Borrelia Burgdorferi

Spiral

Helicobacter pylori

Spiral

Treponema Pallidum

Spiral

What bacteria has a "rigid" appearance?

Spirillum

What type of bacteria is Helicobacter pylori and what is its appearance?

Spirillum (rigid)

What bacteria has a "thinner, mobile spirals" appearance?

Spirochete

What type of bacteria is Borrelia burgdorferi and what is its appearance?

Spirochete (thinner, mobile spirals)

What type of bacteria is Treponema pallidum and what is its appearance?

Spirochete (thinner, mobile spirals)

Which enzyme does Terbinafine inhibit?

Squalene Epoxidase

What are the inhibitors of the 3 stages of cell wall synthesis?

Stage 1: Cycloserine, Fosfomycin Stage 2: Bacitracin, Vancomycin Stage 3: Beta-lactams

Describe the actions of the 3 stages of cell wall synthesis:

Stage 1: Cytoplasmic stage: Synthesis of precursors NAG and NAM (Abx inhibitors: Cycloserine & Fosfomycin) Stage 2: Membrane stage/elongation and transfer: Transfer of NAG and NAM from cytosol to membrane and incorporates into the growing peptidoglycan (Abx inhibitros: Bacitracin & Vancomycin) Stage 3: Extracellular stage (crosslinking): Crosslinking of peptidoglycan chains by membrane bound transpeptidases (Abx inhibitors: Beta-Lactams)

What bacteria has a "cluster" appearance?

Staphylococcus

What type of bacteria is Staphylococcus aureus and what is its appearance?

Staphylococcus (clusters)

Which Gram + Cocci causes: -Infection of prosthetics, catheters, implants

Staphylococcus Coagulase negative

Which Gram + Cocci causes: -Uncomplicated UTI's

Staphylococcus Saprophyticus

Which Gram + Cocci causes: -Bacteremia -Endocarditis -SSTIs

Staphylococcus aureus

What type of bacteria is Bacillus anthracis and what is its appearance?

Streptobacillus (chain)

Which Gram + Cocci causes: -Oral flora -Endocarditis -Brain abscess

Streptococci Viridians group

What type of bacteria is Streptococcus pneumoniae and what is its appearance?

Streptococcus (chain)

Which Gram + Cocci causes: -Upper/Lower RTI's -Meningitis

Streptococcus Pneumonia

Which Gram + Cocci causes: -Pharyngitis -Cellulitis (Including "Flesh eating")

Streptococcus Pyogenes

Which Gram + Cocci causes: -Neonatal meningitis/bacteremia

Streptococcus agalactiae

What protein synthesis inhibitors binds two sites on 50S using cooperative binding?

Streptogramins (Quinupristin + Dalfopristin) = Synercid

Elvitegravir is only available in combination with cobicistat + emtricitabine + tenofovir as:

Stribild

_____ is a common biotransformation pathway for all H2 antagonists

Sulfoxidation

Pseudomembranous colitis is an example of ___.

Superinfection

Atrial Tachycardia (A-fib) is also known as:

Supraventricular tachcardia

What is Griseofulvin used for?

Systemic treatment of dermatophytes (long-term treatment of about 6-12 months for nails)

What is the abbreviation for Tenofovir Disoproxil

TDF

When the heart has a fast abnormal beat this is known as:

Tachyarrhythmia

The increased firing rate of > 100 bpm is known as: (Remember: this is more common to occur)

Tachyarrhythmias

Ceftolozane is combined with ______ for maximum effectiveness against Beta-Lactamases producers

Tazobactam

Ten Anti HepC viral maturation inhibitors

Telaprevir, boceprevir, simeprevir, paritaprevir, grazoprevir, ombitasvir, ledipasvir, elbasvir, velpatasvir, daclatasvir

Which NRTI is the only nucleotide? (All the others are nucleosides)

Tenofovir

Nucleotide analogs mimic deoxyribonucleotides, which are a chain elongation ______

Terminators

What protein synthesis inhibitor binds to 30S, blocks tRNA acceptor site?

Tetracycline

What is an example of Broad spectrum activity?

Tetracyclines

Posaconazole is an oral suspension that has a wide spectrum compared to other azoles due to the _____ ____ in its structure

Tetrahydrofuran Ring

Why is the Beta-Lactam ring so reactive with Nucleophiles?

The 4-membered cyclic amide ring is under great strain, making it very reactive

What are the structural features that allow antipseudomonal penicillins to penetrate Pseudomonas?

The Urea group helps Antipseudomonal Penicillins penetrate Pseudomonas: -Increased polarity -Better Porin penetration -Evade efflux pumps

Humans do not have cytosine deaminase but we can still find 5-FY in patients taking Flucytosine. Why is that?

The gut flora in humans expresses cytosine deaminase

How does the methoxy substituent on the beta-lactam ring of cephamycins affect their susceptibility to beta-lactamses?

The methoxy group increases the beta-lactam ring stability

What is vertical transfer of resistance and what is the major driving force behind it?

The selection pressure caused by presence of an antibiotic

Are PRPs useful against all beta-lactamase prodrug strains?

These drugs are resistant to Beta-Lactamases but not all -Useful against MSSA -Not useful against MRSA (due to PBPs not being Beta-Lactamse)

What is the underlying mechanism of action & target of Quinupristin + Dalfopristin?

They bind to 2 different sites on 50S

What is the effect of Amantadine and Rimantadine on endosome pH?

They buffer the pH of endosomes and prohibit acidic environment needed

Why should Zidovudine and Stavudine not be used in combination?

They compete with each other for phosphorylation and antagonize each other's effect

How is the coverage of cefoxitin and cefotetan different from other 2nd generation drugs?

They cover Gram - bacteria a lot better but do not have much affinity for Gram + PBPs

How does the coverage of cephamycins DIffer from other 2nd generation drugs?

They cover anareobes and have higher affinity for Gm - PBPs

Why doe Ticagrelor and Cangrelor have an advantage over clopidogrel?

They do not involve CYP2C19 to be activated

What is the major advantage of antipseudomonal penicillins?

They extend the spectrum for difficult to treat Gram - strains (Ex: pseudomonas)

How are Cephamycins different from the rest of the Cephalosporins?

They have a methoxy group (OCH3) instead of H+ group which causes different activity

Why are Cefaclor and Cefprozil oral drugs?

They have amino groups (NH2) in their structures which makes a drug more stable in stomach acid

Why do Ticagrelor and Cangrelor have an advantage over parsugrel?

They have much faster onset of action

What is the purpose of the polymerase inhibitor prodrugs?

They increase oral absorption

Violet and iodine stain cannot be washed off because of the ____ cell wall and G+. The dominant stain then appears violet

Thick

Decolorization (alcohol wash) washes away the stain as well as the outer lipid layer of of the ____ cell wall on G-

Thin

The higher the INR, the _____ the blood

Thinner

Aspirin inhibits enzyme for the entire life of the platelets (7-10 days); which decreases _____

Thromboxane A2

Thymine + sugar = ?

Thymidine

Idoxuridine and Trifluridine are derivatives of:

Thymine and Uracil

Protease Inhibitors are peptidomimetics except for _____

Tipranavir

This aminoglycoside is 2 times more effective than gentamicin against Pseudomonas

Tobramycin

Docosanol is polar and lipophilic, therefore it is not available orally. What formulation is it available in?

Topical (Abreva, used for cold sores)

What is the route of administration available for Penciclovir?

Topical (cream)

Ketoconazole is mostly used _____

Topically

QTc > 510 mS correlates with onset of: (2 issues)

Torsades de pointes (TDP) and Cardiac arrest

This an enzyme that is also known as Penicillin Binding Protein (PBP). It is an enzyme that catalyzes a peptide-peptide bond formation between D-Ala and L-Gly

Transpeptidases

Transposons (Tn) are catalyzed by _____ encoded within the transposon

Transposase

What is Rifampin used for?

Treat TB

Which Spiral bacteria causes: -Syphilis

Treponema pallidum

T/F: 14-a-demethylase is a P450 enzyme (CYP51)

True

T/F: 23s rRNA methylation does not affect the actions of Linezolid and Tedizolid

True

T/F: 3rd generation fluoroquinolones have the longest half-life of all the fluoroquinolones

True

T/F: A normal range for heart rate is 60-100 bpm (Average being 75 beats per min). When heart rate is less than 60 bpm this indicates Bradycardia. When heart rate is more than 100 bpm it indicated tachycardia

True

T/F: A patient should avoid taking clopidogrel or prasugrel if they have an allergy to sulfa due to the drugs having a sulfur bond in their structure

True

T/F: ADP is important in mediating platelet aggregation. This means it causes platelets to become "sticky" and adhere to one another

True

T/F: Adefovir/Tenofovir diphosphate is an active form

True

T/F: Adenosine, Digoxin, and electrolytes such as potassium and magnesium are considered "Class V" antiarrhythmics

True

T/F: All 2nd generation cephamycins are administered parenterally

True

T/F: All NNRTIs must be used in combination with NRTIs

True

T/F: All azoles can potentially be teratogenic

True

T/F: All beta lactam antibiotics are bactericidal

True

T/F: All fluoroquinolones are effective against gram negative bacteria such as Moraxella and H. influenzae

True

T/F: Amantadine and Rimantadine are used for prophylaxis of infections by influenza A virus and are most effective if given during the first 48 hrs of onset of symptoms. However, they are no longer used due to their widespread resistance

True

T/F: Aminoglycosides are aerobic, gram negative drugs

True

T/F: Aminopenicillins and Antipseudomonal Penicillins are part of the Extended spectrum

True

T/F: Amphotericin B has a broad spectrum

True

T/F: An E. coli strain expressing fimbriae is more likely to cause a severe UTI. Therefore, fimbriae is a virulence factor

True

T/F: Antiarrhythmics can cause arrhythmias. These are drugs that are in class I and III

True

T/F: Antibiotics can disrupt the balance of normal flora

True

T/F: Artemether + Lumefantrine (Coartem) is a very effect antimalarial drug

True

T/F: Aspirin binds to the cyclooxygenase enzyme to block TxA2

True

T/F: At equivalent doses, Acyclovir and Valacyclovir would have similar amount of activity

True

T/F: Azithromycin and Erythromycin are recommended to be taken on an empty stomach

True

T/F: Aztreonam only comes as a parenteral route of administration

True

T/F: Bacteria divide by binary fission

True

T/F: Bacteria do not contain a nucleus

True

T/F: Bacteria do not contain cytoplasmic organelles

True

T/F: Bacterial ribosomes contain 30s and 50s subunits

True

T/F: Bacteriocins are toxins produced by normal flora and are toxic to the pathogenic bacteria

True

T/F: Bedaquiline inhibits proton pump - energy metabolism

True

T/F: Beta-Blockers in IHD increase diastolic perfusion time which may increase myocardial perfusion. They have no effect on oxygen supply

True

T/F: Betrixaban is only approved for VTE (prophylaxis) but has been discontinued off the market

True

T/F: Biological heparin is animal based whereas synthetic heparin is made in a lab

True

T/F: Borrelia burgdorferi is a spirochete

True

T/F: Cangrelor cannot be used for prophylaxis because it is an IV only drug

True

T/F: Capsulated strains can cause meningitis

True

T/F: Carbapenem is not available in oral form and is only available parentally because they are poorly absorbed orally

True

T/F: Carbapenems do not cover Atypical strains

True

T/F: Carbapenems penetrate the blood brain barrier and can be used for meningitis

True

T/F: Cefepime is a 4th generation cephalosporin

True

T/F: Cefiderocol can cause false positive Urine dipstick tests

True

T/F: Ceftaroline is the only beta-lactam effective against MRSA

True

T/F: Cephalexin and Cefadroxil are 1st generation cephalosporins that are administered orally while Cefazolin is a 1st generation cephalosporin administered parentally

True

T/F: Chloramphenicol is a broad spectrum antibiotic

True

T/F: Cidofovir is a nucleotide

True

T/F: Cimetidine irreversibly binds to the cytochrome P450 enzymes

True

T/F: Clarithromycin and Telithromycin are ok to take with food

True

T/F: Clarithromycin can be taken with or without food

True

T/F: Class I drugs have effects on ventricular myocytes. They slow phase 0 depolarization and may affect rate of repolarization

True

T/F: Class IC is used typically for life threatening supraventricular tachyarrhythmias (SVT) and ventricular tachyarrhythmia. It may increase risk of death in patients with history of myocardial infarction or sustained ventricular arrhythmias

True

T/F: Class III antiarrhythmic drugs are the most aggressive and have lots of side effects. Most of these agents have mixed (multi-class) activity

True

T/F: Clindamycin has limited porin penetration which is why it does not cover Gram negative bacteria

True

T/F: Clopidogrel and Prasugrel are prodrugs. Ticagrelor is not a prodrug

True

T/F: Colistin is often among the drugs of last resort

True

T/F: Combining Beta-blockers (Class II) with calcium channel blockers (Class IV) together can increase the risk of heart failure

True

T/F: Concentration-dependent killing means the more concentration of a drug the more killing occurs

True

T/F: DDIs with warfarin are much more significant when CYP2C9 is inhibited as compared to when CYP1A1 is inhibited

True

T/F: Dabigatran is a prodrug and is a good alternative to warfarin that does not require frequent blood tests

True

T/F: Dalbavancin is a semisynthetic glycopeptide that has a similar coverage to Vancomycin but has a longer half-life and is better tolerated

True

T/F: Dalbavancin is not effective against vancomycin-resistant strains (VRSA)

True

T/F: Daptomycin should not be used in patients with Pneumoniae

True

T/F: Daptomycin should not be used to treat pneumonia

True

T/F: Decrease in load of H. pylori infection enhances the possibility of ulcer healing

True

T/F: Despite its many adverse reactions, Amiodarone is a highly effective drug for the treatment/prevention of multiple arrhythmias. It also does not cause Torsades de pointes (TDP). These two reasons are why it is still on the market

True

T/F: Didansoine, Abacavir, Tenofovir can be combined

True

T/F: Docosanol is a fatty saturated alcohol that can be used for reducing duration of cold sores caused by herpes virus

True

T/F: Docosanol is used as an HSV inhibitor for viral attachment and entry

True

T/F: Drug resistance to Tenofovir and Entecavir are much less than other drugs making them some of the best drugs for Hepatitis B

True

T/F: During platelet adhesion, von Willebrand factor binds to glycoprotein Ib [GPIb] on the platelet membrane and to the exposed collagen. Glycoprotein Ib [GPLb] helps adhere to platelets

True

T/F: Each bacteria has properties that give it ability to conjugate

True

T/F: Enfuvirtide is an oligo-peptide drug but Atazanavir is a peptidomimetic drug

True

T/F: Ertapenem does not have reliable coverage of pseudomonas, acinetobacter and Enterococci

True

T/F: Erythromycin and Azithromycin can also be used parenterally

True

T/F: Erythromycin is the most acid labile macrolide

True

T/F: Ethambutol works in synergy with Isoniazid

True

T/F: Factor Xa represents the active form of factor X

True

T/F: Fibrinogens are blood soluble proteins but fibrins are quite insoluble and help in blood clotting

True

T/F: Fimbriae, Flagella, and Capsules are virulence factors

True

T/F: Fluconazole is a water soluble drug that has good absorption when taken with or without food

True

T/F: Fluoroquinolones should not be given to patients under 18 years of age to prevent tendinopathy from occurring unless the benefits outweighs the risk

True

T/F: Fondaparinux does not bind to thrombin. It activates AT-III and blocks factor Xa

True

T/F: Fosamprenavir is a phosphate prodrug of amprenavir and offers more patient compliance with decreased dose

True

T/F: Foscarnet can be used for CMV, HSV-1, & HSV-2

True

T/F: Foscarnet does not require activation before attacking viral enzymes

True

T/F: Foscarnet is not a prodrug nor a bioprecursor

True

T/F: Fosfomycin has Oral and Parenteral routes available

True

T/F: Fosfomycin is safe in Pregnancy

True

T/F: Fostemsavir is a phosphate prodrug

True

T/F: Glycopeptides are not orally bioavailable

True

T/F: Gram + bacteria have thicker cell walls but lack an outer lipid layer

True

T/F: Gram + bacteria stain violet/purple at the end of the gram staining procedure

True

T/F: Gram - bacteria contain an outer lipid layer consisting of LPS

True

T/F: Gram - bacteria stain pink/red at the end of the gram staining procedure

True

T/F: H2 antagonists are polar compounds acting as partial agonists at the H2 receptors

True

T/F: Harmless genes may pass genes to harmful ones. An example of this is genes can transfer from E. Coli to Klebsiella to Salmonella

True

T/F: Having a carbon in place of sulfur in a carbapenem makes the ring more strained and reactive which is why it has a broad spectrum

True

T/F: Heparin based drugs can be used in pregnancy

True

T/F: Heparin blocks (inhibits) activated factors

True

T/F: Heparin comes in two forms, biological and synthetic. Biological heparin can be high molecular weight Heparin (HMWH) and low molecular weight Heparin (LMWH)

True

T/F: Heparin is only administered parenterally

True

T/F: Hib vaccine protects against capsulated strains only

True

T/F: Horizontal transfer of resistance occurs between 2 non-pathogenic bacteria, or 2 pathogenic bacteria, or pathogenic and non-pathogenic

True

T/F: If a patient has resistance to carbapenem and colistin we could not treat them

True

T/F: Immunocompromised patients have neutropenia (less neutrophils to phagocytose bacteria), so their immune system is weaker than a healthy adult

True

T/F: In regards to Class IB antiarrhythmic agents, Lidocaine has a large therapeutic window. Phenytoin is no longer used often but is effective for V-Tach in young children and may be useful for patients with arrhythmia and epilepsy

True

T/F: Inactivation process of anti-thrombin III is enhanced by heparin-like molecules

True

T/F: Integrase inhibitors are only used in combination

True

T/F: Isoniazid is orally available and is metabolized by N-acetyltransferase (NAT2)

True

T/F: Itraconazole can worsen congestive heart failure and should be avoided in patients with underlying cardiovascular disease because it can impair cardiac function

True

T/F: Itraconazole is used for non-meningeal systemic mycosis meaning it is not useful for CSF infections

True

T/F: LPS is considered to be an endotoxin

True

T/F: Lamivudine and Emtricitabine are both cytidine derivatives and should not be used in combination

True

T/F: Lefamulin is available in both Oral and Parenteral forms and is approved for CAP (community-acquired pneumonia)

True

T/F: Linezolid and Tedizolid are bacteriostatic

True

T/F: Linezolid and Tedizolid do not cover gram negative bacteria because they get effluxed

True

T/F: Maraviroc mimics the loss of function polymorphism known to protect people from HIV infection, therefore some people cannot get HIV-1 infection because they do not have a functional CCR-5

True

T/F: Meropenem + Vaborbactam is a new non-beta lactam beta lactamase inhibitor that inhibits most ESBL and KPC enzymes but has no effect on MBL

True

T/F: Minocycline is used for inflammatory acne because it has coverage over propionibacterium acnes

True

T/F: Mycobacterium tuberculosis is a disease that mostly affects the lungs and is historically referred to as "consumption"

True

T/F: NS3/4A, NS5A, and NS5B inhibitors inhibit viral maturation

True

T/F: Nalidixic acid was the first quinolone that was very effective but had poor tissue penetration

True

T/F: Nitromidazoles are eliminated renally

True

T/F: Non-thienopyridines such as Ticagrelor and Cangrelor have a much higher bleeding risk when compared to thienopyridines (Ticlopidine, Clopidogrel, Prasugrel)

True

T/F: Normal flora is harmless but can be harmful when it escapes its normal location

True

T/F: Only erythromycin can convert to cyclic ketal that binds to motilin receptors in the GI tract

True

T/F: Oteseconazole is more potent than Fluconazole

True

T/F: PRPs are resistant to acid when compared to Pen G, their GI absorption decreases with food, and they have short half-lifes of about 30-60 mins

True

T/F: Penciclovir is used for the treatment of Genital Herpes whereas Famciclovir is used for the treatment of genital herpes + herpes zoster

True

T/F: Penicillin Binding proteins are inhibited by beta-lactams. Some species may have several of these proteins

True

T/F: Penicillin G Benzathine is used for only highly sensitive strains

True

T/F: Penicillin G has no gram - rod coverage

True

T/F: Piperacillin has better coverage and better potency compared to Ticarcillin

True

T/F: Plazomicin is similar to amikacin (but more effective) in that it is able to overcome resistance

True

T/F: Polymyxins are cationic cyclic peptides with hydrophobic tails

True

T/F: Posaconazole is effective against aspergillus

True

T/F: Prasugrel is a prodrug

True

T/F: Prolonged QT interval is very dangerous and very fatal which can cause cardiac arrest if QTc is > 510 mS

True

T/F: Protease inhibitors should only be used in combination

True

T/F: Protein S is the cofactor required for activation of Protein C

True

T/F: Pseudomonas expresses highly selective porin as well as efflux pump capable of excluding small polar drugs, which makes it harder for antibiotics to stay in the cell and causes resistance. This makes Pseudomonas often difficult to treat

True

T/F: Quinupristin + Dalfopristin have NO gram negative coverage

True

T/F: Resistance develops in gut flora

True

T/F: Reverse transcriptase is a good target for HBV

True

T/F: Rifabutin is more lipophilic and has a larger Volume of distribution and clearance compared to Rifampin

True

T/F: Rifampin is a potent inducer of many CYPs

True

T/F: Rifampin is almost always used in combination with other drugs

True

T/F: Rifampin is antagonistic in combination therapy with protein synthesis inhibitors including with aminoglycosides

True

T/F: Sofosbuvir is a chain terminator because its -OH group is not available for chain elongation

True

T/F: Sofosbuvir is better tolerated than protease inhibitors and requires shorter treatment duration

True

T/F: Streptogramins are incompatible with saline & heparin. We must use only 5% dextrose to flush the IV line

True

T/F: Streptomycin and Kanamycin are reserved for TB and Plague

True

T/F: Sulfonamides are never used for Rickettsia

True

T/F: Synercid (Quinupristin + Dalfopristin) are naturally occurring peptide-like drugs that are easily hydrolysable in the stomach, therefore has NO oral bioavailability and is available as IV only

True

T/F: Terbinafine inhibits ergosterol biosynthesis

True

T/F: Terbinafine is safe to be used in pregnancy because it is ranked as a Category B

True

T/F: Tetracyclines are amphoteric, water soluble, and have good penetration into Gram + and Gram - bacteria

True

T/F: Tetracyclines can bind to bones & teeth. Patients must be counseled to wait a few hours before taking dairy products

True

T/F: The MOA of heparin is that it binds to AT-3, which inactivates thrombin and factor Xa

True

T/F: The S-isomer of warfarin has a more profound effect on vitamin K recycling

True

T/F: The acidic pH in the gastric canaliculus is essential for the activation of PPI's

True

T/F: The difference between polymyxin B and E is that Polymyxin B is not used as inhalation treatment due to damage to the lung epithelium

True

T/F: The main goal of secondary hemostasis is to make a solid blood clot

True

T/F: The phosphate in Cidofovir generally cause less resistance in nucleosides

True

T/F: The plasma elimination half life for PPI's does not correlate to the duration of drug action

True

T/F: The washout period and the long PAE allows us to dose aminoglycosides less frequently to minimize toxicity

True

T/F: There is a link between gut flora and obesity and diseases.

True

T/F: There is cross-resistance between Clindamycin with macrolides & chloramphenicol. This means that if a patient is resistant to one, they will be resistant to the other because all of these drugs compete to bind to the same area

True

T/F: Thrombin is involved in both primary and secondary hemostasis

True

T/F: Thrombolytics are similar to t-PA in that they convert plasmin to plasminogen to degrade fibrins (both clot-bound + free)

True

T/F: Thromboxane A2 (TxA2) is released from the platelets and is responsible in forming clots

True

T/F: Ticagrelor and Cangrelor are not prodrugs and do not require hepatic activation

True

T/F: Ticlopidine and Clopidogrel have the slowest onset of action of all ADP antagonists

True

T/F: Tigecycline has 5x affinity for 30S ribosomes. It is not affected by ribosomal protection and is not susceptible to efflux pumps

True

T/F: Time-dependent killing is the amount of time exposed to exposure to produce a killing effect

True

T/F: Transposon can carry more than one resistance gene

True

T/F: Use of class I antiarrhythmics have declined due to risk of pro-arrhythmic effects particularly in patients with ischemic heart disease

True

T/F: Vancomycin is the only glycopeptide that covers C. difficile

True

T/F: Vitamin K gets oxidized by enzyme glutamyl carboxylase to make factors 2,7,9, and 10?

True

T/F: Vitamin K is fat soluble (lipophilic)

True

T/F: Voriconazole has an increased spectrum of activity compared to Fluconazole as it includes coverage over Aspergillus and Fluconazole resistant bugs

True

T/F: Warfarin blocks the enzymes that recycle vitamin K. When they are blocked, it reduces vitamin K which reduces clot factors by blocking the factors from developing

True

T/F: Warfarin blocks the formation of precursors of factors II, VII, IX, X, but does not affect the activity of already synthesized factors

True

T/F: Warfarin has a hemiketal structure

True

T/F: Warfarin is contraindicated in pregnancy due to its potential to cause fetal hemorrhage and various structural malformations. This is known as fetal warfarin syndrome

True

T/F: Warfarin is dosed based on diet. You need to have a balance of vitamin K in your diet

True

T/F: We infuse vancomycin slowly to try to prevent Phlebitis

True

T/F: We must reduce the dose of tetracyclines in patients with renal insufficiency

True

T/F: We should not use clindamycin & chloramphenicol together with macrolides because they share the same binding site and will compete with each other

True

T/F: When INR is 2-3, the risk of bleeding is less

True

T/F: When an immunocompromised patient gets a bacterial infection, it may be harder to treat which means they need a more intense regimen of antibiotics

True

T/F: Zosyn (Piperacillin + Tazobactam) is an Antipseudomonal Penicillin

True

E. coli, Klebsiella pneumoniae are the most common cause of ___

UTI

What is Cefiderocol used for?

UTI/pyelnephritis

Amanatadine and Rimanatadine are anti-influenza drugs that inhibit viral _____

Uncoating

What is Fosfomycin indicated for?

Uncomplicated lower UTI in women

Erythromycin is ____ in acid

Unstable

Which structural feature of Ceftolozane makes it highly effective against even resistant pseudomonas strains?

Urea group

Sofosbuvir is a pyrimidine of ____

Uridine

Fosfomycin is well absorbed and has high levels present in ____

Urine

Warfarin blocks what to decrease active vitamin K?

VKOR-C1

What is the prodrug of Acyclovir?

Valacyclovir

What is the prodrug of Ganciclover?

Valganciclovir

What is VISA?

Vancomycin Intermediate-Resistant Staphylococcus Aureus -Cell wall is thickened, meaning drug cannot reach target -D-Ala-D-Ala sequesters the drug

What is VRE?

Vancomycin Resistant Enterococcus

What is VRSA?

Vancomycin-Resistant Staphylococcus Aureus -D-Ala replaced with D-Lactate, decreasing affinity

Which 2 Non-DHP calcium channel blockers are used for antiarrhythmias?

Verapamil and Diltiazem

What bacteria has a "curved" appearance?

Vibrio

What type of bacteria is Vibrio cholerae and what is its appearance?

Vibrio (curved)

Which Non-Enteric Gram - Rod causes: -Chlorea

Vibrio Chlorae

Docosonol

Viral attachment and entry

Maraviroc

Viral attachment and entry

Atazanavir

Viral maturation inhibitor

Boceprevir

Viral maturation inhibitor

Darunavir

Viral maturation inhibitor

Fosamprenavir

Viral maturation inhibitor

Grazopravir

Viral maturation inhibitor

Indinavir

Viral maturation inhibitor

Nelfinavir

Viral maturation inhibitor

Paritaprevir

Viral maturation inhibitor

Ritonavir

Viral maturation inhibitor

Saquinavir

Viral maturation inhibitor

Simeprevir

Viral maturation inhibitor

Telaprevir

Viral maturation inhibitor

Tipranavir

Viral maturation inhibitor

Two categories of anti-influenza drugs

Viral uncoating inhibitor, viral release inhibitor

Degree of pathogenicity of a particular species depends on its _____

Virulence

A capsule uses its "slime layer" to surround bacteria and cause a "biofilm" layer that evades the immune system and protects it from drying out even when it leaves the body. This means the capsule is an example of a ____ ____

Virulence Factor

Degree of pathogenicity of a particular species depends on its________ _______.

Virulence factors

Coumarin derivative (Vitamin K antagonists) mimic what structure?

Vitamin K1 (phytonadione)

Which antiplatelet drug is a Thrombin Receptor (PAR-1) Antagonist?

Vorapaxar

Skin necrosis and thrombosis are adverse effects of _____

Warfarin

When is normal flora harmful?

When it is in the wrong location (not in normal area; ex: Bladder infection (E. Coli), skin puncture wounds, dental work) or in the wrong person (Immunocompromised patients)

How should capsule Itraconazole be taken?

With food or acidic drinks such as coca-cola

Why are proteins C and S dependent of vitamin K?

Without enough vitamin K we can not make enough of Protein C

What is the new anti-influenza drug that comes as a single dose treatment used for symptomatic relief against influenza symptoms? (Patients will find relief in 2.3 days)

Xofluza

Which Enteric Gram - Rod causes: -Y. pestis (Plague) -Y. Enterocolitica (GI issues from uncooked meat)

Yersinia

Three viral release inhibitors for influenza

Zanamavir, oseltamivir, peramivir

Amanatadine and Rimantadine enter the cell while Zanamivir and Oseltamivir does not. Why?

Zanamivir and Oseltamivir are polar and lipophilic

Seven anti HIV Nucleoside/Nucleotide analogs (4a)

Zidovudine, Stavudine, Emtricitabine, Didanosine, Abacavir, Lamivudine, Tenofovir

What is Pathogenicity?

ability to cause disease

Suffix for Anti-HSV Nucleoside/Nucleotide analogs

clovir / uridine

What is the abbreviation for Stavudine?

d4T

What is the cause of resistance to Ethambutol?

embB gene; efflux pumps

How do spores contribute to virulence?

has all genetic information that us harmful, but only when alive

How do fimbriae contribute to virulence?

helps bacteria attach to mucous membranes

How do flagella contribute to virulence?

helps bacteria move

What does LPS mean?

lipopolysaccharide

Gut flora protects from ___ bacteria.

pathogenic

Suffix for anti HepC Viral maturation inhibitors

previr / asvirs

How do surface glycoproteins contribute to virulence?

protects bacteria from the immune system

What is the cause of resistance to Rifampin?

rpoB alteration

Gut flora ___ and ___ the immune system.

stimulate and train

Enfivuritide

viral attachment and entry

What are examples of disease that normal flora can cause in immuno-compromised patients?

•AIDS pts •Cancer pts under chemotherapy •Transplant pts

What are examples of disease that normal flora can cause if it escapes its normal location?

•Bladder infection (E. coli) •Skin puncture wounds •Dental work


Kaugnay na mga set ng pag-aaral

Personal Finance- Chapter 14- Investing in Stocks and Bonds

View Set

In preparation for Interpersonal Communication Midterm

View Set

Lesson 2: The Cash Flow Statement: Linkages and Preparation Results

View Set

ECON 1201 Johnson HW and Quizzes

View Set

Chapter 4 (Mutual Funds and Other Investment Companies): Section 4.3, 4.4, 4.5 and 4.6

View Set